theories 3: exam 3 (version 2.0)

Ace your homework & exams now with Quizwiz!

Which type of antidepressants are rarely fatal in overdose? SSRIs Atypical MAOIs Tricyclics

SSRIs Explanation: SSRIs are rarely fatal in overdose, but cyclic and MAOI antidepressants are potentially fatal. Prescriptions may need to be limited to only a 1-week supply at a time if concerns linger about overdose.

A nurse is caring for a client diagnosed with schizophreniform disorder. The nurse demonstrates understanding of this disorder when identifying that the client is at risk for developing what? Schizophrenia Substance abuse Personality disorder Major depression

Schizophrenia Explanation: About one third of the individuals with schizophreniform disorder recover with the other two thirds developing schizophrenia. Schizophreniform disorder is not associated with the development of personality disorder, major depression, or substance abuse.

which of the following score ranges is best associated with the Glasgow Coma Scale?

Score of 3 to 15 The total GCS score is the sum of the numeric values assigned to each category assessed. A score of 15 is the highest awarded to a fully alert person; the lowest possible score is 3. A GCS score of less than 8 often indicates coma

Glasgow Coma Scale (GCS)

Scores range from 3 (deep coma) to 15 (normal) Eye-opening response 1- none 2- to pain 3- to voice 4- spontaneous Best verbal response 1- none 2- incomprehensible sounds 3- inappropriate words 4- confused 5- oriented Best motor response 1- none 2- extension 3- flexion 4- withdraws 5- localizes pain 6- obeys command

prevention and management of anaphylaxis

Screen and prevent Treat respiratory problems, oxygen, intubation, and cardiopulmonary resuscitation as needed Epinephrine 1:1000 subcutaneously Autoinjection system: EpiPen May follow with IV epinephrine IV fluids

A psychiatric-mental health nurse is conducting a pharmacology review class for a group of nurses. The topic is antidepressant medications. The nurse determines that the review was successful when the group identifies which class of antidepressant as associated with fewer side effects? Tricyclic antidepressants (TCAs) Selective serotonin reuptake inhibitors (SSRIs) Serotonin norepinephrine reuptake inhibitors (SNRIs) Monoamine oxidase inhibitors (MAOIs)

Selective serotonin reuptake inhibitors (SSRIs) Explanation: Of the classes listed, SSRIs tend to be safer and have fewer side effects than the other medications, such as TCAs, MAOIs, and SNRIs.

ALS prognosis

Survival time from onset is 25 months. Some say about 3-5 years max. Death occurs from infection, respiratory failure or aspiration.

epilepsy and status epilepticus

a group of syndromes Status Epilepticus: - prolonged seizures - lorazepam - most commonly is a complex seizure or a generalized seizure

persistent vegetative state

a type of coma in which the patient exhibits alternating sleep and wake cycles. their pons and their cerebellum is still intact, so they still wake up and go to sleep and look around. they are devoid of cognitive function.

management of spinal cord injury

acute: goal to prevent secondary injury, watch for progression. C-collar.

how is a definitive diagnosis made for iron deficiency anemia

bone marrow aspirate

Von Willebrand Disease

common bleeding disorder affecting males and females equally - usually inherited as a dominant trait. Disease is caused by a deficiency of vWF which is necessary for factor VIII activity. vWF necessary for platelet adhesion at the site of vascular injury.

latex allergy has been ____________ due to the use of nonlatex gloves

decreasing

type 4: delayed or cellular reaction examples

dermatitis graft-versus-host disease Hashimoto's thyroiditis sarcoidosis

patient education neutropenia

discuss hand washing; avoiding infected individuals; no fresh flowers, vegetables, pets, items with free standing water, avoid those receiving immunizations

SSD is more common in which sex

female

Which of the following is most closely related to a delusion?

fixed, false belief Delusions are false, fixed beliefs.

erythrocytes also need ______________ and _________ for DNA maturation

folate B12

basilar skull fracture

fracture at the base of the skull Raccoon eyes (periorbital ecchymosis) and Battle's sign (mastoid ecchymosis).

platelets

fragments of Megakaryocytes form plug over injury activate clotting factors

nursing management of encephalitis

frequent and ongoing assessment supportive care

Posturing: Decerebrate

hands at sides away from body; usually indicative of severe trauma such *brain stem damage*

spinal cord injury: primary

initial trauma

HIV stages: unknown

no information on CD4 count

two types of myasthenia gravis are

ocular generalized

what does it indicate when bands "shift to left?"

sign of infection

why should two people who are HIV+ not have sex with eachother

there is more than 1 type of HIV

Factor V Leiden treatment

unfractionated heparin or low molecular weight heparin or warfarin

sickle cell: what kinda compress

warm for vasodilation

The nurse observes a client with schizophrenia sitting alone, laughing occasionally, and turning his head as if listening to another person. The nurse assesses this behavior to indicate that the client is experiencing auditory hallucinations and says ... "Are you hearing something?" "It's a beautiful day, isn't it?" "Would you like to go to your room to talk?" "Would you like to take some of your PRN medication?"

"Are you hearing something?" Explanation: Asking the client if he is hearing something validates the nurse's assessment and focuses on the client's experience. Answers B, C, and D do not address the situation of the client experiencing auditory hallucinations at the present time.

brain abscess manifestations

*HA worse in the morning* *Fever* *Vomiting* *Neurologic deficits* *S/sx inc ICP* (dec LOC & seizures)

nursing care of hemorrhagic stroke

- report all changes, literally all changes - monitor ICP - fluid balance

big thing for Factors

- when giving Factor, treat it like you're giving heparin

common somatic symptom disorder complaints

1. PAIN is typically number one complaint - fatigue - dizziness - insomnia - SOB

How long does it take to make a RBC?

5 days

A nurse is caring for a patient diagnosed with a hemorrhagic stroke. When creating this patient's plan of care, what goal should be prioritized? A) Prevent complications of immobility. B) Maintain and improve cerebral tissue perfusion. C) Relieve anxiety and pain. D) Relieve sensory deprivation.

B) Maintain and improve cerebral tissue perfusion.

Two types of meningitis

Bacterial (septic) Viral (aseptic)

_____________ produce and store histamine as well as other substances involved in hypersensitivity reactions.

Basophils

HIV stages: stage 3

CD4 count drops below 200 - considered to have AIDS for surveillance purposes

brain abscess diagnosis

CT scan or MRI

Stages of HIV: stage 0

Early HIV infection, inferred from lab testing (first 6 weeks of exposure, flu like symptoms)

assessment of patients with allergic disorders

History and manifestations; comprehensive allergy history Diagnostic tests: oCBC: eosinophil count oTotal serum IgE oSkin tests: prick, scratch, and intradermal

a nurse in a provider's office is assessing a client who has rheumatoid arthritis (RA). which of the following findings is a late manifestations of this condition? anorexia knuckle deformity low grade fever weight loss

Knuckle deformity Joint deformity is a late manifestation of RA.

active acquired immunity

Made yourself after exposure to antigen

A client diagnosed with schizoaffective disorder exhibits the symptoms of a psychosis in addition to which of the following? Substance use disorder Mood disorder Thought disorder Personality disorder

Mood disorder Explanation: The client exhibits the symptoms of psychosis and, at the same time, all the features of a mood disorder, either depression or mania.

bacterial (septic) meningitis cause

N. meningitidis

what is used to assess patients that are suspected of having a stroke

NIHSS stroke scale 0 = normal 42 = severe stroke

stroke types

- ischemic (80-85% of the time) - hemorrhagic (15-20%)

care of ALS

- mobility care - supportive care - preventing contractures and wounds

ICP normal

0-10 mmHg (15 is the upper limit)

Cushing's triad consists of:

1. Elevated systolic BP (widening pulse pressure, systolic is far from diastolic number with HIGH MAP) 2. Bradycardia 3. Irregular respirations or bradypnea

allergic rhinitis (hay fever)

A common respiratory allergy presumed to be mediated by a type I hypersensitivity Affects 12% of adults Major symptoms: copious amounts of serous nasal discharge nasal congestion sneezing nose and throat itching

Cold Urticaria

A condition characterized by redness, itching, and large, blisterlike wheals on skin that is exposed to cold.

polycythemia

A disorder characterized by an abnormal increase in the number of red blood cells in the blood

Angioedema

Allergic reaction involving the infiltration of fluid in the subq tissue and mucous membranes resulting in diffuse swelling - brawny, nonpruritic, widespread, nonpitting edema - usually lips, eyelids, cheeks, hands, tongue

Which of the following interventions is most likely to be helpful for patients with schizophrenia?

Assistance with self-care Schizophrenic patients often forgo hygiene and self-care, as they are distracted with their symptoms. Thus, assisting patients with their self-care can improve mental status and social functioning. We want these patients to be able to perform their ADLs independently

The nurse is caring for a client admitted with a diagnosis of septic meningitis. The nurse is aware that this infection is caused by which of the following? Bacteria Virus Lymphoma Leukemia

Bacteria Explanation: Septic meningitis is caused by bacteria. In aseptic meningitis, the cause is viral or secondary to lymphoma, leukemia, or human immunodeficiency virus.

education for newly diagnosed rheumatic disease

Explain the disease and principles of disease management Medication teaching and safe self-administration Pain management techniques Cope with stress Dietary plan including vitamin supplementation Identify need for health promotion, prevention, and screening Community resources and sources of support

A client has been taking haloperidol for 5 years when the client is admitted to the inpatient unit for relapse of symptoms of schizophrenia. Upon assessment, the client demonstrates akathisia, dystonia, a stiff gait, and rigid posture. The nurse correctly identifies these symptoms are indicative of what? Psychosis Progressed schizophrenia Extrapyramidal side effects Tardive dyskinesia

Extrapyramidal side effects Explanation: Extrapyramidal side effects include severe restlessness, muscle spasms, or contractions; chronic motor problems such as tardive dyskinesia; and the pseudoparkinsonian symptoms of rigidity, masklike faces, and stiff gait.

A client has been diagnosed with a frontal lobe brain abscess. Which nursing intervention is appropriate? Assess for facial weakness. Assess visual acuity. Ensure that client takes nothing by mouth. Initiate seizure precautions.

Initiate seizure precautions. Explanation:

Which cell of hematopoiesis is responsible for the production of red blood cells (RBCs) and platelets? Monocyte Lymphoid stem cell Neutrophil Myeloid stem cell

Myeloid stem cell Explanation: The myeloid stem cell is responsible not only for all nonlymphoid white blood cells, but also for the production of red blood cells and platelets. Lymphoid cells produce either T or B lymphocytes. A monocyte is large WBC that becomes a macrophage when is leaves the circulation and moves into body tissues. A neutrophil is a fully mature WBC capable of phagocytosis.

A child is brought to the clinic with a rash and is subsequently diagnosed with measles. The parent reports also having had measles as a young child. What type of immunity to measles develops after the initial infection? Naturally acquired active immunity Artificially acquired passive immunity Naturally acquired passive immunity Artificially acquired active immunity

Naturally acquired active immunity Explanation: Immunity to measles that develops after the initial infection is an example of naturally acquired active immunity. Artificially acquired active immunity results from the administration of a killed or weakened microorganism or toxoid (attenuated toxin), whereas passive immunity develops when ready-made antibodies are given to a susceptible client.

meds for allergic reactions

Oxygen, if respiratory assistance is needed Epinephrine used for anaphylactic reactions Histamines Corticosteroids

febrile non-hemolytic reaction

Patients have antibodies to the leukocytes in the blood. - Most common type of reaction. - Most commonly in people who have previous transfusion. - Rh negative woman with Rh positive children (exposure to Rh + fetus raised antibody levels in untreated mother). Signs - chills, fever (more than 1 degree c). Fever begins within 2 hours of transfusion begins. May premedicate

A nurse is transfusing whole blood to a client with impaired renal function. During the transfusion, the client tells the nurse, "I feel very short of breath all of a sudden." What is the nurse's primary action? Slow the infusion. Stop the infusion. Call the health care provider. Assess the client's vital signs.

Stop the infusion. Explanation: A client with impaired renal function is at increased risk for transfusion-associated circulatory overload (TACO). Signs of circulatory overload include dyspnea, orthopnea, tachycardia, an increase in blood pressure, and sudden anxiety. If the symptoms are mild, the nurse may be able to slow the infusion and administer diuretics; however, sudden shortness of breath should clue the nurse to immediately stop the infusion and sit the client upright with feet dangling. Next, the nurse will notify the health care provider after normal saline is infused into the site. Only after stopping the infusion will the nurse obtain the client's vital signs.

The nurse assists the health care provider (HCP) in completing a lumbar puncture (LP). Which should the nurse note as a concern? The HCP administers a drug by intrathecal injection. The cerebrospinal fluid (CSF) is cloudy in nature. The pressure is noted to be 90 mm H20. The HCP maintains aseptic procedure.

The cerebrospinal fluid (CSF) is cloudy in nature. Explanation: The CSF is normally clear and colorless; therefore, CSF that is cloudy would be noted by the nurse as a concern. The HCP is correct to maintain aseptic procedure. At 90 mm H20, the client's CSF fluid pressure falls within normal limits (between 80 and 100 mm H20). Sometimes the HCP will administer medication via intrathecal injection during an LP, which should not be a cause for concern.

Factor V Leiden mutation

The most common inherited cause of hypercoagulability.

When teaching about the advantages of autologous blood transfusion to a client, the nurse should include which information? Select all that apply. If not needed immediately, the blood can be frozen for future use. Blood can be transfused to family members and close relatives. The primary advantage is prevention of viral infections. It is safer for clients with a history of transfusion reactions. It resolves anemia for clients with a hemoglobin less than 11g/dL.

The primary advantage is prevention of viral infections. It is safer for clients with a history of transfusion reactions. If not needed immediately, the blood can be frozen for future use.

treatment of parkinsons disease

Treatment is to control symptoms and individualized by restoring dopamine levels. Dopamnine does not cross blood brain barrier, so we don't just give them dopamine. Give them precursor med called levodopa. Not a curative disease. Looking at other treatment options

A schizophrenic patient presents to your clinic stating that he saw a rainbow unicorn gallop into his garage this morning. Which of the following best describes this type of hallucination?

Visual When the patient states that they are seeing persons or things

A client is scheduled for CT scanning of the head because of a recent onset of neurologic deficits. What should the nurse tell the client in preparation for this test? A."No metal objects can enter the procedure room." B."There will be a lot of noise during the test." C."You need to fast for 8 hours prior to the test." D."You will need to lie still throughout the procedure."

You will need to lie still throughout the procedure.

The client is diagnosed with schizophrenia and the nurse is observing for effects of medication during the teaching session. Which medication rarely causes extrapyramidal side effects (EPS)? Fluphenazine Ziprasidone Haloperidol Chlorpromazine

Ziprasidone Explanation: First-generation antipsychotic drugs cause a greater incidence of EPS than do atypical antipsychotic drugs, with ziprasidone rarely causing EPS. Chlorpromazine, haloperidol, and fluphenazine are all first-generation antipsychotic drugs.

Kernig's sign

a diagnostic sign for meningitis marked by the person's feelings of pain when the leg/knee is brought up towards the chest when the patient is sitting or lying down

seizure

abnormal episodes of motor, sensory, autonomic, or psychic activity (or a combination of these) resulting from a sudden abnormal uncontrolled electrical discharge from cerebral neurons

which of the following is considered a "negative" symptom in patients with schizophrenia?

absence of something that should be present patients display a lack of something that should be present. this includes a lack of interest in hygiene, motivation, or even the ability to experience pleasure. this can be described as anhedonia.

A nurse is teaching a client with a long leg cast how to use crutches properly while descending a staircase. The nurse should tell the client to transfer body weight to the unaffected leg, and then: advance the affected leg. advance both legs. advance the unaffected leg. advance both crutches.

advance both crutches. Explanation: The nurse should instruct the client to advance both crutches to the step below, then transfer his body weight to the crutches as he brings the affected leg to the step. The client should then bring the unaffected leg down to the step.

a nurse is performing an admission assessment for a client who has schizophrenia. which of the following findings should the nurse identify as a negative symptom? affective flattening bizarre behavior illogicality somatic delusions

affective flattening

Polyarticular arthritis

affects multiple joints

Monoarticular arthritis

affects one joint

IVIG has an extremely high risk of

anaphylaxis

a nurse in a mental health clinic is conducting a staff education session on schizophrenia. which of the following manifestations should the nurse identify as a negative symptom? (SATA) delusions hallucinations anhedonia poor judgement blunt affect

anhedonia blunt affect

A psychiatric-mental health nurse is assessing a client who is suspected of experiencing depression. During the interview, the client says, "I just don't care any more. I used to enjoy doing all sorts of things outdoors, but now, I don't. Nothing seems to make me happy." The nurse interprets this statement as: aphasia. labile mood. anhedonia. affect.

anhedonia. Explanation: The patient's statement reflects a loss of interest or pleasure in activities that previously brought enjoyment . This is termed anhedonia. Labile mood is the outward emotional expression that is varied, rapid, and abruptly shifts. Affect is the outward emotional expression of a person that gives clues to the person's mood. Aphasia involves a difficulty with speaking or communicating.

Which of the following signs, symptoms, or diseases is most commonly seen when evaluating a patient for the "5 A's" of Alzheimer's disease? Agnathia Arthritis Agoraphobia Acrocephaly Anomia Acidemia

anomia Anomia is an inability to remember names of things.

once patient is determined to be a candidate for tPA, what medication / devices must be held for the next 24 hours after tPA has been given?

anticoagulants / antiplatelet medications it is also recommended to delay the placement of NGT, foleys, and intra-arterial catheters for the first 24 hours.

treatment of HIV and AIDS

antiretroviral therapy (ART) the goal is to suppress the replication of the HIV virus (this decreases mortality) to hopefully stay in stage 1 - Overarching goal to suppress HIV replication - Reduce HIV-associated morbidity and prolong duration and quality of life - Restore and preserve immunologic function - Maximally and durably suppress plasma HIV viral load - Prevent HIV transmission

comorbities for somatic symptoms disorder

anxiety and depression

Which of the following signs, symptoms, or diseases is most commonly seen when evaluating a patient for the "5 A's" of Alzheimer's disease? Acrocephaly Adenomyosis Aphasia Achalasia Abdominal Pain Achondroplasia

aphasia Aphasia is the inability to express oneself through speech. This may include a loss of the ability to speak or understand spoken, written, or sign language.

myasthenia gravis

autoimmune disorder of the myoneural junction antibodies directed at acetylcholine at the myoneural junction

Which of the following interventions is most likely to be helpful for patients with schizophrenia?

build trust Often these patients suffer from delusions, so it is important to build trust and rapport with the patient. It is through this relationship that you can better help the patient.

sickle cell anemia

cell shape causes ischemia/pain/crisis because it causes hemoglobin molecule to be defective. - can cause Acute Chest Syndrome or Splenic Sequestration - happens when O2 is low and cells lose its shape - ALWAYS ANEMIC - normally they will have pulmonary hypertension

a nurse is assessing a client who has illness anxiety disorder. which of the following findings should the nurse expect? prior physical health followed by the need for two surgeries within the last three months obsession over a fictitious defect in physical appearance sudden unexplained loss of peripheral sensation constant worry about the undiagnosed presence of an illness

constant worry about the undiagnosed presence of an illness

medical management of brain abscess

control ICP drain abscess

VWD type 1

decreased in structurally normal, Type 2 variable qualitative defects based on the specific vWF subtype involved.

Which of the following is considered a "positive" symptom in patients with schizophrenia?

delusions Patients can have delusions, which are false beliefs or misinterpretations of events and their significance.

T cells or T lymphocytes assist B cells and secrete substances that ____________________________________. They also digest antigens and remove debris.

destroy target cells and stimulate macrophages

Increase in ______________ levels in allergic states indicated that these cells are involved in the hypersensitivity reaction; they neutralize histamine.

eosinophil

diagnostic findings of lupus

erythematous rashes hyperpigmentation sensitivity to light alopecia mouth ulcers pericardial friction rub pleural effusions joint swelling, tenderness, warmth, painful movement, stiffness, edema behavioral changes depression, seizures, chorea (spastic jerky movements of hands, legs, face) Antinuclear antibody Anti-DNA Anti-Sm

Labs f/diagnostics or rheumatoid arthritis

erythrocyte sedimentation rate and CRP elevation Synovial fluid analysis. X ray and MRI of joints to see erosions and track disease progression Rheumatoid factor titer, CRP, Hgb tb test hepatitis b and c test liver monitoring and kidney monitoring for DMARD meds

Myeloid line

erythrocytes all leukocytes except lymphocytes megakaryocytes

which of the following is considered a negative symptom in patients with schizophrenia?

flat affect patients with schizophrenia may not show the signs of normal emotion, and may have diminished facial expressions. they may appear extremely apathetic and can speak in a monotonous voice.

Leukocytes

granulocytes monocytes

Which of the following is considered a "positive" symptom in patients with schizophrenia?

hallucinations Patients with schizophrenia have hallucinations, which are typically auditory. This often is reported as hearing voices, speaking to the patient, but can also present as music, body noises or machinery. While rare, visual, somatic, olfactory and gustatory hallucinations can also be experienced.

Posturing: Decorticate

hands pulled up to the core of the body **turning arms to the "core" of the body

neurostorming

happens when patients are taken off of sedation to see to see what kind of neuro function is there. Pt will be very agitated, tachycardic.

classic indication of a brain abscess

headache that is worse in the morning fever signs of ICP

manifestations of encephalitis

headache, fever, confusion, changes in LOC vector-borne: rash, flaccid paralysis, Parkinson's-like movement

assessement of immune system

health history, nutrition, infections, immunizations, allergies, autoimmune disorders, cancer, chronic illness -lymph node assessment

mast cells or basophils release

histamine, serotonin, kinins, SRS-A, and neutrophil factor

Reticulocyte

immature red blood cell

a nurse is providing teaching for a client who has schizophrenia and a new prescription for risperidone. which of the following statements should the nurse include in the teaching? increase your fluid and fiber intake to prevent constipation have your blood pressure checked frequently for hypertension expect to have your blood checked weekly for serum electrolyte imbalances increase caloric intake to prevent weight loss

increase caloric intake to prevent weight loss

FFP (fresh frozen plasma)

indications: pts who need coagulation (clotting) factors - liver failure, DIC, massive RBC transfusion *Product of choice for pts with multiple deficiencies*

What types of cells are the primary targets of the healthy immune system? Select all that apply. cancerous cells typical cells infectious cells foreign cells

infectious cells foreign cells cancerous cells

physical barriers in natural immunity

intact skin, chemical barriers, and acidic gastric secretions or enzymes in tars and saliva

monocytes

largest white blood cell! An agranular leukocyte that is able to migrate into tissues and transform into a macrophage.

what can you give in order to help with opioid tolerance to help with pain control that is IV?

low dose, continuous Naloxone drip

SSD is more common in what educational status

low or lack of education

Vitamin B and folic acid deficiencies are characterized by production of abnormally large erythrocytes called megaloblasts. blast cells. monocytes. mast cells.

megaloblasts. Explanation: Megaloblasts are abnormally large erythrocytes. Blast cells are primitive WBCs. Mast cells are cells found in connective tissue involved in defense of the body and coagulation. Monocytes are large WBCs that become macrophages when they leave the circulation and move into body tissues.

Illusions are defined by which condition?

misinterpretation of stimulus Illusions are a misperception of a real experience. Ex: Being out in the desert with a hot sun and seeing an oasis of water just a few feet in front of you, when in reality all that is seen is desert.

What best describes a consideration when managing a schizophrenic patient with pharmacotherapy?

monitor for tardive dyskinesia A very common side effect of antipsychotic medications is tardive dyskinesia. This is described as stereotypic oral-facial (lip-smacking) and other motor movements. These are potentially irreversible. You can track involuntary movements using the AIMS (abnormal involuntary movement scale) test.

Which of the following is a most likely a component of the Glasgow Coma Scale (GCS)?

motor response Motor response is rated 1 to 6 points. Examples of how to rate motor response: They are moving limbs normally with no pain (6). With a painful stimulus, such as a trapezius pinch, the patient brings a hand above the clavicle to the site of physical stimulus (5). With a painful stimulus, the patient displaying normal flexion (e.g. rapid, variable, away from body) without abnormality in movement (e.g. slow stereotyped, rotation of forearm, leg extends with painful trapezius stimulus) is withdrawing in response to pain (4). If they have decorticate posturing which is curling of the limbs towards the spinal cord or "towards dee cord" as we sometimes say (3). If they have decerebrate posturing which is curling of the limbs away from the spinal cord (2). And lastly, if they have no motor response (1). 6 = normal, 5 = localized to pain, 4 = withdraws to pain, 3 = decorticate posture, 2 = decerebrate posture, 1 = none.

bacterial meningitis needs to be put in a

negative pressure room

Causes of Neurological Dysfunction

neuro: stroke, injury (will have pupillary response) toxic: drug OD, alcohol withdrawal, DTs metabolic: renal/hepatic failure, DKA

A nursing instructor is giving a lecture on the immune system. The instructor's discussion on phagocytosis will include: lymphokines and suppressor T cells. regulator T cells and helper T cells. neutrophils and monocytes. plasma cells and memory cells.

neutrophils and monocytes. Explanation: Neutrophils and monocytes are phagocytes, cells that perform phagocytosis.

functional mobility interventions for rhematic disease

oAssess for need of PT/OT oEncourage independence in mobility

self care interventions for rheumatic disease

oAssist in identifying self-care deficits and factors that interfere with ability to perform self-care activities oProvide assistive devices oConsult with community agencies

Physical and psychological changes interventions for rheumatic disorders

oAssist to identify elements of control over disease oEncourage verbalization of feelings/fears

fatigue interventions for rheumatic disorders

oExplain energy‐conserving techniques oFacilitate development of activity/rest schedule

effective coping behaviors for rheumatic disorders

oIdentify areas of life affected by disease oDevelop plan for managing symptoms and enlisting support of family and friends to promote daily function

problems of rheumatic diseases

oLimitations in mobility and activities of daily living oPain and fatigue oAltered self-image oSleep disturbances oSystemic effects that can lead to organ failure and death

Allergic rhinitis: improving breathing

oModify the environment to reduce allergens oReduce exposure to people with URI oTake deep breaths and cough frequently

absence of complications secondary to medications interventions for rheumatic disease

oPerform periodic clinical assessment and laboratory evaluation oProvide education about correct self-administration, potential side effects, and importance of monitoring oCounsel regarding methods to reduce side effects and manage symptoms oAdminister medications in modified doses as prescribed if complications occur

pain interventions for rheumatic disorders

oProvide comfort measures oAdminister anti-inflammatory, analgesic medications

goals for rheumatic disease

oRelief of pain and discomfort oRelief of fatigue oMaintain optimal functional mobility oPerforms self-care activities independently oAdapts to physical and psychological changes oEffective coping behaviors oAbsence of complications

planning and goals for patient with allergic rhinitis

oRestoration of normal breathing pattern oIncreased knowledge about the causes and control of allergic symptoms oImproved coping with alterations and modifications oAbsence of complications

premonitory phase of migraine

occur hours to days before a migraine - depression - irritability - feeling cold - food cravings - anorexia - increased urination - GI problems

headache (cephalagia)

one of the most common complaints in medicine - 2 kinds: primary secondary - can cause significant discomfort and interfere with activities and lifestyle

white blood cell action in natural immunity

orelease cell mediators such as histamine, bradykinin, and prostaglandins and engulf (phagocytize) foreign substances

noninflammatory arthritis

osteoarthritis (OA) -is not systemic

postdrome phase of migraine

pain gradually subsides - tiredness - weakness - cognitive difficulties - mood changes for hours/days muscle contraction in neck and scalp may sleep for extended amt of time

locked-in syndrome

paralyzed cannot speak may use eyes to communicate their cognition is there, but they cannot communicate with anything but eyes common with ALS

most common food allergy

peanut allergy

B cells or B lymphocytes are programmed to produce one specific antibody and stimulates production of

plasma cells and antibody production resulting in the outpouring of antibodies

nursing management of CJD

prevention of disease transmission; blood and body fluid precautions supportive care

myasthenic crisis or cholinergic crisis priority

secure that airway and breathing!!!!!! -oxygenate -intubate then give antidote for cholinergic crisis

headache phase of migraine

severe and incapacitating often associated with: - photophobia - phonophobia (sound sensitivity) - allodynia (abnormal perception of innocuous stimuli) -N/V Vasodilation that causes the pain. There is a decrease serotonin Can last a while

what kind of pain is common early on in parkinson's disease?

shoulder pain due to rigidity

Early onset RA treatment

single DMARD

iron deficiency anemia sx

smooth sore tongue, brittle nails, fatigue, pallor or jaundice, cardiac and respiratory symptoms, angular cheilitis, PICA

aplastic anemia

so much damage to stem cells that myeloids are no longer making RBCs

a nurse in an acute mental health facility is caring for a client who jumps out of her chair and begins to shout angrily at the clients around her. which of the following actions should the nurse take first? call for assistance to place the client in restraints escort the client to an unlocked seclusion room offer the client a PRN antianxiety medication speak to the client calmly, giving simple directions

speak to the client calmly, giving simple directions

acquired immunity

specific against a foreign antigen that they have had prior exposure to

problem with directed donations is

specifically donated from one person to another. No safer because someone may not admitted to something

Bells Palsy

steroids patient education (close eye and put patch over it, use eye drops, sunglasses)

Disease Acquisition Blood Transfusion

still possible very rare Hepatitis B or C HIV/AIDS Cytomegalovirus chronic graft vs. host disease Creutzfeldt-Jakob disease

chronic subdural hematoma can be mistaken for

stroke

spinal shock

sudden depression of reflex below injury •Muscular flaccidity, lack of sensation and reflexes

During a routine physical examination to assess a client's deep tendon reflexes, a nurse should make sure to: tap the tendon slowly and softly. hold the reflex hammer tightly. support the joint where the tendon is being tested. use the pointed end of the reflex hammer when striking the Achilles tendon.

support the joint where the tendon is being tested. Explanation: The nurse should support the joint where the tendon is being tested to prevent the attached muscle from contracting. The nurse should use the flat, not pointed, end of the reflex hammer when striking the Achilles tendon. (The pointed end is used to strike over small areas, such as the thumb placed over the biceps tendon.) Tapping the tendon slowly and softly wouldn't provoke a deep tendon reflex response. The nurse should hold the reflex hammer loosely, not tightly, between the thumb and fingers so it can swing in an arc.

Transischemia attack (TIA)

temporary brain ischemia; warning sign for more serious CVAs

concussion

temporary loss of function no treatment rouse frequently observation

PHQ-15

the Patient Health Questionnaire 15 is the somatization module of the PHQ. - comprises 15 somatic symptoms that correspond to both the most common physical symptoms in outpatients and the most important DSM-5 criteria for somatization disorder. - consists of 13 items from the Somatoform Disorders module and 2 items from the Depressive Disorders modules.

allergen

the substance that causes the allergic response

Assertiveness training in SSD

this is huge for SSD! they try to teach the healthy expression of feelings and make your needs known.

what is the effect that B12 and folate has on RBCs?

too little creates large megaloblasts that cannot leave the bone marrow

if encephalitis is fungal

treat with antifungal or amphotericin (amphotericin B)

If the Bone marrow is still the site of erythropoiesis - then an erythroblast (former myeloid) is stimulated by erythropoietin (hormone produced in kidney). True or False?

true.

type 3: immune complex reaction symptoms

urticaria joint pain fever rash adenopathy (swollen glands)

goal time for diagnostic imaging results for strokes

within 25 mins of arrival to ED

does epi have a rebound

yes normally in 4 hours or so so take the person to the ER

if patient has a bone flap:

you need to be cautious about where the bone flap is, as you don't normally turn them onto the side that the flap is. patients are often given helmets if they go to rehab with bone flap.

skull fractures classifications

•4 Classifications •Localized, persistent pain •Fractures of the base of the skull •Bleeding from nose pharynx or ears •Battle's sign •Halo sign •Raccoon Eyes •Treatment Monitoring

Anemias: Manifestations of anemia

•Fatigue, weakness, malaise •Pallor or jaundice •Cardiac and respiratory symptoms (angina, shortness of breath, heart failure) •Tongue changes •Nail changes •Angular cheilitis •Pica

contusion

•Moderate to severe BI - brain bruised. Impact of brain on skull leaves contusion; may cause hemorrhage •Loss of consciousness associated with stupor and confusion •Peaks injury 18 - 36 hours

spinal cord injury nursing care

•Monitor carefully to detect potential respiratory failure •Pulse oximetry and ABGs •Lung sounds •Early and vigorous pulmonary care to prevent and remove secretions •Suctioning with caution •Breathing exercises •Assisted coughing •Humidification and hydration •Maintain proper body alignment •Turn only if spine is stable and as indicated by physician •Monitor blood pressure with position changes •PROM at least four times a day •Use neck brace or collar, as prescribed, when patient is mobilized •Move gradually to erect position

Primary Polycythemia Vera treatment

•phlebotomy, (500 mL or once or twice a week) •chemotherapeutic agents •platelet aggregation inhibitors interferon

The nurse is working with a client who has been diagnosed with depression. When performing a strength assessment with the client, what is the nurse's best statement or question? "Do you consider yourself to be a strong person overall?" "How have you dealt with feelings like this in the past?" "It's important that you remember that you're an exceptionally strong and capable person." "What can the care team do to help you become a stronger person?"

"How have you dealt with feelings like this in the past?" Explanation: A strength assessment is aimed at identifying the client's previous coping skills and strategies. Seeking suggestions for the care team does not achieve this purpose. Asking if the client considers himself or herself to be strong is likely to be answered with "no" in a person who is depressed. The nurse must avoid false reassurance.

the nurse suspects that a client is experiencing somatic symptom disorder (SSD). Which question should the nurse include in the assessment process to determine specific gastrointestinal (GI) symptoms? "When did you first notice your legs aching?" "How often do you experience headaches?" "How many episodes of diarrhea do you have each week?" "When did you first notice pain with intercourse?"

"How many episodes of diarrhea do you have each week?" Explanation: Somatic symptom disorder (SSD) is one of the most difficult disorders to manage because its symptoms tend to change, are diffuse and complex, and vary and move from one body system to another. Although all these assessment questions focus on SSD symptoms, it is only the question "How many episodes of diarrhea do you have each week?" that addresses GI symptoms. Headache is a neurologic symptom, aching legs is a musculoskeletal symptom, and pain with intercourse is a sexual issue.

Dermatitis Medicamentosa (Drug Reactions)

*A type I hypersensitivity disorder causing skin rashes associated with certain medications *Avoidance (discontinuation) of the offending medication is the priority measure

what sign for meningitis is a more sensitive indicator that Kernig's sign?

+Brudzinski sign

transfusion administration

- 2nd nurse to verify in room - primary nurse remains in the room for the first 15 minutes to monitor for reactions - cannot last longer than 4 hours

DIC treatment

- Anticoagulant therapy (Heparin) - Replacement therapy: Transfusion of platelets, FFP, or concentrations of coagulation factors - correct tissue ischemia - replace fluids and electrolytes - maintain blood pressure - Priorities are circulation and oxygenation Most effective nursing intervention is anticipation of DIC in any patient with infection, trauma, cancer, or obstetric emergency!!

HIV/AIDS: decreasing the sense of isolation

- Assess pattern of social interaction - Observe for behaviors indicative of social isolation - Assist with identifying resources

intraoperative blood salvage

- Blood loss during surgery is collected into sterile cell saver, washed, filtered and returned.

labs for patients with hematologic disorders

- CBC with differential - Pt/PTT/INR - Bone Marrow Aspirate - Bone Marrow Biopsy

medical management of anemias

- Correct or control the cause - Transfusion of packed RBCs Treatment specific to the type of anemia: - Dietary therapy - Iron or vitamin supplementation: iron, folate, B12 - Transfusions - Immunosuppressive therapy - Other

signs and symptoms of late ICP changes

- Cushing's Triad (bradycardia, HTN with WIDE pulse pressure (systolic is far from diastolic number with HIGH MAP), bradypnea) - vomiting - change in LOC - loss of reflex - respiratory changes

clinical manifestations of HIV: neurologic

- Effects on cognition, motor function attention, visual memory, visuospatial function - Peripheral neuropathy - HIV encephalopathy - Fungal infection, Cryptococcus neoformans - Progressive multifocal leukoencephalopathy - Depression and apathy - HIV-Associated Neurocognitive Disorders (change in language, memory, problem solving, slowing psychomotor)

epidural hematoma treatment

- Extreme emergency - Burr holes to decrease ICP - Remove clot, control bleeding - May require craniotomy - Drain usually inserted after creation of burr holes

latex allergy

- Must use latex-free gloves; - Schedule surgery first thing in the morning; - Label O.R. as "latex free"; - clients w/latex allergy usually have an allergy to foods such as bananas, kiwis, and avocados

amphotericin things to know

- NOT COMPATIBLE with anything but D5W - runs over 2-4 hours - horrible s/e: rigor, seizures, lability of BP - try to give it to patient while they are asleep

tissue plasminogen activator eligibility

- Older than 18 years old - Clinical dx of ischemic stroke - Systolic bp <185, diastolic bp <110 - No minor (non disabling) stroke - Pt <15 seconds or INR <1.7 - Not received heparin in last 24 hrs - Platelet >100,000 - No s/s of infective endocarditis - No prior intracranial hemorrhage - No subarachnoid hemorrhage - No stroke, serious head trauma, or intracranial surgery within 3 months - No GI bleed within 21 days

complications of altered LOC

- Respiratory failure - Pneumonia / aspiration - Pressure ulcers - Aspiration - DVT - Contractures

if splenic sequestration is suspected, you need to:

- get US of spleen - do fluid resuscitation

PRBC donation eligibility

- greater than 110 lbs and over 17 years old - temperature <99.6, pulse 50-100, SBP 90 or 180, and DBP 50-100. - Hemoglobin 12.5 - without history of a) Hx of viral hepatitis at any time or close contact with a patient who have hepatitis or was undergoing dialysis within 6 months b) Hx of receiving blood transfusion or blood derivative (except albumin) within 12 months c) Previous transfusions in UK, Gibraltar, or Falkland Islands. Increased chance of having Mad Cow; Total stay since 1980 in UK for more than 3 months or other European country for 6 months d) Untreated syphilis/malaria. Malaria must be free of symptom and off therapy for 3 years. e) Evidence/hx of drug abuse with IV use f) Possible exposure of HIV (anal sex, multiple partners, IV drugs, sexual partners with people at risk for HIV, People with hemophilia) g) Skin infection h) Hx of asthma, urticaria or medication allergy because it can transfer i) Pregnancy j) Tooth extraction/oral surgery within 72 hours k) Untreated exposure to infectious disease within 3 weeks l) Recent immunizations m) Recent tattoo n) Cancer o) Donation within 56 days.

Primary Immune Deficiency Disorders important to remember

- hand washing - positive pressure rooms - neutropenic precautions

postop nursing management of intracranial surgery

- mannitol and dexamethasone to reduce cerebral edema - acetaminophen for pain and fever, sometimes codeine, sometimes morphine if severe - anticonvulsants (phenytoin or levetiracetam) for seizure prevention and monitor serum levels to prevent toxicity - management of ICP monitor or cerebral oxygenation monitor to prevent infection and manage ICP

epidural hemorrhage / hematoma treatment

- measures to reduce ICP - remove the clot and stop bleeding; sometimes Burr holes or Craniotomy - drain placement to prevent reaccumulation of blood

spinal cord injury: secondary

- nerves swell / disintegrate - ischemia, hypoxia, edema destruction

HIV biggest 3 things

- prevention - early detection - ongoing treatment

nursing role for migraine

- provide individualized care and treatment - prophylactic medications may be used for recurrent migraines - migraines and cluster headaches requires abortive medications instituted as soon as possible with onset - provide meds at prescribed - provide comfort measures (quiet, dark room. massage. local heat for tension) - help patient identify triggers and develop a preventative strategies and lifestyle changes for headache prevention - medication instructions and treatment regimen - stress reduction techniques - nonpharmacologic therapies - follow-up care - encouragement of healthy lifestyle and health promotion activities

Bacterial contamination blood transfusions

- puts pt at great risk. - Occurs anytime during procurement or processing. - Signs are fever, chills, and hypotension. May not occur during transfusion or several hours after transfusions.. Can lead to septic shock - as soon as these signs are recognized, STOP THE TRANSFUSION.

postop nursing assessment of intracranial surgery

- respiratory function bc even a small event of hypoxia can cause cerebral ischemia - temperature is measured to assess for hyperthermia secondary to infection or damage to hypothalamus - neuro checks to detect increased ICP - inspect dressing for bleeding or CSF - inspect incision for bulging, redness, tenderness, separation, foul order - assess for sodium retention as this is common in immediate postop period - serum and urine electrolytes, BUN, blood glucose, weight, and clinical status monitoring - I&Os - assess for seizures and report all seizure activity - expect restlessness as pt becomes more responsive - report to MD if periorbital edema increases significantly bc this can mean there is a clot or increased ICP and poor venous drainage

Concussion Management

- rest - observation - teach family to report difficulty waking or speaking, severe headache, vomiting, or one sided weakness

reducing the risk of transmission to health care providers

- standard precautions - hand hygiene - postexposure prophylaxis

nursing management for blood transfusion reactions

- stop the infusion girl!!! - then assess - notify MD - possibly obtain labs - return to patient - treat reaction

for increased ICP, DO NOT

- suction frequently - make them strain at all because this causes their brain to swell

symptoms of hemorrhagic stroke

- sudden severe headache - projectile vomiting - photosensitivity (changes in eyesight) - early and sudden change in LOC, confusion - decreased motor skills - sleepiness - random eye movements - paralysis on one side of the body - personality changes

intracerebral hemorrhage treatment

- supportive care of symptoms - control of ICP - administration of fluids, electrolytes, and antihypertensive meds - craniotomy or craniectomy to remove clot and control hemorrhage

hemolytic anemias

- thalassemia (inherited) - glucose-6-phosphate dehydrogenase deficiency (inherited) - sickle cell anemia

circulatory overload blood reaction

- too much too quickly. Sx: Dyspnea, orthopnea, tachycardia, and sudden anxiety. Symptoms of CHF. you see this in traumas when we dump blood into them.

fibromyalgia treatment

- tricylcic antidepressants or SSRIs (help with sleeping) - NSAIDs - exercise

headache causes

- vascular changes / inflammation - hereditary - diet - stress - contraception

Type 1 (Anaphylactic) Reactions symptoms

- vasodilation - increased capillary permeability - smooth muscle contraction - eosinophilia - stridor - angioedema - hypotension - bronchial, GI, or uterine spasms - hives

complications of hemorrhagic stroke

- vasospasm - seizures - hydrocephalus - rebleeding - hyponatremia - secondary ischemia

malingering

-DSM-V dropped as primary mental disorder (co-occurring only) -Motives separate from primary mental illness -Done for personal gain -Many similar behaviors to factitious disorder -May seek financial assistance from public -Can face legal charges r/t fraud -May seek disability payments or other benefits

parkinson's disease manifestations

-Do not appear until about 60% of pigmented neurons are lost and dopamine level is decreased by 80%. -are variable and may include weakness, paralysis, paresthesia, pain -diminished or absent reflexes starting with the lower extremities and progressing upward; -Tremor slow unilateral - worsening at rest -Rigidity - passive limb movement jerky movement -Bradykinesia - most common feature - overall slow movement - longer to complete activities -Dementia - decline of higher brain function, cognitive impairment (mimics dementia) -Dysphonia - altered voice due to incoordination of muscles responsible for speech - Basal ganglia - low dopamnine levels, patient have lose more than half of dopamine neurons in this region.

primary chemical mediators

-Histamine -Eosinophil chemotactic factor of anaphylaxis -Platelet-activating factor -Prostaglandins

immune deficienct is acquired

-Immune deficiency is acquired: -Due to medical treatment such as chemotherapy -Infection from agents such as HIV

planning and goals for patients with HIV/AIDS

-Improved nutritional status - Increased socialization and expression of grief - Increased knowledge regarding disease prevention and self-care - Absence of complications

potential complications of intracranial surgery

-Increased ICP -Bleeding and hypovolemic shock -Fluid and electrolyte disturbances -Infection -CSF leak -Seizures

Nursing interventions for malingering

-Interdisciplinary approach, communication -Redirect/teach positive behaviors -Family education/support -Confront only if strong rapport -Hidden/unlikely to admit -Detailed history to determine poss. patterns

clinical manifestations of HIV: oncologic

-Kaposi sarcoma -AIDS-related lymphomas

clinical manifestations of HIV: GI

-Loss of appetite, N/V, oral candidiasis, diarrhea -Wasting syndrome

Nursing Interventions for Factitious disorders

-Medical/surgical history uInterdisciplinary approach (communication vital) -Redirect/teach positive behaviors/responses -Family education/support -Confront only if strong rapport -Must report suspected Factitious Disorder Imposed on Another -Detailed history to determine poss. patterns

HIV/AIDS: improving nutritional status

-Monitor weight, dietary intake, serum albumin - Dietary consult as needed - Instruct ways to supplement nutrition

clinical manifestations of PIDD

-Multiple infection despite treatment -Infection with unusual/opportunistic organisms -Failure to thrive, poor growth -Positive family history

meds for somatic disorders

-SSRIs -TCAs -MAOIs -Mood stabilizers -Careful use of pain meds/anxiolytics

Tremor dominant Parkinson's

-most disabling feature is the presence of both action and resting tremors -rigidity and slow movement are relatively mild, and tremors are the primary factor interfering with daily activities

standard blood donation

-venipuncture and blood withdraw

symptoms of ischemic stroke

-weakness/numbness - confusion - difficulty speaking - visual issues -HA -trouble walking -affected side -hemiplegia (1 sided paralysis) -hemiparesis (1 sided weakness)

SSD is not usually diagnosed in DSM before the age of _________, but it can be seen in teens and children sometimes.

30

A nurse is reviewing a client's most recent platelet count and identifies the need to institute bleeding precautions. Which result would the nurse most likely have noted? 200,000 /mm3 45,000/mm3 110,000/mm3 90,000/mm3

45,000/mm3 Explanation: Bleeding precautions are recommended for clients with a platelet count of less than 50,000/mm3.

A client with a history of congestive heart failure has an order to receive 1 unit of packed red blood cells (RBCs). If the nurse hangs the blood at 12:00 pm, by what time must the infusion be completed? 3:00 pm 4:00 pm 2:00 pm 6:00 pm

4:00 pm Explanation: When packed red blood cells (PRBCs) or whole blood is transfused, the blood should be administered within a 4-hour period because warm room temperatures promote bacterial growth.

A nurse is preparing a dose of furosemide for an older adult with heart failure. The health care provider orders furosemide 1 mg/kg to be given intravenously. The client weighs 50 kg. The concentration of the drug is 40 mg/4mL (10 mg/mL). How many milliliters would the nurse administer? Record your answer using a whole number.

5 mL

A client diagnosed with schizophreniform disorder must have symptoms present for at least 1 month but with a duration of less than: 6 months. 4 weeks. 2 weeks. 12 months.

6 months Explanation: The essential features of schizophreniform disorder are identical to those of criteria A for schizophrenia, with the exception of the duration of the illness, which can be less than 6 months. Symptoms must be present for at least 1 month to be classified as a schizophreniform disorder.

A patient who just suffered a suspected ischemic stroke is brought to the ED by ambulance. On what should the nurse's primary assessment focus? A) Cardiac and respiratory status B) Seizure activity C) Pain D) Fluid and electrolyte balance

A) Cardiac and respiratory status

Following diagnostic testing, a patient has been admitted to the ICU and placed on cerebral aneurysm precautions. What nursing action should be included in patient's plan of care? A) Supervise the patient's activities of daily living closely. B) Initiate early ambulation to prevent complications of immobility. C) Provide a high-calorie, low-protein diet. D) Perform all of the patient's hygiene and feeding.

A) Supervise the patient's activities of daily living closely.

Lesions in the temporal lobe may result in which type of agnosia? Tactile Auditory Relationship Visual

Auditory Explanation: Lesions in the temporal lobe (lateral and superior portions) may result in auditory agnosia. Lesions in the occipital lobe may result in visual agnosia. Lesions in the parietal lobe may result in tactile agnosia. Lesions in the parietal lobe (posteroinferior regions) may result in relationship and body part agnosia.

A family member brings the patient to the clinic for a follow-up visit after a stroke. The family member asks the nurse what he can do to decrease his chance of having another stroke. What would be the nurse's best answer? A) "Have your heart checked regularly." B) "Stop smoking as soon as possible." C) "Get medication to bring down your sodium levels." D) "Eat a nutritious diet."

B) "Stop smoking as soon as possible."

A community health nurse is giving an educational presentation about stroke and heart disease at the local senior citizens center. What nonmodifiable risk factor for stroke should the nurse cite? A) Female gender B) Asian American race C) Advanced age D) Smoking

C) Advanced age

Cerebral Perfusion Pressure (CPP)

CPP = MAP - ICP Normal is 70-100 LESS THAN 50 IS DEVASTATING (stasis, no circulation going on)

a nurse in a clinic is assessing a client who has AIDS and a significantly decreased CD4 t cell count. the nurse should recognize that the client is at risk for developing which of the following infectious oral conditions? halitosis gingivitis xerostomia candidiasis

Candidiasis MY ANSWER Although oral candidiasis can affect anyone, it occurs most often in infants, toddlers, older adults, and clients whose immune systems have been compromised by illness, such as AIDS, or medications.

medical management of bells' palsy

Corticosteroid therapy may be used to reduce inflammation and diminish severity of the disorder

The public health nurse is planning a health promotion campaign that reflects current epidemiologic trends. The nurse should know that hemorrhagic stroke currently accounts for what percentage of total strokes in the United States? A) 43% B) 33% C) 23% D) 13%

D) 13%

A patient with a cerebral aneurysm exhibits signs and symptoms of an increase in intracranial pressure (ICP). What nursing intervention would be most appropriate for this patient? A) Range-of-motion exercises to prevent contractures B) Encouraging independence with ADLs to promote recovery C) Early initiation of physical therapy D) Absolute bed rest in a quiet, nonstimulating environment

D) Absolute bed rest in a quiet, nonstimulating environment

A patient diagnosed with a cerebral aneurysm reports a severe headache to the nurse. What action is a priority for the nurse? A) Sit with the patient for a few minutes. B) Administer an analgesic. C) Inform the nurse-manager. D) Call the physician immediately.

D) Call the physician immediately.

A nursing student is writing a care plan for a newly admitted patient who has been diagnosed with a stroke. What major nursing diagnosis should most likely be included in the patient's plan of care? A) Adult failure to thrive B) Post-trauma syndrome C) Hyperthermia D) Disturbed sensory perception

D) Disturbed sensory perception

What should be included in the patient's care plan when establishing an exercise program for a patient affected by a stroke? A) Schedule passive range of motion every other day. B) Keep activity limited, as the patient may be over stimulated. C) Have the patient perform active range-of-motion (ROM) exercises once a day. D) Exercise the affected extremities passively four or five times a day.

D) Exercise the affected extremities passively four or five times a day.

A female patient is diagnosed with a right-sided stroke. The patient is now experiencing hemianopsia. How might the nurse help the patient manage her potential sensory and perceptional difficulties? A) Keep the lighting in the patient's room low. B) Place the patient's clock on the affected side. C) Approach the patient on the side where vision is impaired. D) Place the patient's extremities where she can see them.

D) Place the patient's extremities where she can see them.

After a subarachnoid hemorrhage, the patient's laboratory results indicate a serum sodium level of less than 126 mEq/L. What is the nurse's most appropriate action? A) Administer a bolus of normal saline as ordered. B) Prepare the patient for thrombolytic therapy as ordered. C) Facilitate testing for hypothalamic dysfunction. D) Prepare to administer 3% NaCl by IV as ordered.

D) Prepare to administer 3% NaCl by IV as ordered.

When could a "rebound" anaphylactic reaction occur after an initial attack even when epinephrine has been given? A. 1 hour B. 2 hours C. 3 hours D. 4 hours

D. 4 hours Rationale: A "rebound" anaphylactic reaction can occur 4 to 8 hours after an initial attack even when epinephrine has been given.

Antiretroviral medications as postexposure prophylaxis for health care workers are started within how many hours of exposure? A. 24 hours B. 12 hours C. 36 hours D. 72 hours

D. 72 hours Rationale: Antiretroviral medications are started as soon as possible, but no more than 72 hours (3 days) after possible HIV exposure.

Which of the following is a clinical syndrome that is characterized by a progressive decline in cognitive, behavioral, and motor functions as a direct result of HIV infection? A. Cryptococcal meningitis B. Neuropathy C. Progressive multifocal leukoencephalopathy D. HIV encephalopathy

D. HIV encephalopathy Rationale: Formerly referred to as AIDS dementia complex, HIV encephalopathy is a clinical syndrome that is characterized by a progressive decline in cognitive, behavioral, and motor functions as a direct result of HIV infection.

a nurse is teaching a client who has septic shock about the development of DIC. which of the following statements should the nurse make? "DIC is controllable with lifelong heparin usage." "DIC is characterized by an elevated platelet count." "DIC is caused by abnormal coagulation involving fibrinogen." "DIC is a genetic disorder involving a vitamin K deficiency."

DIC is caused by abnormal coagulation involving fibrinogen." MY ANSWER DIC is caused by abnormal coagulation involving the formation of multiple small clots that consume clotting factors and fibrinogen faster than the body can produce them, increasing the risk for hemorrhage.

ALS patient and family education

Decisions about life support based on understanding the disease process. Encouraged to keep advanced directives to preserve their wises and autonomy

rheumatic diseases

Encompass autoimmune, degenerative, inflammatory, and systemic conditions Affect the joints, muscles, and soft tissues of the body

A client with chronic kidney disease has chronic anemia. What pharmacologic alternative to blood transfusion may be used for this client? GM-CSF Thrombopoietin Eltrombopag Erythropoietin

Erythropoietin Explanation: Erythropoietin (epoetin alfa [Epogen, Procrit]) is an effective alternative treatment for clients with chronic anemia secondary to diminished levels of erythropoietin, as in chronic renal disease. This medication stimulates erythropoiesis.

diagnostic tests for rheumatic disease

Imaging studies oX-rays oCT scan oMRI oArthrography

A nurse cares for a client with myelodysplastic syndrome who requires frequent PRBC transfusions. What blood component does the nurse recognize as being most harmful if accumulated in the tissues due to chronic blood transfusions? Iron Calcium Hemoglobin Potassium

Iron Explanation: Iron overload is a complication unique to people who have had long-term PRBC transfusion. Over time, the excess iron deposits in body tissues can cause organ damage, particularly in the liver, heart, testes, and pancreas.

Which of the following is most likely the purpose of the Glasgow Coma Scale (GCS)?

LOC assessment The GCS allows for quick assessment that can be documented in the patient record or provided to another healthcare provider to easily assess the patient's level of consciousness.

nonatopic allergic reaction

Lack genetic component Latex

A client in acute renal failure has been prescribed 2 units of packed red blood cells (PRBCs). The nurse explains to the client that the blood transfusion is most likely needed for which reason? Lack of erythropoietin Hypervolemia Preparation for likely nephrectomy Increases the effectiveness of dialysis

Lack of erythropoietin Explanation: The kidneys produce erythropoietin, a hormone that stimulates red blood cell production. A lack of this hormone is the most likely reason for blood transfusion due to the acute kidney failure. There is no indication for a nephrectomy in this question. A blood transfusion will not necessarily increase the effectiveness of dialysis. Transfusing a client with hypervolemia could lead to circulatory overload.

The nurse is preparing a patient for a bone marrow aspiration and biopsy from the site of the posterior superior iliac crest. What position will the nurse place the patient in? Lithotomy position Jackknife position Lateral position with one leg flexed Supine with head of the bed elevated 30 degrees

Lateral position with one leg flexed Explanation: Bone marrow aspiration procedure. The posterior superior iliac crest is the preferred site for bone marrow aspiration and biopsy because no vital organs or vessels are nearby. The patient is placed either in the lateral position with one leg flexed or in the prone position.

A 53-year-old man presents to the emergency department with a chief complaint of inability to form words, and numbness and weakness of the right arm and leg. Where would you locate the site of injury? Left frontoparietal region Left temporal region Left basal ganglia Right frontoparietal region

Left frontoparietal region Explanation: The patient is exhibiting signs of expressive aphasia with numbness/tingling and weakness of the right arm and leg. This indicates injury to the expressive speech center (Broca's area), which is located in the inferior portion of the frontal lobe. The motor strip is located in the posterior portion of the frontal lobe. The sensory strip is located in the anterior parietal lobe.

Which is an action of cytotoxic T cells? Lyse cells infected with virus Produce circulating antibodies Directly attack foreign antigens Decrease B-cell activity to a level at which the immune system is compatible with life

Lyse cells infected with virus Explanation: Cytotoxic T cells play a role in graft rejection. B cells are lymphocytes important in producing circulating antibodies. Suppressor T cells are lymphocytes that decrease B-cell activity to a level at which the immune system is compatible with life. Helper T cells are lymphocytes that attack antigens directly.

Myasthenic Crisis vs. Cholinergic Crisis

MC: Under Medicated -Acute muscle weakness -Can't Cough or Clear Secretion -Impaired Swallowing, Ventilation Issue -Sudden Rise in BP & Hr d/t Hypoxia -Preceded by INFECTION. TENSILON TEST MAKES IT BETTER! -Higher doses or more frequent meds given. -Intubation & Ventilation if Respiratory Muscle Failure. CC: Over Medicated -Acute muscle weakness -N/V -Diarrhea/Abd Pain -Blurred Vision -Pallor -Facial Muscle Twitching TENSIOLON TEST MAKES IT WORSE! -HOLD ALL MEDS -INTUBATION & VENTILATION -ANTIODITE: ATROPINE!!!!

symptoms of rheumatic disease

Pain Joint swelling Limited movement Stiffness Weakness Fatigue

VWD type 2

Type 2 rate by severe vWF deficiency as well as significant deficiency of factor VIII.

Ocular Myasthenia Gravis

What: 1) condition that causes muscle weakness in the body, including the eye muscles 2) causes eye turns, droop lids, double vision, especially when Pt is tired

Gullain-Barre Syndrome (GBS)

acute attack of peripheral nerve myelin most often follows a viral infection

Type 1 anaphylactic reactions examples

asthma allergic rhinitis systemic anaphylaxis reactions to insect stings

a nurse is caring for a client who has an intracranial pressure reading of 40 mm Hg. which of the following findings should the nurse identify as a late sign of increased ICP? Confusion Bradycardia Hypotension Nonreactive dilated pupils Slurred speech

bradycardia nonreactive dilated pupils Bradycardia is correct. Bradycardia is one of three findings of Cushing's triad, which is a late sign of increased intracranial pressure. A client who has hypovolemic shock is more likely to have tachycardia. Nonreactive dilated pupils is correct. Increased intracranial pressure can lead to nonreactive dilated pupils or constricted nonreactive pupils.

cellular immune response

cellular response thymus > t cells

A neurologic deficit is best defined as a deficit of the: central nervous system with absent functioning. peripheral nervous system with decreased or impaired functioning. central and peripheral nervous systems with decreased, impaired, or absent functioning. central nervous system that affects one body system.

central and peripheral nervous systems with decreased, impaired, or absent functioning. Explanation: A client with a neurologic deficit may have decreased, impaired, or absent functioning of the central and peripheral systems.

what may be the first indication of increased ICP in a patient who recently had an intracranial surgery

change in LOC or response to stimuli

hemophilia: what kinda compress

cold compress for vasoconstriction

a nurse is caring for a client who has schizophrenia and is experiencing a variety of hallucinations. which of the following hallucinations is the priority for the nurse to address? visual hallucination gustatory hallucination command hallucination tactile hallucination

command hallucination

bells' palsy recovery

complete recovery in 3-5 weeks, rarely recurs

Age-related changes in the neurologic system must be carefully assessed. Which of the following changes does the nurse expect to find in some degree depending on the patient's age and medical condition? Select all that apply. Stage IV sleep is prolonged Hyper-reactive deep tendon reflexes Reduced papillary responses Increased sensitivity to heat and cold Increased sensitivity of taste buds Decreased muscle mass

decreased muscle mass increased sensitivity to heat and cold reduced papillary responses

If a client has a lower motor neuron lesion, the nurse would expect the client to exhibit muscle spasticity. no muscle atrophy. decreased muscle tone. hyperactive reflexes.

decreased muscle tone. Explanation: A client with a lower motor neuron lesion would be expected to have decreased muscle tone. Those with upper motor neuron lesions would have hyperactive reflexes, no muscle atrophy, and muscle spasticity.

a nurse is teaching a client who has a new prescription for prednisone to treat rheumatoid arthritis. the nurse should inform the client that which of the following is a therapeutic effect of this medication? reduces risk of infection decreases inflammation improves peripheral blood flow increases bone density

decreases inflammation Prednisone is used to treat rheumatoid arthritis because it produces anti-inflammatory and immunosuppressive effects, which reduces inflammation, decreases pain, and increases mobility.

intracranial hemorrhage

delayed symptoms related to ICP changes

A trauma client in the ICU has been declared brain dead. What diagnostic test is used in making the determination of brain death? Electromyeloography (EMG) Computed tomography (CT) Electroencephalography (EEG) Magnetic resonance imaging (MRI)

electroencephalography (EEG)

acute chest syndrome

emergency!! vasoocclusive crisis due to SCD that can be associated with pneumonia; - presents similar to pulmonary embolism common signs and symptoms include chest pain, fever, cough, tachypneic, shortness of breath, hypoxia

nursing interventions to parkinsons

enhancing self care ability: ◦Encourage, teach, and support independence ◦Environmental modifications ◦Use of assistive and adaptive devices ◦Consultation with occupational therapy support of coping: ◦Set achievable, realistic goals ◦Encourage socialization, recreation, and independence ◦Planned programs of activity ◦Support groups and referral to supportive services- counselors, social worker, home care

Bands in CBC with differential

immature granulocytes

A critical care nurse is documenting the assessment of a client. The client is status postresection of a brain tumor. The nurse documents that the client is flaccid on the left. This means that the client: is not responding to stimuli. is hyperresponsive on the left. is hyporesponsive on the left. has an abnormal posture response to stimuli.

is not responding to stimuli. Explanation: Flaccidity is when the client has no motor response to stimuli. Flaccidity is a motor assessment.

type 3: immune complex reaction examples

joints and kidneys are particularly susceptible to this kind of reaction. lupus erythematosus serum sickness nephritis rheumatoid arthritis

Open brain injury treatment

keep comfortable (jobe says there is no treatment and it is always fatal)

secondary chemical mediators

leukotrienes, bradykinin, serotonin

SSD in elderly

lifelong pattern often related to stressful life changes

classic symptom of GBS

marching paralysis (paralysis going up all the way to the top and then back down)

IgE molecules attach to allergens and trigger

mast cells or basophils

Creutzfeldt-Jakob disease (CJD)

progressive, incurable, neurologic disease caused by infectious prions spread by eating infected meat

carotid endarterectomy (CEA) indications

pts with symptoms of TIA or mild stroke who are found to have severe carotid artery stenosis or moderate stenosis w other significant risk factors

how often to take vital signs during blood transfusions

q15 minutes after you start blood then q1hr until the transfusion is done at 4 hours

nursing care of ischemic stroke after stabilized

report all changes, literally all changes go to ICU super duper close monitoring

Patch testing

the gold standard performed to identify substances to which the patient has developed an allergy

Urticaria and Angioedema

two most common signs of anaphylaxis flat wheals surrounded by erythema Tx: identify cause, Epi, corticosteroids, cyclosporin (immunosuppressant), or monoclonal antibody for older adults

To evaluate a client's cerebellar function, a nurse should ask: "Have you noticed any changes in your muscle strength?" "Do you have any difficulty speaking?" "Do you have any trouble swallowing food or fluids?" "Do you have any problems with balance?"

"Do you have any problems with balance?" Explanation: To evaluate cerebellar function, the nurse should ask the client about problems with balance and coordination. The nurse asks about difficulty speaking or swallowing to assess the functions of cranial nerves IX, X, and XII. Questions about muscle strength help her evaluate the client's motor system.

When obtaining a health history from a patient with possible abnormal immune function, what question would be a priority for the nurse to ask? "Do you have abdominal pain or discomfort?" "Have you ever been treated for a sexually transmitted infection?" "When was your last menstrual period?" "Have you ever received a blood transfusion?"

"Have you ever received a blood transfusion?" Explanation: A history of blood transfusions is obtained, because previous exposure to foreign antigens through transfusion may be associated with abnormal immune function.

Which statement by the client indicates an understanding of somatic symptom disorder? "How I handle stress and emotions can affect my physical health." "Taking medication won't help my pain since it's caused by stress." "I have to avoid stress all my life to avoid getting sick again." "As soon as my symptoms go away, I'll be my old self again."

"How I handle stress and emotions can affect my physical health." Explanation: Clients who come to understand that how they cope with stress affects their physical health demonstrate an understanding of somatic symptom disorder. Clients with somatic symptom disorder eventually may be treated in mental health settings. It is unrealistic to avoid all stress in one's life.

The family members of a client with somatic symptom illness report to the nurse that every time they invite the client to join in an activity the client declines, saying things like, "I wish I could, but I feel so terrible." Which approach should the nurse suggest to encourage activity? "You are fine, the doctor said so. Let's go." "I know this is difficult, but exercise is important. It will be a short walk." "What does your pain feel like right now?" "I'll let you rest. Let me know when you feel better."

"I know this is difficult, but exercise is important. It will be a short walk." Explanation: The nurse must help the client and family learn how to establish a daily routine that includes improved health behaviors. Family members should expect resistance, including protests from the client that she or he does not feel well enough to do these things. The challenge is to validate the client's feelings while encouraging her or him to participate in activities. The nurse should not focus on the client's pain. Deferring to "doctor's orders" does not provide the client with information that could motivate him or her. The nurse should attempt to have the client participate and should address the client's reluctance.

A client is seeking relief for undiagnosed pain. There is no history of significant physical illness. The history reveals that the client was laid off 4 months ago from a job. The nurses assessment is unremarkable. Which statement made by the client would most strongly suggest a somatoform disorder? "I probably just overexerted myself working around the house. It's hard to slow down." "I'm sure they will figure out what is wrong with me." "I have been having a hard time lately. It's hard not working like I'm used to." "I seem to have more pain now that I got laid off."

"I seem to have more pain now that I got laid off." Explanation: The correlation between the client's stressors and pain suggests a somatoform disorder. The client's statements about having a "hard time," "overexerting myself" and hoping that the care team finds out what is wrong do not suggest a pathologic response to a stressor.

A patient will need a blood transfusion for the replacement of blood loss from the gastrointestinal tract. The patient states, "That stuff isn't safe!" What is the best response from the nurse? "I understand your concern. The blood is carefully screened but is not completely risk free." "You will have to decide if refusing the blood transfusion is worth the risk to your health." "I agree that you should be concerned with the safety of the blood, but it is important that you have this transfusion." "The blood is carefully screened, so there is no possibility of you contracting any illness or disease from the blood."

"I understand your concern. The blood is carefully screened but is not completely risk free." Explanation: Despite advances in donor screening and blood testing, certain diseases can still be transmitted by transfusion of blood components (Chart 32-4).

The nurse is working with a client who has schizophrenia, disorganized type. It is time for the client to get up and eat breakfast. Which statement by the nurse would be most effective in helping the client prepare for breakfast? "It's time to put your clothes on now." "Stay right there and I'll get your clothes for you." "I'll expect you in the dining room in 20 minutes." "Why don't you stay here and I'll get your tray for you."

"It's time to put your clothes on now." Explanation: Clients with schizophrenia may have significant self-care deficits. The client needs clear direction, with tasks broken into small steps, to begin to participate in the client's own self-care. The other choices do not support the client effectively. "I'll expect you in the dining room in 20 minutes," is authoritarian and does not allow the client dignity. "Stay right here, and I'll get your clothes for you," is also authoritarian and does not allow the client dignity. "Why don't you stay here and I'll get your tray for you," is kinder but it robs the client of the opportunity to do for himself or herself as much as possible.

A preoperative client is discussing blood donation with the nurse. Which statement by the client indicates to the nurse the need for further teaching? "My family will donate blood, because it's safer." "I should expect blood withdrawal to take about 15 minutes." "Donated blood is tested for blood type and infections." "I could donate my own blood in case I need a transfusion."

"My family will donate blood, because it's safer." Explanation: Directed donations from friends and family members are not any safer than those provided by random donors. Withdrawal of 450 mL of blood usually takes about 15 minutes. Specimens from donated blood are tested to detect infections and to identify the specific blood type. Autologous blood donation is useful for many elective surgeries where the potential need for transfusion is high.

The nurse is teaching a client with schizoaffective disorders (SAD) about the client's prescribed medication therapy. The nurse determines that additional education is needed when the client states what? "One day, I won't have to worry about taking any medication." "I need to change my position slowly when getting up from lying down." "I need to make sure that I drink enough fluids throughout the day." "If I notice any strange muscle movements, I should call my provider."

"One day, I won't have to worry about taking any medication." Explanation: After the client's condition has stabilized (i.e., the client exhibits a decrease in positive and negative symptoms), the treatment that led to remission of symptoms should be continued. Titrating antipsychotic agents to the lowest dose that provides suitable protection may enable optimal psychosocial functioning while slowing the recurrence of new episodes. Clients diagnosed with SAD are unlikely to be medication free. Clients also need education about preventing orthostatic hypotension, such as changing positions slowly, as well as drinking adequate amounts of fluid each day. Clients also need to notify their health care provider if they notice any abnormal muscle movement or the inability to control motor movement.

A nurse is explaining treatment options to a client diagnosed with an immune dysfunction. Which statement by the client accurately reflects the teaching about current stem cell research? "Stem cell transplantation has been carried out in humans with certain types of immune dysfunction, and clinical trials using stem cells are underway in clients with a variety of disorders with an autoimmune component." "Currently, stem cell transplantation has only been performed in the laboratory, but future research with embryonic stem cell transplants for humans with immune dysfunction has been promising." "Stem cell transplantation has been discontinued based on concerns about safety, efficacy, resource allocation, and human cloning." "Stem cell clinical trials have only been attempted in clients with acquired immune deficiencies, but plans are underway to begin human cloning using embryonic stem cells."

"Stem cell transplantation has been carried out in humans with certain types of immune dysfunction, and clinical trials using stem cells are underway in clients with a variety of disorders with an autoimmune component." Explanation: Research has shown that stem cells can restore an immune system that has been destroyed (Ko, 2012). Stem cell transplantation has been carried out in humans with certain types of immune dysfunction, such as severe combined immunodeficiency; clinical trials using stem cells are underway in clients with a variety of disorders having an autoimmune component, including systemic lupus erythematosus, rheumatoid arthritis, scleroderma, and multiple sclerosis. Research with embryonic stem cells has enabled investigators to make substantial gains in developmental biology, gene therapy, therapeutic tissue engineering, and the treatment of a variety of diseases (Ko, 2012). However, along with these remarkable opportunities, many ethical challenges arise, which are largely based on concerns about safety, efficacy, resource allocation, and human cloning.

A nurse is explaining treatment options to a client diagnosed with an immune dysfunction. Which statement by the client accurately reflects the teaching about current stem cell research? A.) "Stem cell transplantation has been discontinued based on concerns about safety, efficacy, resource allocation, and human cloning." B.) "Currently, stem cell transplantation has only been performed in the laboratory, but future research with embryonic stem cell transplants for humans with immune dysfunction has been promising." C.) "Stem cell clinical trials have only been attempted in clients with acquired immune deficiencies, but plans are underway to begin human cloning using embryonic stem cells." D.) "Stem cell transplantation has been carried out in humans with certain types of immune dysfunction, and clinical trials using stem cells are underway in clients with a variety of disorders with an autoimmune component."

"Stem cell transplantation has been carried out in humans with certain types of immune dysfunction, and clinical trials using stem cells are underway in clients with a variety of disorders with an autoimmune component." Explanation: Research has shown that stem cells can restore an immune system that has been destroyed (Ko, 2012). Stem cell transplantation has been carried out in humans with certain types of immune dysfunction, such as severe combined immunodeficiency; clinical trials using stem cells are underway in clients with a variety of disorders having an autoimmune component, including systemic lupus erythematosus, rheumatoid arthritis, scleroderma, and multiple sclerosis. Research with embryonic stem cells has enabled investigators to make substantial gains in developmental biology, gene therapy, therapeutic tissue engineering, and the treatment of a variety of diseases (Ko, 2012). However, along with these remarkable opportunities, many ethical challenges arise, which are largely based on concerns about safety, efficacy, resource allocation, and human cloning.

A nurse and nursing student are caring for a client recovering from a lumbar puncture yesterday. The client reports a headache despite being on bedrest overnight. The physician plans an epidural blood patch this morning. The student asks how this will help the headache. The correct reply from the nurse is which of the following? "The blood will seal the hole in the dura and prevent further loss of cerebral spinal fluid." "The blood provides moisture at the site, which encourages healing." "The blood can repair damage to the spinal cord that occurred with the procedure." "The blood will replace the cerebral spinal fluid that has leaked out."

"The blood will seal the hole in the dura and prevent further loss of cerebral spinal fluid." Explanation: Loss of CSF causes the headache. Occasionally, if the headache persists, the epidural blood patch technique may be used. Blood is withdrawn from the antecubital vein and injected into the site of the previous puncture. The rationale is that the blood will act as a plug to seal the hole in the dura and prevent further loss of CSF. The blood is not put into the subarachnoid space. The needle is inserted below the level of the spinal cord, which prevents damage to the cord. It is not a lack of moisture that prevents healing; it is more related to the size of the needle used for the puncture.

A nurse cares for a client with megaloblastic anemia who had a total gastrectomy three years ago. What statement will the nurse include in the client's teaching regarding the condition? "The condition is likely caused by a vitamin B12 deficiency." "The condition is likely caused by a folate deficiency." "The condition causes abnormally rigid red blood cells." "The condition causes abnormally small red blood cells."

"The condition is likely caused by a vitamin B12 deficiency." Explanation: Vitamin B12 combines with intrinsic factor produced in the stomach. The vitamin B12 -intrinsic factor complex is absorbed in the distal ileum. Clients who have had a partial or total gastrectomy may have limited amounts of intrinsic factor, and the absorption of vitamin B12 may be diminished. Megaloblastic anemia may be caused by a folate deficiency; however, the client's history of gastrectomy indicates the likely cause is a vitamin B12 deficiency. Megaloblastic anemia causes large erythrocytes (RBCs), not small or rigid.

a nurse is reviewing the cause of gout with a group of nurses. which of the following statements should the nurse make? uric acid levels drop and calcium forms precipitate tophi form in the kidneys and they impair the excretion of uric acid the intraarticular deposition of urate crystals causes inflammation articular cartilage thins leading to splitting and fragmentation

"The intra-articular deposition of urate crystals causes inflammation." MY ANSWER Gout, or gouty arthritis, develops when urate crystals deposit in joints and tissues and cause inflammation and pain.

A client with Guillain-Barré syndrome has paralysis affecting the respiratory muscles and requires mechanical ventilation. When the client asks the nurse about the paralysis, how should the nurse respond? "You'll first regain use of your legs and then your arms." "The paralysis caused by this disease is temporary." "You'll be permanently paralyzed; however, you won't have any sensory loss." "It must be hard to accept the permanency of your paralysis."

"The paralysis caused by this disease is temporary." Explanation: The nurse should inform the client that the paralysis that accompanies Guillain-Barré syndrome is only temporary. Return of motor function begins proximally and extends distally in the legs.

The nurse working on a mental health unit is teaching a nursing student. The student asks the nurse about what constitutes a diagnosis for major depressive disorder. What is the nurse's best response? "The physician diagnoses depression when a client has feelings of sadness several times a year." "Feelings of anxiety and sadness as a response to a life event are the most important qualifiers for depression." "The primary diagnostic criterion is one or more major depressive episodes for at least 2 weeks with other symptoms present." "Depression is a mood variation to life events."

"The primary diagnostic criterion is one or more major depressive episodes for at least 2 weeks with other symptoms present." Explanation: Normal variations in mood (such as sadness, euphoria, and anxiety) occur in response to life events; they are time limited and not usually associated with significant functional impairment. The primary diagnostic criterion for major depressive disorder is one or more major depressive episodes (either a depressed mood or a loss of interest of pleasure in nearly all activities) for at least 2 weeks. Four of seven other symptoms must be present. Thus, the best response from the nurse is "the primary diagnostic criterion is one or more major depressive episodes for at least 2 weeks with other symptoms present."

The parents of a client intubated due to the progression of Guillain-Barré syndrome ask whether their child will die. What is the best response by the nurse? "There are no guarantees, but a large portion of people with Guillain-Barré syndrome survive." "Don't worry; your child will be fine." "It's too early to give a prognosis." "Once Guillain-Barré syndrome progresses to the diaphragm, survival decreases significantly."

"There are no guarantees, but a large portion of people with Guillain-Barré syndrome survive." Explanation: The survival rate of Guillain-Barré syndrome is approximately 90%. The client may make a full recovery or suffer from some residual deficits. Telling the parents not to worry dismisses their feelings and does not address their concerns. Progression of Guillain-Barré syndrome to the diaphragm does not significantly decrease the survival rate, but it does increase the chance of residual deficits. The family should be given information about Guillain-Barré syndrome and the generally favorable prognosis. With no prognosis offered, the parents are not having their concerns addressed.

The spouse of a client with hypochondriasis has accompanied the client to the follow-up doctor's visit. While waiting for the doctor, the spouse expresses to the nurse frustration with the client's obsession about illness. The spouse asks the nurse, "What can I do?" What would be the best response by the nurse? "Try to be the client and understand that the client is worried that the client is sick." "Try to give the client some sort of reward when the client resists complaining about the client's illnesses." "Try ignoring the client's complaints, and they should subside." "Try finding an activity you enjoy doing together to help the client feel better overall."

"Try finding an activity you enjoy doing together to help the client feel better overall." Explanation: Building a trusting relationship with the client, providing empathy and support, and being sensitive to rather than dismissive of complaints are skills that the nurse can use in any setting where clients are seeking assistance. Encouraging clients to find pleasurable activities or hobbies may help to meet their needs for attention and security. Ignoring the complaints is likely to exacerbate the situation. Providing rewards for participating does not address the root causes of this phenomenon. Telling the husband to just "understand" his wife's worries is simplistic because this does not give him any tools to address this challenging situation.

A client with schizophrenia is reluctant to take prescribed oral medication. Which is the most therapeutic response by the nurse to this refusal? "What is it about the medicine that you don't like?" "If you refuse these pills, you'll have to get an injection." "You know you have to take this medicine for your own good." "I can see that you're uncomfortable now, so we can wait until tomorrow."

"What is it about the medicine that you don't like?" Explanation: Asking the client why the client does not like the medication explores the client's reason for refusal, which is the first step in resolving the issue. The nurse must determine the barriers to compliance for each client. Threatening the client with an injection is assault. Waiting until tomorrow puts off the inevitable. Telling the client it is for the client's own good is not the most therapeutic response in order to get the client to take medication.

A client who is depressed begins to cry and states, "I'm just really sick of feeling this way. Nothing ever seems to go right in my life." Which would be the most appropriate response by the nurse? "Nothing ever goes right?" "Don't cry. Try to look at the positive side of things." "Hang in there. Your medication will start helping in a few days." "You are feeling really sad right now. It's a hard time."

"You are feeling really sad right now. It's a hard time." Explanation: Do not cut off interactions with cheerful remarks or platitudes. Do not belittle the client's feelings. Accept the client's verbalizations of feelings as real, and give support for expressions of emotions, especially those that may be difficult for the client (like anger). Allow (and encourage) the client to cry. It is important that the nurse does not attempt to "fix" the client's difficulties.

A nurse is caring for a client who will undergo total knee replacement and will have an autologous transfusion. Which statement will the nurse include when teaching the client about the transfusion? "You typically donate blood 4 to 6 weeks before the surgery." "You will likely not need the blood that is donated." "You typically donate blood the day of the surgery." "You will be prescribed calcium to replace what is lost during donation."

"You typically donate blood 4 to 6 weeks before the surgery." Explanation: With autologous donation, a client's own blood may be collected for future transfusion; this is an effective method for orthopedic surgery, where the likelihood of transfusion is high. Preoperative donation is ideally collected 4-6 weeks before surgery. The nurse will not tell the client that the blood will not be needed; orthopedic surgeries often require transfusion of blood. The client will be prescribed iron supplements during the donation time, not calcium.

manifestations of meningitis

**nuchal rigidity** stiffness and pain in the neck - r/t meningeal involvement headache fever changes in LOC behavioral changes positive kernig's sign positive brudzinski's sign photophobia

Epidural hemorrhage / hematoma

- EXTREME EMERGENCY - characterized by brief LOC, then lucid interval in which patient is conversant and awake., then increased restlessness / agitation / confusion then coma - after this, then signs of herniation occur (deterioration of consciousness and signs of focal neuro deficits like dilation and fixation of pupils or extremity paralysis) - between the skull and the dura - neuro damage / respiratory arrest quick onset - high risk for herniation (fast developing, fast growing bleed, usually an artery)

autonomic symptoms of PD

- Excessive sweating - Paroxysmal flushing - Orthostatic hypotension - Urinary retention/constipation -Dementia/sleep disturbance - Hallucinations

Glucose-6-phosphate dehydrogenase deficiency

- G-6-PD gene produces an enzyme within the RBC that is needed for membrane stability -caused by G-6-PD abnormality - hemolysis can occur when the cells are stressed (like during fevers or certain medications) - people normally will live their life with normal lab values until the stressor (fever or drug introduction) happens

clinical manifestations of HIV: gynecologic

- Genital ulcers - Persistent, recurrent vaginal candidiasis - Pelvic inflammatory disease - Menstrual abnormalities

side effects of tPA

- Hemorrhage - Hypotension - Fever - recurrent ischemia/thromboembolism - monitor IV site, foley, stool, ETT, NGT, emesis for bleeding.

clinical manifestations of HIV: integumentary

- Herpes zoster - Seborrheic dermatitis

therapy for pts with ischemic stroke not receiving tPA

- IV anticoagulant administration (IV heparin or LMWH) - osmotic diuretics (mannitol) - supplemental oxygen - elevate HOB 30 degrees - hemicraniectomy - intubation - continuous hemodynamic monitoring - frequent neuro assessments - monitoring for development of fever (this has increased mortality within first 24 hrs) - monitoring of blood glucose and use of sliding scale insulin to keep levels in range of 140-180

assessment of patient with HIV/AIDS

- Identification of potential risk factors - Physical status - Psychological status - Immune system functioning - Nutritional status - Respiratory status - Neurologic status - Fluid and electrolyte balance - Knowledge level

HIV/AIDS: coping with grief

- Identify resources for support and mechanisms for coping - Consult mental health counselors as needed

HIV/AIDS: improving knowledge of HIV

- Instruct patient and family about routes of transmission and prevention - Avoid sexual contact with multiple partners - Use condoms - Do not use IV/injection drugs

nursing roles for neuro patients

- MAINTAIN AIRWAY - protect patient - fluid/nutrition balance - mouth care - skin/joint integrity - body temperature - prevent urinary retention - bowel function - health maintenance - family counseling

acute hemolytic reaction

- MOST DANGEROUS - INCOMPATIBLE BLOOD - potentially life threatening. Antibodies already present in recipients plasma rapidly combine with antigens on donor RBC and the RBC are destroyed in circulation. Rapid hemolysis occur with ABO incompatibility - can occur after as little as 10 mls of blood. Can be caused by errors in labeling. SX: fever, chills, low back pain, nausea, chest tightness, dyspnea, anxiety. Sense of impending doom. Hypotension, bronchospasm, vascular collapse. Diminished renal perfusion - acute renal failure and possible DIC. STOP TRANSFUSION!!

HIV/AIDS: monitoring and managing potential complications

- Monitor for side effects of medications - Monitor for HAND - Encourage verbalization of body image changes

nursing assessment SSD

- ROS - meds / PMHx - 1-10 pain scale - functional status - substance / ETOH abuse - polypharmacy / polyproviders - Mental Status Exam - H/O adverse events - relationship status - primary / secondary gains (attention, escapism) - risk of self-harm / suicide - cultural factors / community

teaching for MG

- Stress the importance of lifestyle adaptations such as avoiding heat (e.g., sauna, hot tubs, sunbathing), crowds, overeating, erratic changes in sleep habits, or emotional extremes. - Teach the signs of exacerbation, such as increased weakness, increased diplopia, ptosis, and problems with chewing or swallowing. - Remind the patient to plan activities to allow for rest periods and to conserve energy. - Provide the drug regimen in a written format that includes the names, purposes, dosages, scheduled dosage times, and side effects of the drugs. - Explain that the drugs are normally taken before activities such as eating, participating in sports, or working. - Stress the importance of maintaining therapeutic blood levels by taking the medications on time and as prescribed and not missing or postponing doses. - encourage family members to learn resuscitation procedures - Encourage family members or significant others to discuss their feelings with one another. - Teach the patient the importance of obtaining and wearing a medical alert (MedicAlert) bracelet or necklace and to carry identification at all times.

major complications of carotid endarterectomy (CEA)

- Stroke, cranial nerve injuries, infection or hematoma at the incision, and carotid artery disruption - Maintain bp levels post op - Neurological observation record used to monitor all body systems - if any new sudden onset of neuro deficits, primary doctor notified immediately and prepared for repeat endarterectomy - Cranial nerve injury is most common complication - Emergency airway supplies are essential to have

Babinski sign

- This is used to test for brain damage or upper motor neuron damage. It is considered positive if the toes flare up upon stroking the plantar aspect of the foot.

suspected things intracranial surgery indications

- a clot must be suspected in any patient who does not awaken as expected or whose condition deteriorates - an intracranial hematoma must be suspected if the pt has any new postop neuro deficits like a dilated pupil on the operative side in which case the pt is returned to the OR - Cerebral edema, infarction, metabolic disturbances, and hydrocephalus are conditions that may mimic the clinical manifestations of a clot

Later indications of increasing ICP include:

- abnormal respiratory responses - abnormal vasomotor response - cushing's triad (bradycardia, widened pulse pressure, irregular pulse pressure) - vomiting

Nursing interventions for Functional Neurologic and Illness Anxiety Disorder

- acknowledge symptoms - rapport / trust / therapeutic communication - education on problem solving - redirection - refer for therapy - family education / support

diagnostic findings of gout

- acute gouty arthritis - tophi - gouty nephropathy - uric acid renal calculi acute arthritis is the most common early clinical manifestation and the MTP joint of big toe is the most common spot and attacks tend to subside without treatment over 3 to 10 days

what groups are most likely to have Glucose-6-Phosphate dehydrogenase deficiency?

- african americans - greeks - italians

prevention of cerebrovascular disorders: nonmodificable risk factors

- age (over 55) - male gender - African American race

immune system influences

- age and gender - nutrition - history of infection or immunization - allergies - presence of autoimmune disorders, cancer, chronic illness, surgery, or trauma - medications -transfusion - lifestyle - psychoneurologic factors (stress)

what causes someone to be more likely to have intracranial bleeding after being given tPA?

- age over 70 - baseline NIHSS score greater than 20 - serum glucose concentration of 300 or higher - edema or mass effect (local pressure or bleeding from a tumor on adjacent parts of brain) observed on the pt's initial CT scan

postop management for intracranial surgery goals

- aimed at detecting and reducing cerebral edema - relieving pain - preventing seizures - monitoring ICP - monitoring neuro status

HIV/AIDS postexposure actions

- antiretroviral medications within 72 hours of exposure - 2 to 3 drugs prescribed for 28 days

head trauma management

- assess GCS - dx with CT or MRI - supportive measures - benzos for sedation do not affect vascular flow restrictions

nursing care of seizures: after

- assess speech - consciousness - sleep - place side lying

head trauma management

- assume cervical spine injury until this is ruled out - therapy to preserve brain homeostasis and prevent secondary damage - treat

carotid atherosclerosis nursing assessment

- auscultate carotids for bruit

preop nursing management of intracranial surgery

- baseline LOC, responsiveness, neuro deficits, motor strength - gauge the pt and their family's understanding of the surgery and purpose - pt should plan to shower and wash hair - surgical site is shaved and prepped - IV abx given 1 hr before surgery - foley placed in OR - can sometimes have art line placed

big thing to know with IVIG

- big anaphylactic shock risk - START IT SLOW AND THEN WORK IT UP - there is no time limit to give it

assessment for patients with hematologic disorders:

- bruising and bleeding - fatigue

causes of seizures

- cerebrovascular disease - hypoxemia - fever (in kids) febrile seizures usually have to do with how FAST the temperature came on, NOT how high the seizure is - head injury - hypertension - CNS infections - metabolic and toxic conditions - brain tumor - drug and alcohol withdrawal - allergies

Acute SDH s/s

- changes in LOC - pupillary changes - hemiparesis - sometimes there are no symptoms - coma, hypertension, bradycardia, bradypnea are all signs of rapidly expanding bleed requiring immediate intervention

nursing care of seizures: during

- circumstances prior - first action, last action - type of movement / area - pupil size and action / response - automatisims - incontinence - duration - consciousness - paralysis / weakness - turn on side - nothing in mouth - pad head - stay with patient - time it

Brain Death (Total Brain Death) (must have at least 3 of these reflexes be abnormal)

- coma - loss of brain stem reflexes - no pupil response - no corneal response - no gag reflex - doll's eye (eyes stay fixed when you turn their head) - caloric test / ice water test (ice water in ear normally causes nystagmus) - apnea

4 types of brain injury

- concussion - contusion - diffuse axonal injury - intracranial hemorrhage

stroke protocol: post tPA

- continuous cardiac monitoring - frequent neuro assessments - frequent vitals (q15 mins for first 2 hours, then q30 mins for next 6 hours, then every hour until 24 hours after treatment - maintain systolic BP at less than 185 - maintain diastolic BP at less than 110 - treat fever - airway management is based on pt's clinical condition and ABGs

medical management of hemorrhagic stroke

- control htn - care is supportive - sedation - oxygen - treat increased ICP, prevent further damage

improve mobility

- daily exercise program - ROM exercises - postural exercises - learn to walk with walker - frequent rest periods - proper shoes - use of assistive devices

complications from ischemic stroke

- decreased cerebral blood flow - inadequate oxygen delivery to brain - pneumonia - immobility issues

complications of brain bleed

- decreased perfusion - edema/herniation - impaired O2 / ventilation - seizure risk - impaired fluid / nutrition electrolyte balance

skull fractures

- defined by location / type - we don't usually treat a skull fracture unless it is depressed or symptomatic

psychological symptoms of PD

- depression - anxiety - dementia - delusions - hallucinations - psychosis

assessment / diagnosis of headache

- description: -- age of onset -- frequency, location, duration -- type of pain -- relief - history: -- include medication -- detailed history and physical assessment -- persistent headache require further assessment - diagnostic testing to evaluate underlying cause

Potential complications of the Patient with HIV/AIDS

- development of HIV-Associated Neurocognitive Disorder - body image changes - adverse effects of medications

types of blood donation

- directed - standard - autologous - intraoperative blood supply - hemodilution

Signs and symptoms of early ICP changes

- disorientation - increase RR - confusion - pupillary changes - one sided weakness - constant headache - agitation - slowed speech - delayed in response to verbal commands - constant HA

HIV: Prevention

- education on how to eliminate or reduce risks associated with HIV infections and AIDS prevention education - behavioral interventions - HIV testing - linkage to treatment and care

nursing intervention SSD

- encourage independence - identifying strengths - dont dwell on symptoms - possible PT or OT referall - groups - schedule follow up to avoid unnecessary urgent care visits

ALS manifestations

- fatigue - progressive muscle weakness - cramps - fasciculation (twitching) - incoordination

transfusion complications

- febrile non-hemolytic reaction - acute hemolytic reaction - allergic reaction - circulatory overload - bacterial contamination - transfusion related acute lung injury - delayed hemolytic reaction - disease acquisition

assessment of parkinson's disease

- focus on the degree of disability and function of the patient including ALDs, IADLs, and cognitive function - medications -coping - safety concerns is the biggest thing

anemia in renal disease

- generally not anemic until serum creatinine level exceeds 3/mg/100 ml. If untreated Hematocrit falls between 20 to 30 %. Caused by both a mild shortening of RBC lifespan and deficiency of RBC.

primary immune deficiency disorders (PIDD) pathophysiology

- genetic - majority of the time is diagnosed in infancy; some may be diagnosed during adolescence - Occasionally, adults may present with persistent, recurrent, or resistant infections -Prevent body from developing normal immune responses -May affect phagocytic function, B cells or T cells, or the complement system

pre-transfusion assessment

- have to have consent - vital signs - cardiac assessment - respiratory assessment - skin assessment

considerations for patients postop after intracranial surgery

- head dressing may impair hearing temporarily - vision may be impaired if eyes are swollen shut - if trach or ETT is still in place, pt will not be able to speak until it is removed so make an alternative way to communicate

Prevention of ischemic stroke

- health maintenance measures -- work on modifiable risk factors - carotid endarterectomy (strip out the plaque) - medications: -- anticoagulants -- antiplatelet (statins) -- antihypertensives

allergic reaction to blood

- hives - immune response rxn - sensitivity to plasma protein. Mild and respond to antihistamine. Rarely is severe

prevention of cerebrovascular disorders: modifiable risk factors

- hypertension is the primary risk factor - other CV disease - elevated cholesterol or hematocrit - obesity - diabetes - oral contraceptive use - smoking, drug, alcohol abuse

Acute SDH Treatment

- immediate craniotomy to evacuate the subdural clot - control of ICP - monitoring of respiratory function

nursing diagnosis for parkinsons

- impaired physical mobility and risk for activity intolerance - disturbed thought processes - self-care deficits - imbalanced nutrition - constipation - impaired verbal communication - ineffective coping and compromised family coping - deficient knowledge - risk for injury

nurse planning for parkinsons disease

- improved functional ability - maintaining independence in activities of daily living - achieving adequate bowel elimination - attaining and maintaining acceptable nutritional status - achieving effective communication - developing positive individual and family coping skills

hemodilution

- initiated before or after induction of anesthesia. 1 -2 units of blood are removed through venous/arterial line and simultaneously replaved with colloid or cystalloid solution. Blood obtained is reinfused after surgery.

Hypoproliferative anemias

- iron-deficiency anemia - anemia in renal disease - anemia of chronic disease - megaloblastic anemia

Transfusion Related Acute Lung Injury (TRALI)

- leading cause of death in relation to blood transfusions - sx: pulmonary edema, frothy pink sputum, sob, hypoxia, fever, hypotension - happens within 6 hours of transfusion - typically with plasma or platelets

care for carotid endarterectomy

- maintain adequate blood pressure - cardiac monitoring - neuro observation record and notify MD if any neuro changes occur - monitor for thrombus at site of endarterectomy (this can be the cause of sudden neuro deficits) - monitor for difficulty swallowing, hoarseness, or cranial nerve dysfunction - monitor for excessive edema in neck and hematoma formation as this can cause airway obstruction so keep emergency airway supplies in room - assess for bilateral pulses distal to cath site - monitor for neck pain and wound expansion and report feelings of pressure in neck and difficulty breathing - monitor for severe unilateral headache improved by sitting upright or standing. - monitor for changes in LOC or confusion and report immediately

basic care of patients intracranial surgery

- maintain head of bed elevated at 30 degrees with neck neutral to promote venous drainage - maintain nasal packing in place and reinforce PRN - tell Pt not to blow nose - provide oral care

Medical management of increased ICP

- monitoring: Intraventricular cath, subarachnoid bold, epi./subdural cath (keep transducer level at the TALLUS OF THE EAR) - drainage (ventriculostomy, intraventricular cath) STERILE. LABEL LINE BC IT LOOKS SIMILAR TO IV TUBING. **high risk for developing meningitis** - decrease edema; osmotic diuretics (mannitol, 3% NS-- these dehydrate the cells and pull fluid off of the cells), fluid restriction, cerebral cooling - maintain cerebral perfusion: manipulation of cardiac output (intentionally keep these patients HYPERTENSIVE BC THIS KEEPS MAP UP), decompressive hemicraniectomy or Bone Flap to allow brain to swell - decrease flow and CSF - control fever - maintain oxygenation - reduce metabolic demands - decrease increasing ICP

iron deficiency anemia

- most common type of anemia - intake of dietary iron is not enough - commonly caused by blood loss - CHECK FOR BLEEDING - smooth tongue - take iron with orange juice - usually the bleeding comes from gastritis, IBD, or GI tumors.

post tPA care

- neuro assessments q15 minutes for the first 6 hours after tPA administration - close BP assessment - bleeding precautions - supportive care

SSD education

- nutrition - lifestyle - hygiene - meds / surveillance - pain management - relaxation techniques and assertiveness

ischemic stroke

- obstruction of blood flow - causes: -- artery thrombosis (large or small penetrating) -- cardiogenic embolism (microclots during DIC or a-fib) -- cryptogenic (fatty embolism or amniotic embolism) -- other (coronary artery plaques, atherschlerotic causes)

intracranial surgery

- open craniotomy - transsphenoidal craniotomy - Burr holes (temporary pressure valves) - seizure risk (give anticonvulsant before / after) - goals are: focus on reducing cerebral edema, pain seizures, ICP monitoring - complications: Diabetes insipidus, SIADH, infection, leakage, visual changes

nursing goals for ICP management

- optimize cerebral tissue perfusion - negative fluid balance - prevent infection - monitor / manage complications - monitor ICP change

autologous transfusion

- own blood for future transfusion. Elective surgeries. I unit each week (4-6 weeks prior), prevention of viral infections. Useful with people with hx of transfusion reactions Expensive takes time.

glucose-6-phosphate dehydrogenase deficiency: medications to NOT give these people

- oxidative drugs have a hemolytic effect on them - ex: antimalarials, sulfonamides (Bactrim), Dapson, Primaquine, Fava Beans

diagnostic findings of fibromyalgia

- pain upon pinching Achilles tendon using 4kg of pressure for 4 seconds - an affirmative answer to statement "i have a persistent deep aching all over my body"

management of meningitis

- prevent w vaccine - early doses of appropriate IV antibiotics for bacterial meningitis - dexamethasone - treat dehydration, shock, seizures - frequent assessment of VS and LOC - protect patient from injury related to seizures or altered LOC - monitor daily weight, serum electrolytes, urine volume, specific gravity, and osmolality - prevent complications r/t immobility - infection control precautions - supportive care - measures to facilitate coping of patient and family

complications to LOC changes

- respiratory failure (protecting airway is a priority when patient has a massive LOC change) - pneumonia - aspiration - pressure ulcer - venous thromboembolism - contractures

rheumatoid arthritis diagnostic findings

- rheumatoid nodules - joint inflammation on palpation - stiffness, tenderness, swelling, and temp changes in joints - weight loss - sensory changes - lymph node enlargement - fatigue

treatments for hematologic disorders

- splenectomy - therapeutic apheresis (filter out certain types of cells) - transplant - phlebotomy (can go thru and remove specific cells if there are too much) - PRBC - IVIG (INCREDIBLY HIGH RISK OF ANAPHYLAXIS) - Factors (has to be given IV)

vascular neurological issues

- when blood supply is disrupted - stroke: -- primary cerebrovascular disorder -- also the 3rd leading cause of death in the US - stroke is THE LEADING CAUSE OF LONG-TERM DISABILITY IN THE USA

Delayed hemolytic transfusion reaction

- within 14 days! sx: Fever, anemia, increased bilirubin, decreased or absence haptoglobin, and jaundice (possible). - Occurs when level of antibody has been increased to the extent that a reaction can occur. Limit number of transfusions.

parkinsons disease medications

-Amantadine = mild symptoms -Anticholinergics (Benztropine) = for tremor and rigidity -Levodopa/Carbidopa -Dopamine agonists (Pramipexole, Ropinirole, Bromocriptine) -MAO-I (Rasagiline, Selegiline) -COMT-I (Tolcapone, Entacapone) -Antipsychotic (Clozapine)

preop management of intracranial surgery

-CT & MRI for baseline (where & why surgery) -cerebral angiogram (check for altered blood flow) -transcranial duplex (check for altered blood flow) -antiseizure meds (dilantin, cerebryx) because of high risk for sz if manipulating brain tissue -corticosteroids (dexamethasone) to decrease swelling assoc w/ surgery -fluid restriction (mannitol, furosemide) -abx -anxiolytic (diazepam)

strategies to protect against HIV infection

-Consistent and correct use of condoms -Medical male circumcision -Female condom -Harm reduction framework for people who inject drugs -Needle exchange -May use bleach to clean used needles and syringes -Avoid sharing needles and syringes - reproduction education - benefits of antiretroviral treatment - artificial insemination in some cases - refrain from breastfeeding - remain nonjudgmental in educating about prevention

SSD DSM 5 Criteria

-One or more s/s with persistent distress > 6 months -Excessive thoughts about seriousness -May have pseudoneurologic symptoms -Mild, moderate, severe classifications -Behaviors r/t stress regarding symptoms -Excessive spending -Disproportionate time focusing on illness -Decreased functional ability -May be further specified as pain related

factitious disorder imposed on another

-Previously known as Munchausen's Syndrome by Proxy -Usually, mother causing illness in child -Receives public/family sympathy -Craves attention

DIC manifestations

-bleeding from openings in skin, bleeding from umbilicus/trachea, bleeding from mucus membranes, hypotension -organ dysfunction from clots in organs causing ischemia/infarction

How to test for CSF leak

1) Position patient at semi fowler position 2) Tell patient to flex neck 3) Collect sample using 4x4 gauze 4) Assess gauze for halo ring OR CSF will be POSITIVE FOR GLUCOSE SO TEST WITH A GLUCOSE STICK

Types of MS

1. Relapsing-remitting 2. Primary-progressive 3. Secondary-progressive 4. Progressive-relapsing

A client who was receiving a monoamine oxidase inhibitor (MAOI) is to be switched to a selective serotonin reuptake inhibitor (SSRI). The nurse would expect to begin administering the SSRI how many days after the MAOI is discontinued? 7 days 21 days 28 days 14 days

14 days Explanation: To prevent possible interactions, 14 days should elapse between the discontinuation of the MAOI and the start of the SSRI.

SSD is not genetic, but can be a learned behavior if the patient has a ________________.

1st degree relative with SSD.

The mental health nurse appropriately provides education on light therapy to which client? 20-year-old college student who reports being "too tired, sad, and unfocused" to enroll for classes in the winter term 45-year-old lawyer whose medication therapy needs an additional treatment 50-year-old farmer whose major depression has not responded to any treatment modality 58-year-old showing signs of early Alzheimer's disease

20-year-old college student who reports being "too tired, sad, and unfocused" to enroll for classes in the winter term Explanation: Phototherapy—or the exposure to bright artificial light—can markedly reverse the symptoms of seasonal affective disorder, which occurs in the fall and winter. Phototherapy would be most appropriate for a 20-year-old college student who reports being "too tired, sad, and unfocused" to enroll for classes in the winter term.

platelets lifespan

7-10 days

which of the following score ranges is best associated with patient coma using the Glasgow Coma Scale?

8 or Less = coma Any patient who has a score of less than 8 is considered comatose, if they do not have communication impairment like being deaf or blind or debilitating muscular disease. Remember to assess your patients fully

Which client best exhibits the characteristics that are typical of the prodromal period of schizophrenia? A 25-year-old woman who is free of any of the symptoms of schizophrenia. A 30-year-old man who has experienced an exacerbation of symptoms after deciding not to take his atypical antipsychotic A 28-year-old woman who exhibits bizarre and disruptive behavior A 20-year-old man who is exhibiting a gradual decrease in his ability to concentrate and function in daily activities

A 20-year-old man who is exhibiting a gradual decrease in his ability to concentrate and function in daily activities Explanation: A prodromal period for individuals with schizophrenia has usually been identified in retrospect and is evidenced by some change in overall function (difficulties in school or work, within relationships, or daily activities) accompanied by transient or weak symptoms of psychosis. Absence of symptoms is not associated with the prodromal period. An exacerbation of symptoms after stopping medication suggests a relapse. Bizarre and disruptive behavior suggests acute illness.

a nurse is caring for a client who has a mild traumatic brain injury (TBI). which of the following manifestations should the nurse immediately report to the provider? a change in GCS from 13 to 11 diplopia a drop in heart rate from 76 to 70/min ataxia

A change in the Glasgow Coma Scale score from 13 to 11 In a client who has mild TBI, a decrease of 2 points on the Glasgow Coma Scale indicates a decrease in level of consciousness and that the client is risk of a deteriorating neurologic status. Therefore, this finding is the priority to report to the provider.

a charge nurse is planning to admit several clients to the medical unit. which of the following clients should the nurse assign to a private room? a client who has a fever of unknown origin a client who had a total hip arthroplasty a client who is HIV positive a client who is neutropenic

A client who is neutropenic MY ANSWER Clients who have neutropenia (a low count of neutrophils, a type of WBC that helps fight infection) due to immune system compromise, such as clients who have leukemia or major burns or are receiving chemotherapy or allogenic hematopoietic stem cell transplants, require a protective environment to prevent the spread of pathogens to the clients requiring the protective environment. This means a private room with positive airflow.

a group of nurses are discussing risk factors for transmission of human immunodeficiency virus from clients. which of the following individuals should the nurse identify as being at the greatest risk for contracting HIV? an occupational therapist who works with a client who has HIV a personal trainer who works with a client who has HIV a phlebotomist who collects blood from clients who have HIV a nurse who works for an insurance company and collects urine samples from clients who have HIV

A phlebotomist who collects blood from clients who have HIV The greatest risk for exposure to HIV is from a needle stick; therefore, the phlebotomist who collects blood is at greatest risk.

A client was abandoned by the parents at age 3, resulting in the client's perception of the world as a hostile place and the subsequent development of rage against men. This statement is an example of what? A biological explanation for the client's depressive disorder. A psychodynamic interpretation of the client's major depressive disorder. A reason the client has become lesbian at the age of 23. A feminist viewpoint of depression.

A psychodynamic interpretation of the client's major depressive disorder. Explanation: Psychodynamic theories postulate that clients with depression have unexpressed and unconscious anger about feeling helpless or dependent on others. Such anger begins in childhood when basic developmental needs are not met. Clients cannot express this anger toward the person or people on whom they feel dependent, so their anger turns inward.

A patient is brought by ambulance to the ED after suffering what the family thinks is a stroke. The nurse caring for this patient is aware that an absolute contraindication for thrombolytic therapy is what? A) Evidence of hemorrhagic stroke B) Blood pressure of 180/110 mm Hg C) Evidence of stroke evolution D) Previous thrombolytic therapy within the past 12 months

A) Evidence of hemorrhagic stroke

The nurse is assessing a patient with a suspected stroke. What assessment finding is most suggestive of a stroke? A) Facial droop B) Dysrhythmias C) Periorbital edema D) Projectile vomiting

A) Facial droop

When preparing to discharge a patient home, the nurse has met with the family and warned them that the patient may exhibit unexpected emotional responses. The nurse should teach the family that these responses are typically a result of what cause? A) Frustration around changes in function and communication B) Unmet physiologic needs C) Changes in brain activity during sleep and wakefulness D) Temporary changes in metabolism

A) Frustration around changes in function and communication

During a patient's recovery from stroke, the nurse should be aware of predictors of stroke outcome in order to help patients and families set realistic goals. What are the predictors of stroke outcome? Select all that apply. A) National Institutes of Health Stroke Scale (NIHSS) score B) Race C) LOC at time of admission D) Gender E) Age

A) National Institutes of Health Stroke Scale (NIHSS) score C) LOC at time of admission E) Age

After a major ischemic stroke, a possible complication is cerebral edema. Nursing care during the immediate recovery period from an ischemic stroke should include which of the following? A) Positioning to avoid hypoxia B) Maximizing PaCO2 C) Administering hypertonic IV solution D) Initiating early mobilization

A) Positioning to avoid hypoxia

The patient has been diagnosed with aphasia after suffering a stroke. What can the nurse do to best make the patient's atmosphere more conducive to communication? A) Provide a board of commonly used needs and phrases. B) Have the patient speak to loved ones on the phone daily. C) Help the patient complete his or her sentences. D) Speak in a loud and deliberate voice to the patient.

A) Provide a board of commonly used needs and phrases.

A patient who suffered an ischemic stroke now has disturbed sensory perception. What principle should guide the nurse's care of this patient? A) The patient should be approached on the side where visual perception is intact. B) Attention to the affected side should be minimized in order to decrease anxiety. C) The patient should avoid turning in the direction of the defective visual field to minimize shoulder subluxation. D) The patient should be approached on the opposite side of where the visual perception is intact to promote recovery.

A) The patient should be approached on the side where visual perception is intact.

A patient who has experienced an ischemic stroke has been admitted to the medical unit. The patient's family in adamant that she remain on bed rest to hasten her recovery and to conserve energy. What principle of care should inform the nurse's response to the family? A) The patient should mobilize as soon as she is physically able. B) To prevent contractures and muscle atrophy, bed rest should not exceed 4 weeks. C) The patient should remain on bed rest until she expresses a desire to mobilize. D) Lack of mobility will greatly increase the patient's risk of stroke recurrence.

A) The patient should mobilize as soon as she is physically able.

types of hypersensitivity reactions

ACID 1: A naphylaxis - IgE-mediated degranulation of mast cells and basophils 2: C ytotoxic - IgG mediated complex fixation 3: I mmune complex - IgG or IgM antigen-antibody complex deposition 4: D elayed - T cell mediated

Which of the following neurotransmitters are deficient in myasthenia gravis? Dopamine Acetylcholine Serotonin GABA

Acetylcholine Explanation: A decrease in the amount of acetylcholine causes myasthenia gravis. A decrease of serotonin leads to depression. Parkinson's disease is caused by a depletion of dopamine. Decreased levels of GABA may cause seizures.

The nurse is completing a neurological assessment and uses the whisper test to assess which cranial nerve? Olfactory (I) Facial (VII) Acoustic (VIII) Vagus (X)

Acoustic (VIII) Explanation: Clinical examination of the acoustic nerve, or vestibulocochlear nerve, can be done by the whisper test. Having the client say "ah" tests the vagus nerve. Observing for symmetry when the client performs facial movements tests the facial nerve. The olfactory nerve is tested by having the client identify specific odors.

sickle cell treatment / management

Acute: - fluids - pain medication (specifically Morphine because it is also a smooth muscle dilator) - blood transfusion - eventually transplant may be indicated Long-term: - hydroxyurea - educate patient about importance of hydration

SSD is very common with a history of

Adverse Childhood Event or neglect

A client taking an antidepressant has experienced a 12-pound weight gain in 1 month as a side effect of the medication. Which of the following are nursing interventions to help this patient with this problem? Select all that apply. Recommend daily exercise. Recommend a nutritionally balanced diet. Remind the patient that weight gain is better than feeling depressed. Reassure the patient that the weight gain is not that significant. Advocate with the physician to consider changing the medication.

Advocate with the physician to consider changing the medication. Recommend a nutritionally balanced diet. Recommend daily exercise. To relieve the side effect of weight gain from an antidepressant, appropriate nursing interventions are to help the client explore a change in medication, promote a nutritionally balanced diet, and recommend regular exercise.

A nurse is working with a patient to establish a bowel training program. Based on the nurse's understanding of bowel function, the nurse would suggest planning for bowel evacuation at which time? Before bed Upon arising After breakfast Around lunchtime

After breakfast Explanation: Natural gastrocolic and duodenocolic reflexes occur about 30 minutes after a meal; therefore, after breakfast is one of the best times to plan for bowel evacuation.

A nurse is assigned to care for a patient with ascites, secondary to cirrhosis. The nurse understands that the fluid accumulation in the peritoneal cavity results from a combination of factors including an alteration in oncotic pressure gradients and increased capillary permeability. Therefore, the nurse monitors the serum level of the plasma protein responsible for maintaining oncotic pressure, which is: Albumin. Prothrombin. Globulin. Fibrinogen.

Albumin. Explanation: Albumin, only produced in the liver, is essential for maintaining oncotic pressure in the vascular system. A decrease in oncotic pressure due to low albumin causes fluid to leak into the peritoneal cavity.

function of chemical mediators

Allergen triggers the B cell to make IgE antibody, which attaches to the mast cell. When that allergen reappears, it binds to the IgE and triggers the mast cell to release its chemicals.

a nurse is reviewing the laboratory results of an adolescent female client and notes a WBC count of 16,000 with increased immature neutrophils (bands) and normal monocytes. which of the following is the appropriate analysis of the results? an acute infectious process neutropenia allergic reaction a resolving inflammatory process

An acute infectious process MY ANSWER The white blood cell (WBC) count is greatly elevated; however, even more telling is the elevated neutrophil count, sometimes referred to as a "shift to the left." So, with the combined information from the elevated WBC count indicating infection or inflammation and the elevated neutrophil count indicating an acute process, the appropriate analysis is that the client has an acute infectious process.

Which of the following is accurate regarding acquired immunity? Select all that apply. An immunologic response acquired during life but not present at birth Usually develops as a result of exposure to an antigen through immunization A nonspecific immunity present at birth Can develop by contracting a disease Also know as innate immunity

An immunologic response acquired during life but not present at birth Usually develops as a result of exposure to an antigen through immunization Can develop by contracting a disease rationale: Acquired immunity is a immunologic responses acquired during life but not present at birth, and usually develops as a result of exposure to an antigen through immunization (vaccination) or by contracting a disease, both of which generate a protective immune response. Natural (innate) immunity is a nonspecific immunity present at birth that provides protection against an infectious agent without ever encountering it before.

A client was admitted to the psychiatric intensive care unit with schizophrenia. Among the client's signs and symptoms, the client was experiencing nihilistic delusions. The nurse understands that these delusions involve a belief about what? An impending calamity, such as death Feeling of being watched, such as by the government Belief that communications intended for a broad audience have special meaning for the client Possession of exceptional powers, such as the ability to communicate with a powerful person

An impending calamity, such as death Explanation: Delusions are erroneous, fixed beliefs that cannot be changed by reasonable argument. Nihilistic delusions involve the belief that one is dead or a calamity is impending; when these delusions involve bodily illness, they take hypochondriacal concerns to the utmost extreme. Grandiose delusions involve the belief that one has exceptional powers, wealth, skill, influence, or destiny. Persecutory delusions involve the belief that one is being watched, ridiculed, harmed, or plotted against. Referential delusions, or ideas of reference, involve a belief that communications such as television broadcasts or website posts are directed toward the client or have special meaning for the client.

allergy

An inappropriate, often harmful response of the immune system to normally harmless substances Hypersensitive reaction to an allergen initiated by immunologic mechanisms that is usually mediated by IgE antibodies

potential problems or complications of allergic rhinitis

Anaphylaxis Impaired breathing Nonadherence to therapeutic regimen

Which of the following would the nurse identify as a negative symptom associated with schizophrenia? Loose associations Anhedonia Hallucinations Bizarre behavior

Anhedonia Explanation: Positive symptoms reflect an excess or distortion of normal functions, including delusions and hallucinations. Negative symptoms reflect a lessening or loss of normal functions, such as restriction or flattening in the range and intensity of emotion (diminished emotional expression), reduced fluency and productivity of thought and speech (alogia), withdrawal and inability to initiate and persist in goal-directed activity (avolition), and inability to experience pleasure (anhedonia). Loose associations reflect disorganized thinking, a neurocognitive impairment.

The physician performs a bone marrow biopsy from the posterior iliac crest on a client with pancytopenia. What intervention should the nurse perform after the procedure? Elevate the head of the bed to 45 degrees Administer a topical analgesic to control pain at the site Apply pressure over the site for 5-7 minutes Pack the wound with half-inch sterile gauze

Apply pressure over the site for 5-7 minutes Explanation: Hazards of either bone marrow aspiration or biopsy include bleeding and infection. The risk of bleeding is somewhat increased if the client's platelet count is low or if the client has been taking a medication (e.g., aspirin) that alters platelet function. After the marrow sample is obtained, pressure is applied to the site for several minutes. The site is then covered with a sterile dressing.

Which of the following signs, symptoms, or diseases is most commonly seen when evaluating a patient for the "5 A's" of Alzheimer's disease? Agoraphobia Adenomyosis Achromatopsia Acrodysostosis Abdominal Pain Apraxia

Apraxia Apraxia is the inability to perform purposeful movements or once familiar tasks, such as dressing, cooking, or shaving.

The nurse plans care for a client in the primary care setting who is diagnosed with somatic symptom disorder (SSD). Which intervention does the nurse include in the client's plan of care? Ensure the client sees different practitioners at each visit. Conduct a holistic client assessment with a full head-to-toe physical examination. Use cognitive behavioral therapy (CBT) techniques with the client. Ask the client if there is a history of trauma.

Ask the client if there is a history of trauma. Explanation: The cornerstones of management are trust and believing. The provider should assess for any stressors, life changes, and/or a history of trauma that has been linked to the excessive expression of somatic symptoms. Ideally, the client should see only one health care provider at regularly scheduled visits. During each primary care visit, the provider should conduct a holistic assessment including a partial physical examination of the organ system, focusing on the areas where the client reports issues. Physical symptoms are treated conservatively using the least intrusive approach. In the mental health setting, the use of cognitive behavior therapy (CBT) can be effective; however, this client is seeking care in the primary care setting.

A client's depression is being treated in the community with phenelzine. The client has presented to the clinic stating, "I had a few beers and I'm feeling absolutely miserable." What is the nurse's best action? Assess the client's jugular venous pressure Perform a Mini Mental Status Examination (MMSE) Call an emergency code Assess the client's blood pressure

Assess the client's blood pressure Explanation: Combining phenelzine with beer can precipitate a hypertensive crisis. There is no immediate indication that an emergency code is needed. The client's jugular venous pressure is less likely to be affected and is not a priority for assessment. Performing the MMSE is not a short-term priority.

Which nursing intervention is the priority for a client in myasthenic crisis? Ensuring adequate nutritional support Assessing respiratory effort Preparing for plasmapheresis Administering intravenous immunoglobin (IVIG) per orders

Assessing respiratory effort Explanation: A client in myasthenic crisis has severe muscle weakness, including the muscles needed to support respiratory effort. Myasthenic crisis can lead to respiratory failure and death if not recognized early. Administering IVIG, preparing for plasmapheresis, and ensuring adequate nutritional support are important and appropriate interventions, but maintaining adequate respiratory status or support is the priority during the crisis.

A client feels faint and becomes dizzy after donating a unit of blood. Which actions will the nurse perform at this time? Select all that apply. Place the client's head lower than the knees Assist the client to lie down Assess the client's apical heart rate Observe the clent for 30 minutes Provide 1000 mL of fluid stat

Assist the client to lie down Assess the client's apical heart rate Place the client's head lower than the knees Observe the client for 30 minutes Explanation: Fainting may occur after blood donation and may be related to emotional factors, a vasovagal reaction, or prolonged fasting before donation. Because of the loss of blood volume, hypotension and syncope may occur when the client assumes an erect position. A client who appears pale or reports feeling faint should immediately lie down or sit with the head lowered below the knees. The client should be observed for another 30 minutes. Giving 1000 mL of fluid would be too much fluid at once, and the apical heart rate does not need to be assessed.

Clinical Manifestations of HIV

Asymptomatic during first stage or may exhibit fatigue or skin rash Later stages have variety of symptoms related to immunosuppressed state respiratory manifestations: - SOB, dyspnea, cough, chest pain - Pneumocystis pneumonia, Mycobacterium avium complex, Tuberculosis

Which term refers to the inability to coordinate muscle movements, resulting difficulty walking? Spasticity Rigidity Ataxia Agnosia

Ataxia Explanation: Ataxia is the inability to coordinate voluntary muscle action; tremors (rhythmic, involuntary movements) noted at rest or during movement suggest a problem in the anatomic areas responsible for balance and coordination. Agnosia is the loss of ability to recognize objects through a particular sensory system. Spasticity is the sustained increase in tension of a muscle when it is passively lengthened or stretched.

A schizophrenic patient presents to your clinic continuing to hear voices. Which of the following best describes this type of hallucination?

Auditory This is when the patient hears voices or sounds. It is common in schizophrenia.

a nurse in an emergency department is caring for a client who has anaphylaxis following a bee sting. which of the following actions should the nurse take first? assess the client's level of consciousness administer epinephrine auscultate for wheezing monitor for hypotension

Auscultate for wheezing. When using the airway, breathing, circulation approach to client care, the nurse should place the priority on assessing the client's respiratory status. Bronchoconstriction or closure of the upper airway may occur, which places the client at risk for respiratory arrest.

Parkinson's disease (PD)

Autoimmune disorder with acute attack of peripheral nerve myelin Degeneration and death of neurons Rapid demyelination may produce respiratory failure and autonomic nervous system dysfunction with CV instability Decrease levels of dopamine - dopamine is the happy hormone, feel good hormone Most often follows a viral infection

Immune Thrombocytopenic Purpura (ITP)

Autoimmune platelet destruction is a common cause of thrombocytopenia and should be supsected in patients with echymoses, petechiae, mucosal bleeding, and no other obvious causes of thrombocytopenia (ex- medications, bone marrow failure) affects all ages - common in kids and young women. 2 forms chronic and acute.

The client is about to have a skin test for an allergic disorder. What critical instruction should the nurse give this client? Avoid sunlight for 48 to 72 hours before the test. Avoid red meat for 48 to 72 hours before the test. Avoid antihistamines and cold preparations for 48 to 72 hours before the test. Avoid strenuous physical activity for 24 hours before the test.

Avoid antihistamines and cold preparations for 48 to 72 hours before the test. Explanation: The nurse should instruct the client to avoid taking prescribed or over-the-counter antihistamine or cold preparations for at least 48 to 72 hours before a skin test because this reduces the potential for false-negative test results. The nurse should not ask the client to avoid red meat, strenuous physical activity, or sunlight because these do not affect the test results.

A rehabilitation nurse caring for a patient who has had a stroke is approached by the patient's family and asked why the patient has to do so much for herself when she is obviously struggling. What would be the nurse's best answer? A) "We are trying to help her be as useful as she possibly can." B) "The focus on care in a rehabilitation facility is to help the patient to resume as much self-care as possible." C) "We aren't here to care for her the way the hospital staff did; we are here to help her get better so she can go home." D) "Rehabilitation means helping patients do exactly what they did before their stroke."

B) "The focus on care in a rehabilitation facility is to help the patient to resume as much self-care as possible."

When caring for a patient who had a hemorrhagic stroke, close monitoring of vital signs and neurologic changes is imperative. What is the earliest sign of deterioration in a patient with a hemorrhagic stroke of which the nurse should be aware? A) Generalized pain B) Alteration in level of consciousness (LOC) C) Tonic-clonic seizures D) Shortness of breath

B) Alteration in level of consciousness (LOC)

A preceptor is discussing stroke with a new nurse on the unit. The preceptor would tell the new nurse which cardiac dysrhythmia is associated with cardiogenic embolic strokes? A) Ventricular tachycardia B) Atrial fibrillation C) Supraventricular tachycardia D) Bundle branch block

B) Atrial fibrillation

When caring for a patient who has had a stroke, a priority is reduction of ICP. What patient position is most consistent with this goal? A) Head turned slightly to the right side B) Elevation of the head of the bed C) Position changes every 15 minutes while awake D) Extension of the neck

B) Elevation of the head of the bed

A patient has recently begun mobilizing during the recovery from an ischemic stroke. To protect the patient's safety during mobilization, the nurse should perform what action? A) Support the patient's full body weight with a waist belt during ambulation. B) Have a colleague follow the patient closely with a wheelchair. C) Avoid mobilizing the patient in the early morning or late evening. D) Ensure that the patient's family members do not participate in mobilization.

B) Have a colleague follow the patient closely with a wheelchair.

A patient has been admitted to the ICU after being recently diagnosed with an aneurysm and the patient's admission orders include specific aneurysm precautions. What nursing action will the nurse incorporate into the patient's plan of care? A) Elevate the head of the bed to 45 degrees. B) Maintain the patient on complete bed rest. C) Administer enemas when the patient is constipated. D) Avoid use of thigh-high elastic compression stockings.

B) Maintain the patient on complete bed rest.

As a member of the stroke team, the nurse knows that thrombolytic therapy carries the potential for benefit and for harm. The nurse should be cognizant of what contraindications for thrombolytic therapy? Select all that apply. A) INR above 1.0 B) Recent intracranial pathology C) Sudden symptom onset D) Current anticoagulation therapy E) Symptom onset greater than 3 hours prior to admission

B) Recent intracranial pathology D) Current anticoagulation therapy E) Symptom onset greater than 3 hours prior to admission

The nurse is caring for a patient recovering from an ischemic stroke. What intervention best addresses a potential complication after an ischemic stroke? A) Providing frequent small meals rather than three larger meals B) Teaching the patient to perform deep breathing and coughing exercises C) Keeping a urinary catheter in situ for the full duration of recovery D) Limiting intake of insoluble fiber

B) Teaching the patient to perform deep breathing and coughing exercises

The nurse is performing stroke risk screenings at a hospital open house. The nurse has identified four patients who might be at risk for a stroke. Which patient is likely at the highest risk for a hemorrhagic stroke? A) White female, age 60, with history of excessive alcohol intake B) White male, age 60, with history of uncontrolled hypertension C) Black male, age 60, with history of diabetes D) Black male, age 50, with history of smoking

B) White male, age 60, with history of uncontrolled hypertension

Which of the following statements should help guide nursing care for the patient with fibromyalgia? A.Patients with fibromyalgia rarely respond to treatment B.Patients with fibromyalgia may feel as if their symptoms have not been taken seriously C.All patients with fibromyalgia have the same type of symptoms D.Patients with fibromyalgia will eventually lose their ability to walk

B. Patients with fibromyalgia may feel as if their symptoms have not been taken seriously Rationale: Typically, patients with fibromyalgia have endured their symptoms for a long period of time and may feel as if their symptoms have not been taken seriously. Nurses need to pay special attention to supporting these patients and providing encouragement as they begin their program of therapy. Symptoms of disease vary from patient to patient, as do responses to different treatments. Fibromyalgia does not lead to an inability to walk.

Which of the following are common physical signs and symptoms of rheumatoid arthritis? A.Pain on weight-bearing, rash, and low-grade fever B.Small joint swelling, joint stiffness, and joint pain C.Crepitus, development of Heberden's nodes, and anemia D.Fatigue, leukopenia, and joint pain

B. Small joint swelling, joint stiffness, and joint pain Rationale: The most common symptom in the rheumatic diseases is pain. Other common symptoms include joint swelling, limited movement, stiffness, weakness, and fatigue.

A nurse is caring for a client who is receiving 2 units of packed RBCs. Nurses' Notes Initiated first unit of blood. After 30 min of blood infusing client reports chills along with headache. Flushing of face and client appears anxious. Alert and oriented to place and time. Lungs clear to auscultation. Obtained vital signs and oxygen saturation. IV site is clean and intact. Vital Signs Blood pressure 138/76 mm HgHeart rate 96/minRespiratory rate 22/minTemperature 38.5º C (101.3º F)Oxygen saturation 94% on room air Which of the following actions should the nurse take? (Select all that apply.) Begin infusing 0.9% sodium chloride solution. Stop the client's blood transfusion. Request a prescription for an antipyretic medication. Obtain a prescription for a diuretic medication. Insert an indwelling urinary catheter. Obtain a culture sample of the client's blood. Request a prescription for an antihistamine medication.

Begin infusing 0.9% sodium chloride soliution stop the blood transfusion request an rx for an antipyretic Stop the client's blood transfusion is correct. The nurse should identify the client is experiencing a nonhemolytic febrile reaction to the donor blood and should stop the transfusion immediately. Request a prescription for an antipyretic medication is correct. The nurse should identify the client is experiencing a nonhemolytic febrile reaction to the donor blood, which is causing the client to experience fever. Therefore, the nurse should request a prescription for an antipyretic medication for the client. Begin infusing 0.9% sodium chloride solution is correct. The nurse should identify the client is experiencing a nonhemolytic febrile reaction to the donor blood. The nurse should stop the transfusion immediately and begin infusing 0.9% sodium chloride solution to keep the client's vein open.

A nurse is caring for a client receiving a tricyclic antidepressant and is monitoring for anticholinergic side effects. Anticholinergic effects include which of the following? Urinary incontinence Hyperactive bowel sounds Moist skin Blurred vision

Blurred vision Explanation: Anticholinergic effects are prominent with tricyclic antidepressants. These include potentiation of central nervous system drugs, dry mucous membranes, warm and dry skin, blurred vision, decreased bowel motility, and urinary retention.

A patient who has suffered a stroke is unable to maintain respiration and is intubated and placed on mechanical ventilator support. What portion of the brain is most likely responsible for the inability to breathe? Parietal lobe Brain stem Frontal lobe Occipital lobe

Brain stem Explanation: The brain stem consists of the midbrain, pons, and medulla oblongata (see Fig. 65-2). Portions of the pons help regulate respiration. Motor fibers from the brain to the spinal cord and sensory fibers from the spinal cord to the brain are located in the medulla. Reflex centers for respiration, blood pressure, heart rate, coughing, vomiting, swallowing, and sneezing are located in the medulla.

The pathophysiology of an ischemic stroke involves the ischemic cascade, which includes the following steps: 1. Change in pH 2. Blood flow decreases 3. A switch to anaerobic respiration 4. Membrane pumps fail 5. Cells cease to function 6. Lactic acid is generated Put these steps in order in which they occur. A) 635241 B) 352416 C) 236145 D) 162534

C) 236145

The nurse is reviewing the medication administration record of a female patient who possesses numerous risk factors for stroke. Which of the woman's medications carries the greatest potential for reducing her risk of stroke? A) Naproxen 250 PO b.i.d. B) Calcium carbonate 1,000 mg PO b.i.d. C) Aspirin 81 mg PO o.d. D) Lorazepam 1 mg SL b.i.d. PRN

C) Aspirin 81 mg PO o.d.

A patient with a new diagnosis of ischemic stroke is deemed to be a candidate for treatment with tissue plasminogen activator (t-PA) and has been admitted to the ICU. In addition to closely monitoring the patient's cardiac and neurologic status, the nurse monitors the patient for signs of what complication? A) Acute pain B) Septicemia C) Bleeding D) Seizures

C) Bleeding

The nurse is discharging home a patient who suffered a stroke. He has a flaccid right arm and leg and is experiencing problems with urinary incontinence. The nurse makes a referral to a home health nurse because of an awareness of what common patient response to a change in body image? A) Denial B) Fear C) Depression D) Disassociation

C) Depression

A patient diagnosed with a hemorrhagic stroke has been admitted to the neurologic ICU. The nurse knows that teaching for the patient and family needs to begin as soon as the patient is settled on the unit and will continue until the patient is discharged. What will family education need to include? A) How to differentiate between hemorrhagic and ischemic stroke B) Risk factors for ischemic stroke C) How to correctly modify the home environment D) Techniques for adjusting the patient's medication dosages at home

C) How to correctly modify the home environment

1. A patient has had an ischemic stroke and has been admitted to the medical unit. What action should the nurse perform to best prevent joint deformities? A) Place the patient in the prone position for 30 minutes/day. B) Assist the patient in acutely flexing the thigh to promote movement. C) Place a pillow in the axilla when there is limited external rotation. D) Place patient's hand in pronation.

C) Place a pillow in the axilla when there is limited external rotation.

The nurse is preparing health education for a patient who is being discharged after hospitalization for a hemorrhagic stroke. What content should the nurse include in this education? A) Mild, intermittent seizures can be expected. B) Take ibuprofen for complaints of a serious headache. C) Take antihypertensive medication as ordered. D) Drowsiness is normal for the first week after discharge.

C) Take antihypertensive medication as ordered.

The nurse is caring for a patient diagnosed with an ischemic stroke and knows that effective positioning of the patient is important. Which of the following should be integrated into the patient's plan of care? A) The patient's hip joint should be maintained in a flexed position. B) The patient should be in a supine position unless ambulating. C) The patient should be placed in a prone position for 15 to 30 minutes several times a day. D) The patient should be placed in a Trendelenberg position two to three times daily to promote cerebral perfusion.

C) The patient should be placed in a prone position for 15 to 30 minutes several times a day.

A patient diagnosed with transient ischemic attacks (TIAs) is scheduled for a carotid endarterectomy. The nurse explains that this procedure will be done for what purpose? A) To decrease cerebral edema B) To prevent seizure activity that is common following a TIA C) To remove atherosclerotic plaques blocking cerebral flow D) To determine the cause of the TIA

C) To remove atherosclerotic plaques blocking cerebral flow

What part of the brain controls and coordinates muscle movement? Cerebellum Cerebrum Midbrain Brain stem

Cerebellum Explanation: The cerebellum, which is located behind and below the cerebrum, controls and coordinates muscle movement.

A nurse is preparing a client for a lumbar puncture. The client has heard about post-lumbar puncture headaches and asks what causes them. The nurse tells the client that these headaches are caused by which of the following? Not ambulating soon enough after the procedure Damage to the spinal cord Traumatic puncture Cerebral spinal fluid leakage at the puncture site

Cerebral spinal fluid leakage at the puncture site Explanation: The headache is caused by cerebral spinal fluid (CSF) leakage at the puncture site. The supply of CSF in the cranium is depleted so that there is not enough to cushion and stabilize the brain. When the client assumes an upright position, tension and stretching of the venous sinuses and pain-sensitive structures occur.

A client with myelodysplastic syndromes (MDS) receives routine blood transfusions. Which treatment will the nurse expect to be prescribed to prevent the development of iron overload? Romiplostim Chelation therapy Epoetin alpha Eltrombopag

Chelation therapy Explanation: Iron overload is a problem for clients with MDS, especially in those who routinely receive PRBC transfusions (transfusion dependent). Surplus iron is deposited in cells within the reticuloendothelial system, and later in parenchymal organs. To prevent or reverse the complications of iron overload, iron chelation therapy is commonly implemented. Romiplostim and eltrombopag are used to stimulate the proliferation and differentiation of megakaryocytes into platelets within the bone marrow. Epoetin alpha may be used to improve anemia and decrease the need for blood transfusions.

A client with schizoaffective disorder is engaging in an extremely long conversation about a current affairs in the world. The client goes on to provide the nurse with minute details. The nurse interprets this as suggesting what? Circumstantiality Clang association Neologism Verbigeration

Circumstantiality Explanation: The client is demonstrating circumstantiality, which refers to extremely detailed and lengthy discourse about a topic.This can be commonly found in a client with euphoric or elevated mood due to the affective component of schizoaffective disorder. Clang associations are ideas that are related to one another based on sound or rhyming rather than meaning. Neologisms are words invented by the client. A verbigeration is the stereotyped repetition of words or phrases that may or may not have meaning to the listener.

The client has a somatic symptom illness. During individual therapy, the client yells at the nurse, "You are all quacks! Can't you see I am sick?" Which knowledge statement would help the nurse to work most effectively with this client? The client will never be free of somatic symptoms. The nurse has done everything possible to treat the client. Physical illness is the root of the client's problems. Client progress is expected to be very slow.

Client progress is expected to be very slow. Explanation: Clients who cope through physical symptoms can be frustrating for the nurse. Initially, they are unwilling to consider that anything other than major physical illness is the root of all their problems. The client's progress is slow and painstaking, if any change happens at all. The nurse should be realistic about the small successes that can be achieved in any given period. To enhance the ongoing relationship, the nurse must be able to accept the client and his or her continued complaints and criticisms while remaining nonjudgmental. Psychological factors are the root of the illness. The nurse can never know if he or she has done all that can be done for the client and the nurse should not "write off" the client as untreatable.

The nurse is providing care to a client with somatic symptom disorder (SSD). Which would the nurse expect to be included in the client's plan of care? Multiple provider evaluations Cognitive behavior therapy Mood stabilizers to manage the symptoms Electroconvulsive therapy

Cognitive behavior therapy Explanation: The cornerstone of management is trust and believing. Ideally, the client should see only one health care provider at regularly scheduled visits. During each primary care visit, the provider should conduct a partial physical examination of the organ system in which the client has complaints. Physical symptoms are treated conservatively using the least intrusive approach. In the mental health setting, the use of cognitive behavior therapy is effective. Medications may be used, such as monoamine oxidase inhibitors or selective serotonin reuptake inhibitors, but not mood stabilizers. Electroconvulsive therapy is not typically used.

The nurse is planning the inpatient care of a client who has been admitted with major depression. The client's plan of care includes regular exercise, but the client is reluctant to participate due to a lack of energy and motivation. What is the nurse's best action? Describe the benefits of exercise to the client and state that privileges will be lost if the client does not participate Teach the client isometric exercises that the client can complete while in bed Arrange for the client to exercise approximately 1 hour after antidepressant administration Collaborate with the client to choose a manageable amount of exercise and acknowledge the client's subsequent effort

Collaborate with the client to choose a manageable amount of exercise and acknowledge the client's subsequent effort Explanation: In general, a good approach is to collaborate with the client to find an agreeable solution. Energy levels do not change in the short term following antidepressant administration. Threatening the loss of privileges is an inappropriate and unnecessary approach. The nurse should not accommodate the client's remaining in bed unless it is a necessity.

The nurse is assessing the pupils of a patient who has had a head injury. What does the nurse recognize as a parasympathetic effect? One pupil is dilated and the opposite pupil is normal Dilated pupils Constricted pupils Roth's spots

Constricted pupils Explanation: Constricted pupils are a parasympathetic effect; dilated pupils are a sympathetic effect.

A client is admitted to the hospital with a diagnosis of pneumonia. The client informs the nurse of having several drug allergies. The physician has ordered an antibiotic as well as several other medications for cough and fever. What should the nurse do prior to administering the medications? Administer the medications that the physician ordered. Consult drug references to make sure the medicines do not contain substances which the client is hypersensitive. Call the pharmacy and let them know the client has several drug allergies. Give the client one medicine at a time and observe for allergic reactions.

Consult drug references to make sure the medicines do not contain substances which the client is hypersensitive. Explanation: Clear identification of any substances to which the client is allergic is essential. The nurse must consult drug references to verify that prescribed medications do not contain substances to which the client is hypersensitive. Administering the medications or giving one at a time may cause the client to have an allergic reaction. The nurse may call the pharmacy but still maintains responsibility for the medications administered.

The nurse is instructing the client with polycythemia vera how to perform isometric exercises such as contracting and relaxing the quadriceps and gluteal muscle during periods of inactivity. What does the nurse understand is the rationale for this type of exercise? Isometric exercise decreases the workload of the heart and restores oxygenated blood flow. This type of exercise increases arterial circulation as it returns to the heart. Isometric exercise programs are inclusive of all muscle groups and have an aerobic effect to increase the heart rate. Contraction of skeletal muscle compresses the walls of veins and increases the circulation of venous blood as it returns to the heart.

Contraction of skeletal muscle compresses the walls of veins and increases the circulation of venous blood as it returns to the heart. Explanation: Isometric exercise induce contraction of skeletal muscle so that it compresses the walls of veins and increases the circulation of venous blood as it returns to the heart. Isometric exercises do not have an aerobic effect and should not increase the heart rate; although, it may increase blood pressure. Isometric exercise does not decrease the workload of the heart. Arterial flow moves blood flow away from the heart after being oxygenated.

The nurse is obtaining the medical history of a client with Crohn's disease. What medication would the nurse include when asking about what medications the client has taken for suppression of the inflammatory and immune response? Diuretics Nonsteroidal anti-inflammatory Angiotensin-converting enzyme inhibitors (ACE-I) Corticosteroids

Corticosteroids Explanation: The nurse obtains a history of immunizations, recent and past infectious diseases, and recent exposure to infectious diseases. The nurse reviews the client's drug history because certain drugs, such as corticosteroids, suppress the inflammatory and immune responses. Nonsteroidal anti-inflammatory medication does not suppress the inflammatory and immune responses of Crohn's disease. An ACE-I prevents the conversion of angiotensin I to angiotensin II and does not suppress the inflammatory or immune response. Diuretics also do not suppress the immune response but help reduce excess fluid from the kidneys.

The nurse should be alert to which adverse assessment finding when transfusing a unit of packed red blood cells (PRBCs) too rapidly? Crackles auscultated bilaterally Oral temperature of 97°F Pain and tenderness in calf area Respiratory rate of 10 breaths/minute

Crackles auscultated bilaterally Explanation: Increasing the flow rate of a blood transfusion too rapidly can result in circulatory overload. Fluid overload can be manifested by crackles in the lungs. A decreased respiratory rate and decreased temperature are not manifestations of fluid overload. Pain and tenderness in the calf area may indicate a thrombosis which is not as common a manifestation as fluid overload.

There are 12 pairs of cranial nerves. Only three are sensory. Select the cranial nerve that is affected with decreased visual fields. Cranial nerve III Cranial nerve IV Cranial nerve II Cranial nerve I

Cranial nerve II Explanation: The three sensory cranial nerves are I, II and VIII. Cranial nerve II (optic) is affected with decreased visual fields and acuity.

The nurse is assessing the throat of a client with throat pain. In asking the client to stick out the tongue, the nurse is also assessing which cranial nerve? Cranial nerve XII Cranial nerve XI Cranial nerve I Cranial nerve V

Cranial nerve XII Explanation: Assessment of the movement of the tongue is related to cranial nerve XII, the hypoglossal nerve. Cranial nerve I is the olfactory nerve. Cranial nerve V is the trigeminal nerve responsible for sensation to the face and chewing. Cranial nerve XI is the spinal or accessory nerve responsible for head and shoulder and shoulder movement.

Which condition is a rare, transmissible, progressive fatal disease of the central nervous system characterized by spongiform degeneration of the gray matter of the brain? Huntington disease Multiple sclerosis Parkinson disease Creutzfeldt-Jakob disease

Creutzfeldt-Jakob disease Explanation: Creutzfeldt-Jakob disease causes severe dementia and myoclonus. Multiple sclerosis is a chronic, degenerative, progressive disease of the central nervous system characterized by the occurrence of small patches of demyelination in the brain and spinal cord. Parkinson disease is associated with decreased levels of dopamine due to destruction of pigmented neuronal cells in the substantia nigra in the basal ganglia of the brain. Huntington disease is a chronic, progressive, hereditary disease of the nervous system that results in progressive involuntary dancelike movements and dementia.

An experiment is designed to determine specific cell types involved in cell-mediated immune response. The experimenter is interested in finding cells that attack the antigen directly by altering the cell membrane and causing cell lysis. Which cells should be isolated? Macrophages Cytotoxic T cells B cells Helper T cells

Cytotoxic T cells Explanation: Cytotoxic T cells (killer T cells) attack the antigen directly by altering the cell membrane and causing cell lysis (disintegration) and by releasing cytolytic enzymes and cytokines. Lymphokines can recruit, activate, and regulate other lymphocytes and white blood cells. These cells then assist in destroying the invading organism.

A nurse in the ICU is providing care for a patient who has been admitted with a hemorrhagic stroke. The nurse is performing frequent neurologic assessments and observes that the patient is becoming progressively more drowsy over the course of the day. What is the nurse's best response to this assessment finding? A) Report this finding to the physician as an indication of decreased metabolism. B) Provide more stimulation to the patient and monitor the patient closely. C) Recognize this as the expected clinical course of a hemorrhagic stroke. D) Report this to the physician as a possible sign of clinical deterioration.

D) Report this to the physician as a possible sign of clinical deterioration.

A patient recovering from a stroke has severe shoulder pain from subluxation of the shoulder and is being cared for on the unit. To prevent further injury and pain, the nurse caring for this patient is aware of what principle of care? A) The patient should be fitted with a cast because use of a sling should be avoided due to adduction of the affected shoulder. B) Elevation of the arm and hand can lead to further complications associated with edema. C) Passively exercising the affected extremity is avoided in order to minimize pain. D) The patient should be taught to interlace fingers, place palms together, and slowly bring scapulae forward to avoid excessive force to shoulder.

D) The patient should be taught to interlace fingers, place palms together, and slowly bring scapulae forward to avoid excessive force to shoulder.

A patient with chronic kidney disease is being examined by the nurse practitioner for anemia. The nurse has reviewed the laboratory data for hemoglobin and RBC count. What other test results would the nurse anticipate observing? Decreased total iron-binding capacity Decreased level of erythropoietin Increased mean corpuscular volume Increased reticulocyte count

Decreased level of erythropoietin Explanation: Differentiation of the primitive myeloid stem cell into an erythroblast is stimulated by erythropoietin, a hormone produced primarily by the kidney. If the kidney detects low levels of oxygen, as occurs when fewer red cells are available to bind oxygen (i.e., anemia), or with people living at high altitudes with lower atmospheric oxygen concentrations, erythropoietin levels increase. The increased erythropoietin then stimulates the marrow to increase production of erythrocytes. The entire process of erythropoiesis typically takes 5 days (Cook, Ineck, & Lyons, 2011). For normal erythrocyte production, the bone marrow also requires iron, vitamin B12, folate, pyridoxine (vitamin B6), protein, and other factors. A deficiency of these factors during erythropoiesis can result in decreased red cell production and anemia.

The nurse expects which assessment finding when caring for a client with a decreased hemoglobin level? Increased bruising. Bright red venous blood. Decreased oxygen level. Elevated temperature.

Decreased oxygen level. Explanation: Hemoglobin carries oxygen; a decreased hemoglobin level results in decreased oxygen. An elevated temperature is a sign of infection and can result from decreased white blood cells. Arterial blood is more oxygen saturated and brighter red in color than venous blood. Increased bruising results from a decreased platelet level, not decreased hemoglobin.

chronic subdural hematoma

Develops over weeks to months seen more in older adults Causative injury may be minor and forgotten Clinical signs and symptoms may fluctuate Treatment is evacuation of the clot with burr holes or craniotomy

Which is a sympathetic effect of the nervous system? Decreased blood pressure Increased peristalsis Dilated pupils Decreased respiratory rate

Dilated pupils Explanation: Dilated pupils are a sympathetic effect of the nervous system, whereas constricted pupils are a parasympathetic effect. Decreased blood pressure is a parasympathetic effect, whereas increased blood pressure is a sympathetic effect. Increased peristalsis is a parasympathetic effect, but decreased peristalsis is a sympathetic effect. Decreased respiratory rate is a parasympathetic effect, and increased respiratory rate is a sympathetic effect.

A client with schizophrenia is exhibiting hallucinations and delusions. The mental health nurse knows that these symptoms are associated with hyperactivity of which neurotransmitter? Dopamine Serotonin Gamma-aminobutyric acid (GABA) Norepinephrine

Dopamine Explanation: Positive symptoms of schizophrenia (specifically hallucinations and delusions) are thought to be related to dopamine hyperactivity in the mesolimbic tract at the D2 receptor site of the striatal area, where memory and emotion are regulated. Other receptors are also involved in dopamine neurotransmission, especially serotonergic receptors. It is becoming clear that schizophrenia does not result from dysregulation of a single neurotransmitter or biogenic amine (e.g., norepinephrine, dopamine, or serotonin). Investigators are also hypothesizing a role for glutamate and GABA because of the complex interconnections of neuronal transmission and the complexity and heterogeneity of schizophrenia symptoms. The N-methyl-D-aspartate (NMDA) class of glutamate receptors is being studied because of the actions of phencyclidine (PCP) at these sites and the similarity of the psychotic behaviors that are produced when someone takes PCP.

A patient is being tested for a gag reflex. When the nurse places the tongue blade to the back of the throat, there is no response elicited. What dysfunction does the nurse determine the patient has? Dysfunction of the spinal accessory nerve Dysfunction of the acoustic nerve Dysfunction of the vagus nerve Dysfunction of the facial nerve

Dysfunction of the vagus nerve Explanation: The vagus nerve (cranial nerve X) controls the gag reflex and is tested by depressing the posterior tongue with a tongue blade. An absent gag reflex is a significant finding, indicating dysfunction of this nerve.

Which disorder is characterized by at least 2 years of depressed mood for more days than not with some additional, less severe symptoms that do not meet the criteria for a major depressive episode? Cyclothymic disorder Hypomania Dysthymic disorder Seasonal affective disorder

Dysthymic disorder Explanation: Dysthymic disorder is characterized by at least 2 years of depressed mood for more days than not with some additional, less severe symptoms that do not meet the criteria for a major depressive episode. Cyclothymic disorder is characterized by 2 years of numerous periods of hypomanic symptoms that do not meet the criteria for bipolar disorder. Seasonal affective disorder occurs in the winter or spring. Hypomania is a period of abnormally and persistently elevated, expansive, or irritable mood lasting 4 days.

A client diagnosed with schizophrenia is exhibiting disorganized behavior and imitating what the nurse is saying. What term is used to identify this behavior? Echolalia Neologism Tangentiality Perseveration

Echolalia Explanation: Echolalia is the involuntary imitation of another person's speech and gestures, is a disorganized behavior. Neologism, and tangentiality reflect disorganized thinking. Perseveration is the persistent adherence to a single idea or topic and verbal repetition of a sentence, phrase or word even when another persona attempts to change the topic.

A client has undergone a lumbar puncture as part of a neurological assessment. The client is put under the care of a nurse after the procedure. Which important postprocedure nursing intervention should be performed to ensure the client's maximum comfort? Keep the room brightly lit and play soothing music in the background Administer antihistamines according to the physician's prescription Encourage the client to drink liberal amounts of fluids Help the client take a brisk walk around the testing area

Encourage the client to drink liberal amounts of fluids Explanation: The nurse should encourage the client to take liberal fluids and should inspect the injection site for swelling or hematoma. These measures help restore the volume of cerebrospinal fluid extracted. The client is administered antihistamines before a test only if he or she is allergic to contrast dye and contrast dye will be used. The room of the client who has undergone a lumbar puncture should be kept dark and quiet. The client should be encouraged to rest, because sensory stimulation tends to magnify discomfort.

Which neurotransmitter inhibits pain transmission? Dopamine Enkephalin Acetylcholine Serotonin

Enkephalin Explanation: Enkephalins are excitatory neurotransmitters that cause pleasurable sensations and inhibit pain transmission. Acetylcholine is an excitatory transmitter that sometimes has inhibitory parasympathetic effects. Serotonin is an inhibitory transmitter that helps control mood and sleep. Dopamine usually is inhibitory, affecting behavior and fine movement.

What safety actions does the nurse need to take for a client receiving oxygen therapy who is undergoing magnetic resonance imaging (MRI)? Securely fasten the client's portable oxygen tank to the bottom of the MRI table after the client has been positioned on the top of the MRI table Note that no special safety actions need to be taken Check the client's oxygen saturation level using a pulse oximeter after the client has been placed on the MRI table Ensure that no client care equipment containing metal enters the room where the MRI is located.

Ensure that no client care equipment containing metal enters the room where the MRI is located. Explanation: For client safety the nurse must make sure no client care equipment that contains metal or metal parts (e.g., portable oxygen tanks) enters the room where the MRI is located. The magnetic field generated by the unit is so strong that any metal-containing items will be strongly attracted and can literally be pulled away with such great force that they can fly like projectiles toward the magnet.

A client who has experienced the recent loss of an infant child and recent immigration to the United States is admitted to the inpatient psychiatric unit with severe symptoms of depression. The client has expressed thoughts of suicide. Which is the nurse's priority intervention for this client? Encouraging attendance at group cognitive-behavioral therapy on the unit. Exploring the grief and loss issues concerning the baby's death. Encouraging the client to express feelings of isolation following the recent immigration. Ensuring that the client is not permitted to use anything that would be potentially dangerous.

Ensuring that the client is not permitted to use anything that would be potentially dangerous. Explanation: Although grief, loss, and isolation may be influencing the client's depressed state, the priority intervention is to prevent self-harm. All the interventions listed are appropriate, but ensuring safety from potential danger is the priority.

A client with severe anemia is admitted to the hospital. Because of religious beliefs, the client is refusing blood transfusions. The nurse anticipates pharmacologic therapy with which drug to stimulate the production of red blood cells? Sargramostim Eltrombopag Filgrastim Epoetin alfa

Epoetin alfa Explanation: Erythropoietin (epoetin alfa) is an effective alternative treatment for clients with chronic anemia secondary to diminished levels of erythropoietin. This medication stimulates erythropoiesis. Filgratism ( Neupogen) and Sargramostim stimulate granulocytosis( increasing WBC count) , Eltrombopag (Promacta) is used to treat aplastic anemia and thrombocytopenia.

The nurse plans to care for a teenage male client experiencing conversion disorder after a bullying incident. The client has stated, "It does not matter if my legs do not work. I will not need them anyway." What is the priority nursing action for this client? Evaluate the client's risk for suicide. Remind the client that disturbed body image is a normal symptom of their conversion disorder. Have the client journal feelings about enjoyable activities they will miss if they do not use their legs. Ask the client to describe the bullying incident.

Evaluate the client's risk for suicide. Explanation: In this scenario, the nurse must fully investigate any negative thoughts, feelings, or behaviors that place the client at risk of suicide. The statement, "I won't need them anyway," could indicate that the client has suicidal ideations and has decided that using the limbs is unnecessary since they will be deceased. Journaling and talk therapy that addresses the bullying incident are all important nursing care plan items but come below safety and security in order of priority. Clients with conversion disorder have difficulty understanding that their symptoms are part of a coping/disease process and often take on the attitude of "la belle indifférence."

atopic dermatitis

Excess inflammation; dry skin, redness, and itching from allergies and irritants.

secondary polycythemia vera

Excessive production of erythropoietin from reduced amounts of oxygen, cyanotic heart disease, nonpathological conditions or neoplasms •Seen in Smoking, COPD, high altitude, Pts taking Testosterone treatments •Treatment not needed for secondary if condition is mild •Treat underlying cause •Therapeutic phlebotomy

Bell's palsy is a paralysis of which of the following cranial nerves? Optic Trigeminal Otic Facial

Facial Explanation: Bell's palsy (facial paralysis) is caused by unilateral inflammation of the seventh cranial nerve, which results in weakness or paralysis of the facial muscles on the ipsilateral, or same side, of the affected facial nerve. Trigeminal neuralgia is a paralysis of the trigeminal nerve (cranial nerve V). The optic nerve (cranial nerve II) functions in vision. The vestibulocochlear nerve (cranial nerve VIII) functions in hearing.

The nurse is performing an initial assessment on a client with suspected Bell's palsy. Which of the following findings would the nurse be most focused on related to this medical diagnosis? Hyporeflexia and weakness of the lower extremities Fatigue and depression Facial distortion and pain Ptosis and diplopia

Facial distortion and pain Explanation: Bell's palsy is manifested by facial distortion, increased tearing, and painful sensations in the face, behind the ear, and in the eye. Ptosis and diplopia are associated with myasthenia gravis. Hyporeflexia and weakness of the lower extremities are associated with Guillain-Barre syndrome. Fatigue and depression are associated with multiple sclerosis.

While performing an initial nursing assessment on a client admitted with suspected tic douloureux (trigeminal neuralgia), for which of the following would the nurse expect to observe? Hyporeflexia and weakness of the lower extremities Fatigue and depression Ptosis and diplopia Facial pain in the areas of the fifth cranial nerve

Facial pain in the areas of the fifth cranial nerve Explanation: Tic douloureux (trigeminal neuralgia) is manifested by pain in the areas of the fifth (trigeminal) cranial nerve. Ptosis and diplopia are associated with myasthenia gravis. Hyporeflexia and weakness of the lower extremities are associated with Guillain-Barre syndrome. Fatigue and depression are associated with multiple sclerosis.

a nurse is assessing a client who has systemic lupus erythematosus (SLE). which of the following findings should the nurse expect? wrinkles in the skin constipation iritis facial rash

Facial rash SLE affects the skin. A facial "butterfly" rash that is dry, scaly, red, and raised is a manifestation of SLE.

Nurses' Notes Day 1: Client admitted to the medical-surgical unit from the emergency department (ED). Client came to the ED after sudden onset of dizziness, numbness and weakness of right arm, right leg, and right side of the face. Client is awake, responsive, and follows commands. Appears confused and is unable to form words to answer questions Right facial droop noted. Right hand grasp weak, left hand grasp strong. Day 7: Client is awake, alert, and oriented. Able to form some words to answer questions Right facial droop. Right hand grasp weak, left hand grasp strong. Right leg weak. Ambulates with a walker and assistance. Vital Signs Day 1: Temperature 37.5° C (99.5° F) Blood pressure 198/96 mm Hg Heart rate 112/min Respiratory rate 22/min Oxygen saturation 96% on room air Day 7: Temperature 38° C (100.4° F) Blood pressure 166/70 mm Hg Heart rate 88/min Respiratory rate 20/min Oxygen saturation 97% on room air For each client finding, click to specify if the finding is consistent with Parkinson's disease, stroke, and/or multiple sclerosis. Each finding can support more than one disease process. Facial symmetry Mobility status Blood pressure Speech Cognitive function

Facial symmetry is consistent with stroke. Unilateral facial droop can occur in a stroke due to a decrease in cerebral perfusion to one hemisphere of the brain. Rigid facial muscles is consistent with Parkinson's disease. Hypertension is consistent with stroke. Hypertension is a risk factor for the development of a stroke. Orthostatic hypotension can occur in Parkinson's disease due to a decrease in sympathetic nervous system response. Cognitive function is consistent with Parkinson's disease, stroke, and multiple sclerosis. Cognitive impairment can occur in Parkinson's disease and multiple sclerosis due to degeneration of neural pathways. Cognitive impairment is consistent with a stroke due to an interruption in cerebral perfusion. Speech is consistent with Parkinson's disease, stroke, and multiple sclerosis. Slurred speech can occur in Parkinson's disease and multiple sclerosis due to degeneration of neural pathways. Aphasia is consistent with a stroke due to an interruption in cerebral perfusion. Mobility status is consistent with Parkinson's disease, stroke, and multiple sclerosis. Impaired mobility can occur in Parkinson's disease and multiple sclerosis due to degeneration of neural pathways. Impaired mobility is consistent with a stroke due to an interruption in cerebral perfusion.

Assessment of a client reveals that the client has been inflicting illness on her daughter to gain the attention of medical personnel. The nurse identifies this as which of the following? Factitious disorder imposed on another Factitious disorder Conversion disorder Functional neurologic symptom disorder

Factitious disorder imposed on another Explanation: Factitious disorder imposed on another (previously factitious disorder by proxy or Münchausen's by proxy), involves a person who inflicts injury on another person. It is commonly a mother, who inflicts injuries on her child to gain the attention of the health care provider through her child's injuries. Conversion disorder or functional neurologic symptom disorder is a psychiatric condition in which severe emotional distress or unconscious conflict is expressed through physical symptoms.

Is the following statement true or false? An antigen is a protein substance developed by the body in response to and interacting with a specific antibody.

False An antibody, not an antigen, is a protein substance developed by the body in response to and interacting with a specific antigen, not antibody.

Which of the following types of therapy is helpful for patients with schizophrenia?

Family therapy Educating the family on recognizing schizophrenic symptoms, as well as signs of drug toxicity are very important. Goals of family therapy are to develop trust among the patient and family, and allow the family to participate in support groups, if needed.

a nurse is assessing a client for early manifestations of rheumatoid arthritis (RA). which of the following changes is an early manifestation of RA? morning stiffness fatigue temporomandibular joint pain baker's cysts

Fatigue Fatigue, weakness, and anorexia are early manifestations of RA.

A client has recently had a change in job position and a nurse asks the client how they like the new job. The client states, "Oh everything is great. I can really see myself going far in this new position." However, the client's voice was monotone and their face was nearly absent of affective expression. How does the nurse describe the client's facial expression? Inappropriate Flat Constricted Blunted

Flat Explanation: Several terms are used to describe affect, including flat, which is the absent or nearly absent affective expression; inappropriate, which is the discordant affective expression accompanying the content of speech or ideation; blunted, which is significantly reduced intensity of emotional expression; and restricted or constricted, which is mildly reduced in the range and intensity of emotional expression.

Which cerebral lobes is the largest and controls abstract thought? Occipital Frontal Temporal Parietal

Frontal Explanation: The frontal lobe also controls information storage or memory and motor function. The temporal lobe contains the auditory receptive area. The parietal lobe contains the primary sensory cortex, which analyzes sensory information and relays interpretation to the thalamus and other cortical areas. The occipital lobe is responsible for visual interpretation.

A patient sustained a head injury during a fall and has changes in personality and affect. What part of the brain does the nurse recognize has been affected in this injury? Parietal lobe Temporal lobe Frontal lobe Occipital lobe

Frontal lobe Explanation: The frontal lobe, the largest lobe, located in the front of the brain. The major functions of this lobe are concentration, abstract thought, information storage or memory, and motor function. It contains Broca's area, which is located in the left hemisphere and is critical for motor control of speech. The frontal lobe is also responsible in large part for a person's affect, judgment, personality, and inhibitions (Hickey, 2009).

A client with Guillain-Barre syndrome cannot swallow and has a paralytic ileus; the nurse is administering parenteral nutrition intravenously. The nurse is careful to assess which of the following related to intake of nutrients? Condition of skin Gag reflex and bowel sounds Urinary output and capillary refill Respiratory status

Gag reflex and bowel sounds Explanation: Paralytic ileus may result from insufficient parasympathetic activity. The nurse may administer parenteral nutrition and IV fluids. The nurse carefully assesses for the return of the gag reflex and bowel sounds before resuming oral nutrition. The other three choices are important assessment items, but not necessarily related to the intake of nutrients.

A client on the unit suddenly cries out in fear. The nurse notices that the client's head is twisted to one side, the client's back is arched, and the client's eyes have rolled back in the sockets. The client has recently begun drug therapy with haloperidol. Based on this assessment, which would be the first action of the nurse? Give a PRN dose of benztropine IM Place an urgent call to the client's physician Get a stat order for a serum drug level Hold the client's medication until the symptoms subside

Give a PRN dose of benztropine IM Explanation: The client is having an acute dystonic reaction; the treatment is anticholinergic medication. Dystonia is most likely to occur in the first week of treatment, in clients younger than 40 years, in males, and in those receiving high-potency drugs such as haloperidol. Immediate treatment with anticholinergic drugs usually brings rapid relief.

A schizophrenic patient presents to your clinic of complaining of a strange banana taste in his mouth despite not eating anything banana flavored in over a week. Which of the following best describes this type of hallucination?

Gustatory The gustatory hallucination is characterized by experiencing tastes that are not associated with the food that is being ingested. This can lead to patients refusing to eat.

hemophilia B

H B (christmas) - genetic defect that causes deficient or defective factor IX. X Linked.

hemophilia A

HA is caused by a genetic defect that results in deficient or defective factor VIII.

HIV Pathophysiology

HIV is a Retrovirus - carries genetic information in RNA instead of DNA HIV attaches to CD4+ (T helpers) HIV RNA enters the cells and converts its RNA to DNA in the cell, therefore the body's cells begin producing the virus This process kills the CD4+ cells and releases more HIV

HIV: modes of transmission

HIV-1 transmitted in body fluids that contain infected cells: -Blood and blood products -Seminal fluid -Vaginal secretions -Mother-to-child: Amniotic fluid, breast milk -Not through casual contact

During an admission assessment, a client with schizoaffective disorder states that the client hears the voice of God in the client's head and the voice is telling the client that the client is worthless. What would the nurse document this symptom as? Avolition Hallucination Alogia Delusion

Hallucination Explanation: Hallucinations are sensory perceptions with a compelling sense of reality but with no actual objective basis. During auditory hallucinations (the most common form), clients may hear the voice of God or close relatives, two or more voices with a running commentary about the client's behavior, or voices that command certain acts. Delusions are false, fixed beliefs. Avolition involves the withdrawal and inability to initiate and persist in goal-directed activity. Alogia refers to the reduced fluency and productivity of thought and speech.

when talking about patients with hematologic disorders, first thing we talk about with them is:

Health History: - family history (back 3 generations) (hx of clotting disorders, anemia, leukemias) - nutrition (low iron, low folic acid, low B12) - medication (certain medications can affect absorption)

assessment for allergic rhinitis

Health history includes personal and family history Allergy assessment Subjective data include symptoms and how the patient feels before symptoms become obvious Note relationship between symptoms and seasonal changes, emotional problems, or stress Identify nature of antigens, seasonal changes in symptoms, and medication history

assessment for rhematic disease

Health history: oInclude onset of and evolution of symptoms oFamily history oPast health history oContributing factors oPrevious treatments and their effectiveness oPatient's support systems Physical examination Functional assessment oCombination of history and observation oGait, posture, general musculoskeletal size and structure oGross deformities and abnormalities in movement oSymmetry, size, and contour of other connective tissues, such as the skin and adipose tissue

Which term describes the percentage of blood volume that consists of erythrocytes? Erythrocyte sedimentation rate (ESR) Differentiation Hematocrit Hemoglobin

Hematocrit Explanation: Hematocrit is the percentage of blood volume consisting of erythrocytes. Differentiation is the development of functions and characteristics that differ from those of the parent stem cell. ESR is a laboratory test that measures the rate of settling of red blood cells (RBCs); an elevated rate is indicative of inflammation. Hemoglobin is the iron-containing protein of RBCs.

A nurse is caring for a patient who has had a bone marrow aspiration with biopsy. What complication should the nurse be aware of and monitor the patient for? Shock Splintering of bone fragments Blood transfusion reaction Hemorrhage

Hemorrhage Explanation: Hazards of either bone marrow aspiration or biopsy include bleeding and infection. The risk of bleeding is somewhat increased if the patient's platelet count is low or if the patient has been taking a medication (e.g., aspirin) that alters platelet function.

a nurse in an emergency department is caring for a client who had a seizure and became unresponsive after stating she had a sudden, severe headache and vomiting. the client's vital signs are as follows: blood pressure of 198 / 110 mm Hg, pulse of 82/min, respirations of 24/min, and a temperature of 38.2 C (100.8 F). Which of the following neurologic disorders should the nurse suspect? transient ischemic attack (TIA) hemorrhagic stroke thrombotic stroke embolic stroke

Hemorrhagic stroke A client who has a hemorrhagic stroke often experiences a sudden onset of symptoms including sudden onset of a severe headache, a decrease in the level of consciousness, and seizures. Hemorrhagic strokes occur when bleeding occurs in the brain caused by the rupture of an aneurysm or arteriovenous malformation, hypertension and atherosclerosis, or trauma.

Which is the most common cause of acute encephalitis in the United States? St. Louis virus West Nile virus Herpes simplex virus Western equine virus

Herpes simplex virus Explanation: Viral infection is the most common cause of encephalitis. Herpes simplex virus is the most common cause of acute encephalitis in the United States. The Western equine encephalitis virus, West Nile virus, and St. Louis virus are types of arboviral encephalitis that occur in North America, but they are not the most common causes of acute encephalitis.

A 52-year-old married man with two adolescent children is beginning rehabilitation following a motor vehicle accident. The nurse planning the client's care. Who will the client's condition affect? His wife and any children that still live at home Himself Him and his entire family No one, provided he has a complete recovery

Him and his entire family Explanation: Clients and families who suddenly experience a physically disabling event or the onset of a chronic illness are the ones who face several psychosocial adjustments, even if the client recovers completely.

A client is scheduled for surgery to remove an abdominal mass. The nurse knows that which reason hemodilution would be contraindicated as a method to provide blood to the client during the surgery? Takes medications for seasonal allergies Previous thyroidectomy Treatment for osteoarthritis History of renal disease

History of renal disease Explanation: Hemodilution is the removal of 1 to 2 units of blood after induction of anesthesia and replaced with a colloid or crystalloid solution. The blood is then reinfused after the surgery. The purpose of this approach is to reduce the amount of erythrocytes lost during the surgery because the intravenous fluids dilute the concentration of red blood cells and lowers the hematocrit. Hemodilution has been linked to tissue ischemia in the kidneys and would be contraindicated in the client with a history of renal disease. Hemodilution would not be contraindicated for a previous thyroidectomy, treatment for osteoarthritis, or medication used to treat seasonal allergies.

A nurse is assessing a client for potential problems related to function and mobility. Which of the following would alert the nurse to identify a potential problem related to function or movement? Uses the handrail on one side to go down the stairs Keeps the head erect while combing the hair Lifts one leg by raising it off the ground Holds onto the furniture when walking in the house

Holds onto the furniture when walking in the house Explanation: Holding onto the furniture or other objects in the room when ambulating suggests difficulty with movement. Using both hands on a handrail while going down stairs, lifting one leg by using the other leg as support, or tilting the head to reach the back of the side while combing would suggest problems with function and mobility.

HIV/AIDS: resources in community

Home health care nursing - Monitor adherence to therapeutic regimen - Complex wound care - Respiratory care Community programs that help with transportation, shopping, legal and financial assistance Hospice nursing during terminal stages - Provides physical and emotional support

Clients diagnosed with schizophrenia may experience disordered water balance that may lead to water intoxication. Which may occur as a result of water intoxication? Weight loss Oliguria Hypernatremia Hyponatremia

Hyponatremia Explanation: Hyponatremia is a life-threatening complication of unknown cause. When a client ingests an unusually large volume of water, the kidneys' capacity to excrete water is overwhelmed, and serum sodium concentrations rapidly fall below the normal range.

for ITP, give

IVIG

What type of immunoglobulin does the nurse recognize that promotes the release of vasoactive chemicals such as histamine when a client is having an allergic reaction? IgE IgA IgM IgG

IgE Explanation: IgE promotes the release of vasoactive chemicals such as histamine and bradykinin in allergic, hypersensitivity, and inflammatory reactions. IgG neutralizes bacterial toxins and accelerates phagocytosis. IgA interferes with the entry of pathogens through exposed structures or pathways. IgM agglutinates antigens and lyses cell walls.

At 39 weeks' gestation, a pregnant client visits the physician for a scheduled prenatal checkup. The physician determines that the fetus has developed an infection in utero and sends the client for an emergency cesarean delivery. The client is very concerned about the health of her unborn child. Based on knowledge of the immune system, the delivery room nurse explains about which immunoglobulin that will be increased in the fetus at the time of birth and will be actively fighting the infection? IgG IgD IgM IgA

IgG Explanation: IgG composes 75% of total immunoglobulin. It appears in serum and tissues, assumes a major role in bloodborne and tissue infections, and crosses the placenta.

management of ischemic stroke

Immediate: head CT and then initiate thrombolytic care is the big thing - evaluate and give TPA within 4 hours -specific: determine cause so straight to CT, thrombolytics (LKW, recent surgery, trauma recent), elevate HOB, maintain airway, hemodynamic monitoring, neuro monitoring, monitor for muscle and swallowing issues -general: blood pressure control, adequate O2, avoid hyperglycemia, avoid aspiration, DVT prophylaxis, hydration, seizure control, monitor neuro status and swallowing Long term: -risk factor control -ASA; antiplatelet therapy -heparin/warfarin -surgery -rehab - focus on function, selfcare, and coping

The nurse notes that an older adult was treated for a wound infection and pneumonia within the last 6 months. Which factor will the nurse attribute to this client's illnesses? Decline in self-care activities Polypharmacy Immunosenescence Reduced vitamin intake

Immunosenescence Explanation: Immunosenescence is the term for age-related changes in the immune system. These changes have been linked to the increased rates of illness and mortality in older adults, and an increased incidence of infections. There is no evidence that polypharmacy has caused an increase in infections in the older adult. The development of infections is not directly linked to vitamin intake or self-care activities.

which of the following is best considered a cognitive symptom in patients with schizophrenia?

Impaired judgement and memory Patients show impaired judgement and have the inability to plan and organize actions. The ability to inhibit undesirable responses, moral judgement capacity, and inability to persist in a task are other characteristics. Also, there is memory impairment, with trouble related with recalling in a coherent and significant way events that have occurred.

When describing the major difference between somatic symptom disorder and factitious disorders, which would the nurse include? In somatic symptom disorder, clients are not consciously aware that needs are being met through physical complaints. In factitious disorders, clients are unaware that their symptoms are not real. Factitious disorders respond much more readily to psychopharmacologic treatment than does somatic symptom disorder. In somatic symptom disorder, clients consciously seek attention.

In somatic symptom disorder, clients are not consciously aware that needs are being met through physical complaints. Explanation: Clients with somatic symptom disorder do not intentionally cause, and have no conscious or voluntary control over, their symptoms. Lack of voluntary control is in contrast to factitious disorder and malingering. In factitious disorder, clients deliberately make up or inflict symptoms.

PHQ-15 questions

In the past 4 weeks, how much have you been bothered by any of the following problems? - stomach pain - back pain - pain in arms, legs, or joints - menstrual cramps - headaches - chest pain - dizziness - fainting spells - feeling your heart pound or race - SOB - pain during sex - constipation, loose bowels, diarrhea - nausea, gas, indigestion - feeling tired or low energy - trouble sleeping

The nurse is caring for a client with Guillain-Barré syndrome. Which assessment finding would indicate the need for oral suctioning? Increased pulse rate, adventitious breath sounds Decreased pulse rate, abdominal breathing Decreased pulse rate, respirations of 20 breaths/minute Increased pulse rate, respirations of 16 breaths/minute

Increased pulse rate, adventitious breath sounds Explanation: An increased pulse rate above baseline with adventitious breath sounds indicate compromised respirations and signal a need for airway clearance. A decrease in pulse rate is not indicative of airway obstruction. An increase of pulse rate with slight elevation of respirations (16 breaths/minute) is not significant for suctioning unless findings suggest otherwise.

cranial arteritis

Inflammation and damage to blood vessels supplying blood to the head and neck. Also called giant cell arteritis. common cause of HA in geriatric ppl. temporal artery headache.

A patient who has long-term packed RBC (PRBC) transfusions has developed symptoms of iron toxicity that affect liver function. What immediate treatment should the nurse anticipate preparing the patient for that can help prevent organ damage? Iron chelation therapy Therapeutic phlebotomy Anticoagulation therapy Oxygen therapy

Iron chelation therapy Explanation: Iron overload is a complication unique to people who have had long-term PRBC transfusions. One unit of PRBCs contains 250 mg of iron. Patients with chronic transfusion requirements can quickly acquire more iron than they can use, leading to iron overload. Over time, the excess iron deposits in body tissues and can cause organ damage, particularly in the liver, heart, testes, and pancreas. Promptly initiating a program of iron chelation therapy can prevent end-organ damage from iron toxicity.

a nurse is providing teaching to a client who has a new diagnosis of Parkinson's disease. on which of the following medications should the nurse prepare to instruct the client? Piperacillin/tazobactam levothyroxine levodopa/carbidopa carbamazepine

Levodopa/carbidopa Levodopa/carbidopa is the cornerstone of Parkinson's treatment. The nurse should prepare to instruct the client on the use of this medication.

a nurse is planning care for a client who has an absolute neutrophil count (ANC) of less than 1000. which of the following interventions should the nurse include in the plan? take the client's rectal temperature each day increase raw produce in the client's diet limit visitors to healthy adults instruct the client to floss his teeth daily

Limit visitors to healthy adults. MY ANSWER The expected reference range of absolute neutrophil count is 2500 to 8000/mm3. This client has a reduced absolute neutrophil count (neutropenia) and is immunosuppressed. A client who has neutropenia is at an increased risk for infection. The nurse should restrict visitors for a client who has neutropenia to healthy adults to reduce the risk for infection.

Which term refers to a form of white blood cell involved in immune response? Spherocyte Lymphocyte Thrombocyte Granulocyte

Lymphocyte Explanation: Both B and T lymphocytes respond to exposure to antigens. Granulocytes include basophils, neutrophils, and eosinophils. A spherocyte is a red blood cell without central pallor, seen with hemolysis. A thrombocyte is a platelet.

A client is diagnosed with human immunodeficiency virus (HIV). After recovering from the initial shock of the diagnosis, the client expresses a desire to learn as much as possible about HIV and acquired immunodeficiency syndrome (AIDS). When teaching the client about the immune system, the nurse states that humoral immunity is provided by which type of white blood cell? Lymphocyte Basophil Monocyte Neutrophil

Lymphocyte Explanation: The lymphocyte provides humoral immunity — recognition of a foreign antigen and formation of memory cells against the antigen. Humoral immunity is mediated by B and T lymphocytes and can be acquired actively or passively. The neutrophil is crucial to phagocytosis. The basophil plays an important role in the release of inflammatory mediators. The monocyte functions in phagocytosis and monokine production

A patient has enlarged lymph nodes in his neck and a sore throat. This inflammatory response is an example of a cellular immune response whereby: Antibodies are released into the bloodstream Lymphocytes migrate to areas of the lymph node B-lymphocytes respond to a specific antigen Antibodies reside in the plasma

Lymphocytes migrate to areas of the lymph node Explanation: Recognition of antigens as foreign, or non-self, by the immune system is the initiating event in any immune response. Recognition involves the use of lymph nodes and lymphocytes for surveillance. Lymph nodes are widely distributed internally throughout the body and in the circulating blood, as well as externally near the body's surfaces. They continuously discharge small lymphocytes into the bloodstream. These lymphocytes patrol the tissues and vessels that drain the areas served by that node.

The nurse is aware that the phagocytic immune response, one of the body's responses to invasion, involves the ability of cells to ingest foreign particles. Which of the following engulfs and destroys invading agents? Macrophages Basophils Neutrophils Eosinophils

Macrophages Explanation: Macrophages move toward the antigen and destroy it. Eosinophils are only slightly phagocytic.

An 83-year-old woman suffers a stroke at home and is hospitalized for treatment and management. Which of the following diagnostic procedures would be best to visualize the extent of damage? Magnetic resonance angiography (MRA) Magnetic resonance imaging (MRI) Computed tomography (CT) Diffusion-weighted imaging (DWI)

Magnetic resonance angiography (MRA) Explanation: An MRA allows separate visualization of the cerebral vasculature and can be used in place of an MRI.

a nurse is caring for a client who is experiencing Cushing's Triad following a subdural hematoma. Which of the following medications should the nurse plan to administer? albumin 25% dextran 70 hydroxyethl glucose mannitol 25%

Mannitol 25% Cushing's Triad is an indication that the client is experiencing increased intracranial pressure. The nurse should administer mannitol 25%, an osmotic diuretic that promotes diuresis to treat cerebral edema.

lifestyle complications of allergic rhinitis

May affect the quality of life, sleep disturbance, impairment of daily activities, and missed school and work

A client has been diagnosed with major depression. The client reports of waking up during the night and has trouble returning to sleep. A nurse interprets this finding as suggesting what? Hypersomnia Initial insomnia Middle insomnia Terminal insomnia

Middle insomnia Explanation: The most common sleep disturbance associated with major depression is insomnia, which is categorized according to three categories: initial insomnia (difficulty falling asleep), middle insomnia (waking up during the night and having difficulty returning to sleep), and terminal insomnia (waking too early and being unable to return to sleep). Less frequently, the sleep disturbance is hypersomnia (prolonged sleep episodes at night or increased daytime sleep).

The most severe form of factitious disorder includes which of the following? Malingering Alexithymia Munchausen's syndrome Hypochondriasis

Munchausen's syndrome Explanation: In 1951, the term Munchausen's syndrome was used to describe the most severe form of factitious disorder, which was characterized by fabricating a physical illness, having recurrent hospitalizations, and going from one medical provider to another. Malingering occurs when an individual intentionally produces illness symptoms that are motivated by another specific self-serving goal, such as being classified as disabled or avoiding work. Alexithymia and hypochondriasis are not factitious disorders.

A mother rushes her infant to the emergency department and states "Help, my baby is unresponsive!" Which term is applicable when a person inflicts illness or injury on someone else to gain the attention of emergency medical personnel or to be a "hero" for saving the victim? Factitious disorder Malingering Munchausen's syndrome by proxy Induced illness

Munchausen's syndrome by proxy Explanation: A variation of factitious disorder, imposed on others, is commonly called Munchausen's syndrome by proxy and occurs when a person inflicts illness or injury on someone else to gain the attention of emergency medical personnel or to be a "hero" for saving the victim. Malingering is the intentional production of false or grossly exaggerated physical or psychological symptoms; it is motivated by external incentives such as avoiding work, evading criminal prosecution, obtaining financial compensation, or obtaining drugs. Factitious disorder, imposed on self, occurs when a person intentionally produces or feigns physical or psychological symptoms solely to gain attention. Induced illness is another name for factitious disorder.

The nurse is performing an initial nursing assessment on a client with possible Guillain-Barre syndrome. Which of the following findings would be most consistent with this diagnosis? Muscle weakness and hyporeflexia of the lower extremities Fever and cough Hyporeflexia and skin rash Ptosis and muscle weakness of upper extremities

Muscle weakness and hyporeflexia of the lower extremities Explanation: Guillain-Barre syndrome typically begins with muscle weakness and diminished reflexes of the lower extremities. Fever, skin rash, cough, and ptosis are not signs/symptoms associated with Guillain-Barre.

Which type of cell is capable of recognizing and killing infected or stressed cells and producing cytokines? Memory cells Null lymphocytes Cytotoxic T cells Natural killer cells

Natural killer cells Explanation: Natural killer cells are a class of lymphocytes that recognize infected and stressed cells and respond by killing these cells and by secreting macrophage-activating cytokine. Natural killer cells defend against microorganisms and some types of malignant cells. Null lymphocytes do not produce or active cytokine they are stimulated to attack tumor or viral infected cells. Memory cells are important to the immune system but produces memory B cells not cytokine. Cytotoxic -T cells leads to apoptosis ,causing death in infectious cells, however they do not produce cytokine.

An adult client has had mumps when the client was a child. The client had a titer prior to entering nursing school and shows immunity. What type of immunity does this reflect? Natural passive immunity Passive immunity Artificially acquired active immunity Naturally acquired active immunity

Naturally acquired active immunity Explanation: Naturally acquired active immunity occurs as a direct result of infection by a specific microorganism. An example is the immunity to measles that develops after the initial infection. Not all invading microorganisms produce a response that gives lifelong immunity. Artificially acquired immunity is obtained by receiving a killed or weakened microorganism or toxoid. Passive immunity is acquired when ready-made antibodies are given to a susceptible person.

Which type of immunity becomes active as a result of infection by a specific microorganism? Artificially acquired active immunity Artificially acquired passive immunity Naturally acquired passive immunity Naturally acquired active immunity

Naturally acquired active immunity Explanation: Naturally acquired active immunity occurs as a result of an infection by a specific microorganism. Artificially acquired active immunity results from the administration of a killed or weakened microorganism or toxoid. Passive immunity develops when ready-made antibodies are given to a susceptible individual.

a nurse in an emergency department is assessing a client who has been taking haloperidol for 3 months. the client has a temperature of 39.5 C (103.4 F), blood pressure of 150/110mmHg, and muscle rigidity. Which of the following complications should the nurse suspect? agranulocytosis neuroleptic malignant syndrome akathisia tardive dyskinesia

Neuroleptic malignant syndrome Neuroleptic malignant syndrome (NMS) is a rare and potentially fatal adverse effect of antipsychotic medications that requires emergency medical intervention. Manifestations of NMS are sudden and include changes in level of consciousness, seizures, and stupor.

A client with schizophrenia is receiving antipsychotic therapy. The nurse understands that which is a medical emergency should it develop in the client? Akathisia Tardive dyskinesia Parkinsonism Neuroleptic malignant syndrome

Neuroleptic malignant syndrome Explanation: Although tardive dyskinesia, parkinsonism, and akathisia can occur with antipsychotic therapy, neuroleptic malignant syndrome is a life-threatening condition and medical emergency that requires immediate treatment.

The body responds to infection by increasing the production of white blood cells (WBCs). The nurse should evaluate the differential count for what type of WBCs, which are the first WBCs to respond to an inflammatory event? Neutrophils Eosinophils Monocytes Basophils

Neutrophils Explanation: Neutrophils, the most abundant type of white blood cell, are the first of the WBCs to respond to infection or inflammation. The normal value is 3,000 to 7,000/cmm (males) and 1,800 to 7,700/cmm (females).

Hallucinations are defined by which condition?

No external stimulus Hallucinations are false sensory experiences that are tied to the five senses. There is no physiological or real stimulus to cause the sensory experience.

The nurse obtains a psychosocial history from a client who may have psychological factors affecting the medical condition. Which should the nurse recognize as pertinent to this diagnosis? No physiologic cause has been found for the client's symptoms. The client is able to articulate the cause of psychological distress. The client's symptoms are related to conscious motives. The client's symptoms subside with appropriate medical treatment.

No physiologic cause has been found for the client's symptoms. Explanation: One of the important factors regarding the diagnosis of psychological factors affecting medical conditions is that there is no physiologic basis for these symptoms. They are often based on psychological conditions. These symptoms do not usually subside easily, clients cannot often talk about the cause of their distress, and the symptoms are always related to unconscious processes.

nursing management of PIDD

Nursing care meticulos and reduce risk of infection - hand hygiene - infection prevention - continual monitoring for early signs of infection - teach patients and caregivers to administer medications and therapy at home - provide ongoing education and support

a nurse is implementing a plan of care for a client who has AIDS with recurring pneumonia. which of the following actions should the nurse? encourage fluid intake of 1500 ml/day position head of bed at 10 degrees cough and deep breath every 8 hours obtain a sputum culture

Obtain a sputum culture. MY ANSWER The nurse should obtain a sputum culture to determine which antibiotic is needed for the organism that is causing the pneumonia.

A client is diagnosed with a brain tumor. The nurse's assessment reveals that the client has difficulty interpreting visual stimuli. Based on these findings, the nurse suspects injury to which lobe of the brain? Parietal Frontal Occipital Temporal

Occipital Explanation: The occipital lobe is responsible for interpreting visual stimuli. The frontal lobe influences personality, judgment, abstract reasoning, social behavior, language expression, and movement. The temporal lobe controls hearing, language comprehension, and storage and memory recall (although memory recall is also stored throughout the brain). The parietal lobe interprets and integrates sensations, including pain, temperature, and touch; it also interprets size, shape, distance, and texture.

A nurse provides care to a client with schizoaffective disorder during hospitalization for acute psychosis. Nursing interventions to help the client to establish trust with the health care team is best accomplished by what? Encouraging the client to participate in group therapy daily Decrease stressful situations by controlling the client's symptoms Offering reassurance in a soft, nonthreatening voice Reminding the client that delusions are not real

Offering reassurance in a soft, nonthreatening voice Explanation: During periods of acute psychosis, offering reassurance in a soft, nonthreatening voice and avoiding confrontational stances will help the client begin to trust the staff and nursing care.

A schizophrenic patient presents to your clinic stating he smells his deceased mother's blueberry pie at night before bed. Which of the following best describes this type of hallucination?

Olfactory When a client smells an odor that is not present

A nurse is performing passive range of motion to a client's upper extremities. The nurse touches the client's thumb to each fingertip on the same hand. The nurse is performing which of the following? Dorsiflexion Pronation Adduction Opposition

Opposition Explanation: Opposition involves touching the thumb to each fingertip on the same hand. Adduction would involve moving the arm away from the midline of the body. Pronation involves rotating the forearm so that the palm of the hand is down. Dorsiflexion involves movement that flexes or bends the hand back toward the body.

The most common cause of cholinergic crisis includes which of the following? Overmedication Undermedication Compliance with medication Infection

Overmedication Explanation: A cholinergic crisis, which is essentially a problem of overmedication, results in severe generalized muscle weakness, respiratory impairment, and excessive pulmonary secretion that may result in respiratory failure. Myasthenic crisis is a sudden, temporary exacerbation of MG symptoms. A common precipitating event for myasthenic crisis is infection. It can result from undermedication.

A client involved in a motor vehicle accident arrives at the emergency department unconscious and severely hypotensive. The nurse suspects the client has several fractures in the pelvis and legs. Which parenteral fluid is the best choice for the client's current condition? Normal saline solution Packed red blood cells (RBCs) Lactated Ringer's solution Fresh frozen plasma

Packed red blood cells (RBCs) Explanation: In a trauma situation, the first blood product given is unmatched (O negative) packed RBCs. Fresh frozen plasma is commonly used to replace clotting factors. Normal saline or lactated Ringer's solution is used to increase volume and blood pressure; however, too much colloid will hemodilute the blood and doesn't improve oxygen-carrying capacity as well as packed RBCs do.

The pre-nursing class is learning about the nervous system in their anatomy class. What part of the nervous system would the students learn is responsible for digesting food and eliminating body waste? Central Peripheral Parasympathetic Sympathetic

Parasympathetic Explanation: The parasympathetic division of the autonomic nervous system works to conserve body energy and is partly responsible for slowing heart rate, digesting food, and eliminating body wastes.

When describing the role of the various members of the rehabilitation team, which member would the nurse identify as the one who determines the final outcome of the process? Patient Nurse Physician Physical therapist

Patient Explanation: Although the nurse, physician, and physical therapist play important roles in the rehabilitation process, the patient is a key member of the rehabilitation team, the focus of the team's efforts, and the one who determines the final outcomes of the process.

Which is a symptom of severe thrombocytopenia? Dyspnea Inflammation of the mouth Petechiae Inflammation of the tongue

Petechiae Explanation: Clients with severe thrombocytopenia have petechiae, which are pinpoint hemorrhagic lesions, usually more prominent on the trunk or anterior aspects of the lower extremities.

A client is admitted with cellulitis and experiences a consequent increase in white blood cell count. During what process will pathogens be engulfed by white blood cells that ingest foreign particles? Cellular immune response Apoptosis Antibody response Phagocytosis

Phagocytosis Explanation: During the first mechanism of defense, white blood cells, which have the ability to ingest foreign particles, move to the point of attack, where they engulf and destroy the invading agents. This is known as phagocytosis. The action described is not apoptosis (programmed cell death) or an antibody response. Phagocytosis occurs in the context of the cellular immune response, but it does not constitute the entire cellular response.

A client with a somatic symptom illness asks what is causing the physical symptoms. Which would be the appropriate explanation for the nurse to offer? Physical symptoms are independent of the amount of the client's psychic distress. Physical symptoms are an involuntary way of dealing with psychic conflict. Physical symptoms are deliberately expressed in order to benefit in some way. Physical symptoms can be attributed to an organic cause.

Physical symptoms are an involuntary way of dealing with psychic conflict. Explanation: The three central features of somatic symptom are as follows: physical complaints suggest major medical illness, but have no demonstrable organic basis; psychological factors and conflicts seem important in initiating, exacerbating, and maintaining the symptoms; and symptoms or magnified health concerns are not under the client's conscious control. The severity of symptoms corresponds to the severity of psychic conflict.

A nurse is caring for a 20-year-old client who has a fever and reports severe headache. Vital signs: Temperature 38.9° C (102° F); Tympanic Apical pulse 118/min; strong and regular Respirations 20/min; even and unlabored Blood pressure 114/78 mm Hg Oxygen saturation 97% on room air Nurses notes: 0800: Client reports missing classes at a local community college the last two days due to fever and headache. Rates pain with headache as a 9 on a scale of 0 to 10. Verbalizes that headache was not relieved by acetaminophen or ibuprofen taken at home. Client awake, alert, and oriented to person, place, and time. Pupils equal, round, and reactive to light. Temperature elevated. Skin warm and dry, face flushed. Petechiae noted on trunk. Reports nausea and vomiting for the last 24 hr. Bowel sounds positive x 4 quadrants. Abdomen soft and nontender to light palpation. Photophobia present. Nuchal rigidity noted. Brudzinski's sign positive. Respirations easy and unlabored. Lungs clear to auscultation. 0815: Provider notified of client's current vital signs and assessment findings. New prescriptions received. Providers prescription: Prepare for lumbar puncture Draw CBC with differential X-ray: Chest and sinuses A nurse is preparing the client for a lumbar puncture. Which of the following actions should the nurse take? Select all that apply. Place the client in a lateral position with the knees drawn to the abdomen. Assess for allergies to contrast dyes. Ensure informed consent is obtained. Obtain coagulation studies. Place client NPO for 4 to 6 hr. Administer IV sedation as prescribed. Provide education about the procedure. Administer a soapsuds enema.

Place client in a lateral position with the knees drawn to the abdomen ensure informed consent is obtained. obtain coagulation studies provide education about the procedures Place the client in a lateral position with the knees drawn to the abdomen is correct. The client should be placed in a side-lying position with the knees drawn to the abdomen and the chin touching the chest. This position separates the lumbar vertebrae and allows for easier needle insertion. Obtain coagulation studies is correct. Systemic inflammation caused by meningitis can cause coagulopathy to occur. To decrease the risk of bleeding during and after the procedure, the nurse should obtain coagulation studies prior to the procedure. Ensure informed consent is obtained is correct. A lumbar puncture is an invasive procedure. The nurse should ensure that informed consent has been obtained before the procedure occurs. Provide education about the procedure is correct. To decrease the clients fears and misconceptions about the procedure, the nurse should educate the client about what to expect before, during, and after the procedure.

a nurse enters a client's room and finds the client on the floor having a seizure. which of the following actions should the nurse take? Insert a tongue blade in the client's mouth. Place the client on his side. Hold the client's arms and legs from moving. Place the client back in bed.

Place the client on his side. The nurse should place the client on his side. This position drops the tongue to the side of the client's mouth and prevents the client's airway from being obstructed.

a nurse is reviewing the lab findings for a client who has idiopathic thrombocytopenic purpura (ITP). which of the following findings should the nurse expect to be decreased? WBC RBC granulocytes platelets

Platelets MY ANSWER The nurse should recognize that ITP results from the destruction of platelets by antibodies; therefore, the nurse should expect a platelet level below the expected reference range.

Which diagnostic test is used for early diagnosis of HSV-1 encephalitis? Lumbar puncture (LP) Electroencephalography (EEG) Polymerase chain reaction (PCR) Magnetic resonance imaging (MRI)

Polymerase chain reaction (PCR) Explanation: PCR is the standard test for early diagnosis of HSV-1 encephalitis. An LP often reveals a high opening pressure and low glucose and high protein levels in CSF samples. EEG is used to diagnose seizures. An MRI is used to detect brain lesions.

A client with lower back pain is scheduled for myelography using metrizamide (a water-soluble contrast dye). After the test, the nurse should prioritize what action? A.Helping the client perform deep breathing and coughing exercises B.Limiting fluids for the next 12 hours C.Administering IV morphine sulfate to prevent headache D.Positioning the client with the head of the bed elevated 45 degrees

Positioning the client with the head of the bed elevated 45 degrees

The nurse is assessing a newly admitted client with a diagnosis of meningitis. On assessment, the nurse expects to find which of the following? Positive Kernig's sign Positive Romberg sign Hyper-alertness Negative Brudzinski's sign

Positive Kernig's sign Explanation: A positive Kernig's sign is a common finding in the client with meningitis. When the client is lying with the thigh flexed on the abdomen, the leg cannot be completely extended. A positive Brudzinski's sign is usual with meningitis. The Romberg sign would not be tested in this client. The client will develop lethargy as the illness progresses, not hyper-alertness.

BMA extra

Preparation is careful explanation of the procedure but may need to administer antianxiety medications. Aspiration is sharp brief pain

Medical management of MS

Primarily supportive, includes limitation of physical activity, Na restriction, & diuretics. - immunomodulators and steroids (mabs and mibs) - disease-modifying therapy - symptom management

What is the function of the thymus gland? Programs B lymphocytes to become regulator or effector B cells Develops the lymphatic system Produces stem cells Programs T lymphocytes to become regulator or effector T cells

Programs T lymphocytes to become regulator or effector T cells Explanation: The thymus gland is located in the neck below the thyroid gland. It extends into the thorax behind the top of the sternum. The thymus gland produces lymphocytes during fetal development. It may be the embryonic origin of other lymphoid structures such as the spleen and lymph nodes. After birth, the thymus gland programs T lymphocytes to become regulator or effector T cells. The thymus gland becomes smaller during adolescence but retains some activity throughout the life cycle. The other options are incorrect.

Which is a component of the nursing management of the client with variant Creutzfeldt-Jakob disease (vCJD)? Administering amphotericin B Preparing for organ donation Providing palliative care Initiating isolation procedures

Providing palliative care Explanation: vCJD is a progressive fatal disease; no treatment is available. Because of the fatal outcome of vCJD, nursing care is primarily supportive and palliative. Prevention of disease transmission is an important part of providing nursing care. Although client isolation is not necessary, use of standard precautions is important. Institutional protocols are followed for blood and body fluid exposure and decontamination of equipment. Organ donation is not an option because of the risk for disease transmission. Amphotericin B is used in the treatment of fungal encephalitis; no treatment is available for vCJD.

A client diagnosed with factitious disorder tells the nurse about how he overcame a tremendous disability. Based on the client's history, the nurse knows that the story is not all true. The client is exhibiting which of the following? Pseudologia fantastica Malingering Alexithymia Hypochondriasis

Pseudologia fantastica Explanation: With factitious disorder, clients are extremely creative in simulating illnesses, and they tell fascinating but false stories of personal triumph. These tales are referred to as pseudologia fantastica and are a core symptom of the disorder. Pseudologia fantastica are stories that are not entirely improbable and often contain a matrix of truth and falsehood. Malingering involves an individual who intentionally produces illness symptoms but is motivated by as self-serving goal such as being classified as disabled or avoiding work. Individuals with alexithymia have difficulty identifying and expressing their emotions. They have a preoccupation with external events and are described as concrete externally oriented thinkers. Hypochondriasis is seen in individuals who are fearful of developing a serious illness based on their misinterpretation of body sensations.

A client was admitted to the psychiatric unit with major depression after a suicide attempt. In addition to the client's feelings of sadness and hopelessness, the nurse would expect to assess what? Increased energy level Psychomotor retardation Increased focus Decreased complaints of pain

Psychomotor retardation Explanation: Associated signs and symptoms of depression include an inability to think or concentrate, increased complaints of pain, psychomotor retardation, and lack of energy and fatigue.

dosages and administration of TPA

Pt is weighed to determine dose Two or more IV sites established for tpa and fluids Dosage is 0.9 mg/kg w max dose of 90 mg 10% of dose is given as a bolus Pt is admitted to ICU

a nurse is caring for a client who is undergoing a lumbar puncture. which of the following is the priority action for the nurse to take to maintain privacy for the client? close the door to the client's room pull the curtains around the client's bed ask family members to leave the room use sterile drapes to cover the client

Pull the curtains around the client's bed. Pulling the curtains around the client's bed assures privacy for the client should someone open the door or enter the room.

Which of the following is the first-line therapy for myasthenia gravis (MG)? Azathioprine (Imuran) Pyridostigmine bromide (Mestinon) Lioresal (Baclofen) Deltasone (Prednisone)

Pyridostigmine bromide (Mestinon) Explanation: Mestinon, an anticholinesterase medication, is the first-line therapy in MG. It provides symptomatic relief by inhibiting the breakdown of acetylcholine and increasing the relative concentration of available acetylcholine at the neuromuscular junction. If Mestinon does not improve muscle strength and control fatigue, the next agents used are immunosuppressant agents. Imuran is an immunosuppressive agent that inhibits T lymphocytes and reduces acetylcholine receptor antibody levels. Baclofen is used in the treatment of spasticity in MG.

megaloblastic anemia

RBC are abnormally large. Other myeloid cells are also abnormal. RBC Hyperplasia (abnormal increase in the number of cells) can be: - folic acid deficiency - vitamin B12 deficiency

A client is diagnosed with multiple site cancers and has received whole-body irradiation. The nurse is concerned about a compromised immune system in this client for which reason? Radiation causes an excess of circulating hemoglobin. Radiation causes a deficiency of circulating hemoglobin. Radiation destroys lymphocytes. Radiation causes an excess of circulating lymphocytes.

Radiation destroys lymphocytes. Explanation: Radiation destroys lymphocytes and decreases the ability to mount an effective immune response. Radiation is not associated with an excess of lymphocytes or an excess or deficiency of hemoglobin.

interventions to maintain functional ability for patients with rheumatic diseases

Range of motion Isometric exercise Dynamic exercise Aerobic exercise Pool exercise

a nurse is preparing to administer an osmotic diuretic IV to a client with increased intracranial pressure. which of the following should the nurse identify as the purpose of the medication? reduce edema of the brain provide fluid hydration increase cell size in the brain expand extracellular fluid volume

Reduce edema of the brain. An osmotic diuretic is used to decrease intracranial pressure by moving fluid out of the ventricles into the bloodstream.

A client with schizophrenia reads the advice column in the newspaper daily. When asked why the client is so interested in the advice column, the client replies, "This person is my guide and tells me what I must do every day." The nurse would best describe this type of thinking as what? Thought insertion Personalization Grandiose delusion Referential delusion

Referential delusion Explanation: Referential delusions or ideas of reference involve the client's belief that television broadcasts, music, or newspaper articles have special meaning for him or her. Grandiose delusions are characterized by the client's claim to association with famous people or celebrities or the client's belief that he or she is famous or capable of great feats. Thought insertion is the belief that others are placing thoughts in their mind against their will. Personalization is not a psychotic characteristic of schizophrenia.

A client scheduled for magnetic resonance imaging (MRI) has arrived at the radiology department. The nurse who prepares the client for the MRI should prioritize what action? A.Withholding stimulants 24 to 48 hours prior to exam B.Initiating an IV line for administration of contrast C.Removing all metal-containing objects D.Instructing the client to void prior to the MRI

Removing all metal-containing objects

Which condition is associated with impaired immunity relating to the aging client? Renal function decreases Skin becomes thicker Antibody production increases Incidence of autoimmune disease decreases

Renal function decreases Explanation: Decreased renal circulation, filtration, absorption, and excretion contribute to the risk for urinary tract infections. Antibody production decreases, the skin becomes thinner, and the incidence of autoimmune disease increases with age.

When assessing a client with somatic symptom disorder, which would the nurse most likely note? Denial and repression are the chief defense mechanisms used. The client's symptoms are under the conscious control of the client. Reports of physical symptoms do not have a demonstrable organic basis to fully account for them. The client willfully controls the physical symptoms.

Reports of physical symptoms do not have a demonstrable organic basis to fully account for them. Explanation: A central feature of somatic symptom disorder is a report of physical symptoms without a demonstrable organic basis to fully account for the symptoms. Symptoms or magnified health concerns are not under the client's conscious control. Denial and repression are not chief defense mechanisms used. Clients do not willfully control the physical symptoms.

A nurse is caring for a client with an injury to the central nervous system. When caring for a client with a spinal cord insult that is slowing transmission of the motor neurons, in what would the nurse anticipate a delayed reaction? Cognitive ability to understand relayed information. Identification of information due to slowed passages of information to brain. Response due to interrupted impulses from the central nervous system Processing information transferred from the environment.

Response due to interrupted impulses from the central nervous system Explanation: The central nervous system is composed of the brain and the spinal cord. Motor neurons transmit impulses from the central nervous system. Slowing transmission in this area would slow the response of transmission leading to a delay in reaction. Sensory neurons transmit impulses from the environment to the central nervous system, allowing identification of a stimulus. Cognitive centers of the brain interpret the information.

drugs used for ALS

Riluzole Edaravone

A client who just went through an upsetting divorce is threatening to commit suicide with a handgun. The client is involuntarily admitted to the psychiatric unit. Which nursing diagnosis has priority? Risk for suicide related to highly lethal plan Spiritual distress related to conflicting thoughts about suicide and sin Ineffective coping related to inadequate stress management Hopelessness related to recent divorce

Risk for suicide related to highly lethal plan Explanation: Safety is the priority. The overall goals for the client who is suicidal is first to keep the client safe and later to help him or her develop new coping skills that do not involve self-harm. Hopelessness related to recent divorce, ineffective coping related to inadequate stress management, and spiritual distress related to conflicting thoughts about suicide and sin would not be the priority diagnosis for this client.

Some research has suggested that schizophreniform disorder may be an early manifestation of which other mental health condition? Delusional disorder Schizophrenia Schizoaffective disorder Bipolar affective disorder

Schizophrenia Explanation: Some research has suggested that schizophreniform may be an early manifestation of schizophrenia. A client exhibiting an acute reactive psychosis for less than the 6 months necessary to meet the diagnostic criteria for schizophrenia is given the diagnosis of schizophreniform disorder. Symptoms lasting beyond the 6 months warrant a diagnosis of schizophrenia.

A client with major depression is prescribed paroxetine. The nurse develops an education plan for the client based on the understanding that this drug belongs to which class of drugs? Monoamine oxidase inhibitors Selective serotonin reuptake inhibitors Tricyclic antidepressants Serotonin norepinephrine reuptake inhibitors

Selective serotonin reuptake inhibitors Explanation: Paroxetine is a selective serotonin reuptake inhibitor. Serotonin norepinephrine reuptake inhibitors include venlafaxine, nefazodone, duloxetine, and desvenlafaxine. Amitriptyline is an example of a tricyclic antidepressant. Monoamine oxidase inhibitors include phenelzine, tranylcypromine, isocarboxazid, and selegiline.

The nurse is caring for a client recovering from a major burn. Burns affect the immune system by causing a loss of large amounts of which of the following? Serum, which depletes the body's store of glucagon Plasma, which depletes the body's store of catecholamines Plasma, which depletes the body's store of calcitonin Serum, which depletes the body's store of immunoglobulins

Serum, which depletes the body's store of immunoglobulins Explanation: Major burns cause impaired skin integrity and compromise the body's first line of defense. Loss of large amounts of serum occurs with burn injuries and depletes the body of essential proteins, including immunoglobulins. Loss of serum or plasma does not deplete the body of catecholamines (adrenal gland), calcitonin (thyroid gland), or glucagon (pancreas).

Brudzinski's sign

Severe neck stiffness causes a patient's hips and knees to flex when the neck is flexed when the patient is lying down on their back.

Somatic symptom disorder is characterized by what? Self-induced disease states or faked symptoms to garner attention Severe physical symptoms unexplainable by any organic or physical pathology Self-inflicted injuries Physical symptoms coupled with extreme focus on emotional state

Severe physical symptoms unexplainable by any organic or physical pathology Explanation: The diagnostic criteria for somatic symptom disorder were updated in the DSM-5 and include one or more symptoms that cause persistent distress or significant disruption in daily lives for at least 6 months, as well as excessive thoughts about the seriousness of the symptoms, feelings (such as anxiety about the symptoms or health) or behaviors related to the symptoms, or health concerns (such as spending excessive time and energy focusing on these symptoms or health). These symptoms cannot be explained or unexplained by medical evidence.

Before a client became depressed, the client was an active, involved parent with three children, often attending their school functions and serving as a volunteer. The client is hospitalized for a major depressive episode and now reveals that the client feels like an unnecessary burden on the client's family. Which nursing diagnosis is most appropriate? Ineffective coping related to marital disagreements Ineffective activity planning related to depression Anxiety related to side effects of medication Situational low self-esteem

Situational low self-esteem Explanation: The client does not express anxiety, issues with marital disagreements, or problems with activity planning. Instead, the client has experienced a change from being an involved, interested parent to feeling as though the client is a burden, which would be reflective of a disturbance of self-esteem. The self-esteem changes the client is experiencing are related to feelings of worthlessness brought on by the depressive episode.

The nurse expects which assessment finding of the oral cavity when the client is diagnosed with pernicious anemia? Smooth tongue Ulcerations of oral mucosa Angular cheilosis Enlarged gums

Smooth tongue Explanation: On physical assessment, the nurse expects to observe a smooth tongue in the client diagnosed with pernicious anemia. Angular cheilosis (ulceration of the corners of the mouth) is seen with anemia. Ulcerations of the oral cavity indicate infection or possible leukemia. Enlarged gums can be indicative of leukemia.

A client diagnosed with delusional disorder who uses excessive health care resources most likely has which type of delusions? Nihilistic Grandiose Somatic Jealous

Somatic Explanation: Persons who have somatic delusions believe they have a physical ailment. Clients with somatic delusions use excessive health care resources. The central theme of the jealous subtype is the unfaithfulness or infidelity of a spouse or lover. Nihilistic delusions focus on death or calamity. Clients presenting with grandiose delusions are convinced they have a great, unrecognized talent or have made an important discovery; a less common presentation is the delusion of a special relationship with a prominent person or actually being a prominent person.

A nurse is assessing a client who is reporting the sensation of "bugs crawling under the skin" and intense itching and burning. The client states, "I know bugs have invaded my body." There is no evidence to support the client's report. The nurse interprets this as which type of delusion? Persecutory Nihilistic Grandiose Somatic

Somatic Explanation: Somatic delusions involve bodily functions or sensations, such as insects having infested the skin. The client vividly describes crawling, itching, burning, swarming, and jumping on the skin surface or below the skin. The client maintains the conviction that he or she is infested with parasites in the absence of objective evidence to the contrary. Nihilistic delusions focus on impending death or disaster. Clients presenting with grandiose delusions are convinced they have a great, unrecognized talent or have made an important discovery. The central theme of persecutory delusions is the client's belief that he or she is being conspired against, cheated, spied on, followed, poisoned, drugged, maliciously maligned, harassed, or obstructed in pursuit of long-term goals.

labs for hemolytic anemia

Strong correlation between lab iron stores and HgB levels. When iron depleted then HgB falls. Low iron causes small RBC. MCV decreases (measures the size of RBC).

A patient with Bell's palsy says to the nurse, "It doesn't hurt anymore to touch my face. How am I going to get muscle tone back so I don't look like this anymore?" What interventions can the nurse suggest to the patient? Suggest applying cool compresses on the face several times a day to tighten the muscles. Inform the patient that the muscle function will return as soon as the virus dissipates. Suggest massaging the face several times daily, using a gentle upward motion, to maintain muscle tone. Tell the patient to smile every 4 hours.

Suggest massaging the face several times daily, using a gentle upward motion, to maintain muscle tone. Explanation: After the sensitivity of the nerve to touch decreases and the patient can tolerate touching the face, the nurse can suggest massaging the face several times daily, using a gentle upward motion, to maintain muscle tone. Facial exercises, such as wrinkling the forehead, blowing out the cheeks, and whistling, may be performed with the aid of a mirror to prevent muscle atrophy. Exposure of the face to cold and drafts is avoided.

Inflammatory arthritis

Swelling and heat of the joint lining causes a release of enzymes which soften and eventually destroy the cartilage. Rheumatoid arthritis, Lupus and psoriatic arthritis are inflammatory in nature.

anaphylaxis symptoms

Symptoms are sudden in onset and progress in severity over minutes to hours oFlushing oUrticaria oAngioedema oHypotension oBronchoconstriction

The health care provider believes that the client has a deficiency in the leukocyte responsible for cell-mediated immunity. What should the nurse check the WBC count for? Basophils T lymphocytes Plasma cells Monocytes

T lymphocytes Explanation: T lymphocytes are responsible for cell-mediated immunity, in which they recognize material as "foreign," acting as a surveillance system.

The anatomy and physiology instructor is explaining a cell-mediated response to the pre-nursing students. What actions would the instructor explain occur in a cell-mediated response? T-cell lymphocytes survey proteins in the body and attack the invading antigens. Toxins of invading antigens are neutralized. The invading antigens link together (agglutination). The invading antigens precipitate.

T-cell lymphocytes survey proteins in the body and attack the invading antigens. Explanation: During a cell-mediated response, T-cell lymphocytes survey proteins in the body, actively analyze the surface features, and respond to those that differ from the host by directly attacking the invading antigen. For example, a cell-mediated response occurs when an organ is transplanted. Immunoglobulins hinder the antigens physically by neutralizing their toxins through agglutination or by causing them to precipitate.

The anatomy and physiology instructor is explaining a cell-mediated response to the pre-nursing students. What actions would the instructor explain occur in a cell-mediated response? Toxins of invading antigens are neutralized. The invading antigens precipitate. T-cell lymphocytes survey proteins in the body and attack the invading antigens. The invading antigens link together (agglutination).

T-cell lymphocytes survey proteins in the body and attack the invading antigens. Explanation: During a cell-mediated response, T-cell lymphocytes survey proteins in the body, actively analyze the surface features, and respond to those that differ from the host by directly attacking the invading antigen. For example, a cell-mediated response occurs when an organ is transplanted. Immunoglobulins hinder the antigens physically by neutralizing their toxins through agglutination or by causing them to precipitate.

A schizophrenic patient presents to your clinic complaining of feeling bugs crawling on her skin despite no insects or skin changes on exam. Which of the following best describes this type of hallucination?

Tactile This type of hallucination is the feeling of bodily sensations.

A client with severe and persistent mental illness has been taking antipsychotic medication for 20 years. The nurse observes during a therapy session that the client's behavior includes repetitive movements of the mouth and tongue, facial grimacing, and rocking back and forth. The nurse recognizes these behaviors as indicative of what? Posturing Tardive dyskinesia Extrapyramidal side effects Loss of voluntary muscle control

Tardive dyskinesia Explanation: The client's behaviors are classic signs of tardive dyskinesia. Tardive dyskinesia, a syndrome of permanent involuntary movements, is most commonly caused by the long-term use of conventional antipsychotic drugs. Extrapyramidal side effects are reversible movement disorders induced by antipsychotic or neuroleptic medication. The client's behavior is not a loss of voluntary control or posturing.

Which cerebral lobe contains the auditory receptive areas? Temporal Parietal Occipital Frontal

Temporal Explanation: The temporal lobe plays the most dominant role of any area of the cortex in cerebration. The frontal lobe, the largest lobe, controls concentration, abstract thought, information storage or memory, and motor function. The parietal lobe contains the primary sensory cortex, which analyzes sensory information and relays interpretation to the thalamus and other cortical areas. The occipital lobe is responsible for visual interpretation.

a nurse is assessing a client who sustained a basal skull fracture and notes a thin stream of clear drainage coming from the client's right nostril. which of the following actions should the nurse take first? test the drainage for glucose suction the nostril notify the physician ask the client to blow his nose

Test the drainage for glucose. This is the priority nursing action. Because of the high risk of cerebral spinal fluid (CSF) leak in clients with basal skull fractures, the nurse should realize there is a possibility that the clear fluid coming from the client's nostril is CSF, which will test positive for glucose.

The student nurse correctly recognizes that which finding is best supported by genetic studies in the etiology of schizophrenia? That there is a weak correlation between genetics and schizophrenia. That schizophrenia is at least partially inherited. That there is no relationship at all between schizophrenia and genetics. That if a person has schizophrenia, distant relatives are also at risk.

That schizophrenia is at least partially inherited. Explanation: The most important studies have centered on twins; these findings have demonstrated that if one identical twin has schizophrenia, the other twin has a 50% chance of developing it as well. Fraternal twins have only a 15% risk. This finding indicates that schizophrenia is at least partially inherited. An individual's risk is proportionate to how closely he or she is related to a person with the disease. Consequently, distant relatives have a low or non-existent increase in risk.

A client is diagnosed with functional neurologic symptom disorder after extensive evaluation. The client developed blindness after witnessing the death of her twin sister in a car accident. When teaching the client's mother about her daughter's illness, which of the following would the nurse include? Her blindness will gradually disappear if proper ophthalmologic care is provided. Her blindness requires a conscious effort to maintain the feigned symptom. Her blindness results in increased anxiety and attention from family and friends. The blindness is a reaction to the trauma of losing her sister and has no physiologic basis.

The blindness is a reaction to the trauma of losing her sister and has no physiologic basis. Explanation: With functional neurologic symptom disorder, disorder is a psychiatric condition in which severe emotional distress or unconscious conflict is expressed through physical symptoms. In this case, the blindness is the physical symptom manifested due to the extreme emotional stress. There is no physiologic basis for the condition. Itis an unconscious process; it will not disappear with ophthalmologic care. The symptom is unconsciously designed to reduce anxiety,

A nurse is caring for a client who reports fatigue, unexplained bruising, and headaches. Medical History 0800: The client comes to the ED with reports of unexplained fatigue, reoccurring headaches, and bruising. The client states that the excessive fatigue started 6 months ago and has become so intense that they are unable to do their usual activities without needing to rest. The client denies difficulty sleeping or a change in stress level No past medical history per client BMI 20.2 Medications: Not taking any prescription medications. Client states that they have been taking Ibuprofen frequently over the past 6 months due to pain and swelling in joints. Vital Signs 0800: Temperature 36.8º C (98.2º F)Apical pulse 100/min, murmur auscultatedRespiratory rate 26/minBlood pressure 102/72 mm HgPulse oximetry 94% on room air Nurses' Notes 0800: Alert and oriented x 3.Reports generalized joint pain.Slight swelling noted in joints in hands and knees.Rates pain as a 3 on a scale of 0 to 10.Denies medication for pain. Skin pale and cool.Mucous membranes pain with slight bleeding of gums. 1200: Underwent a bone marrow aspiration, specimen sent to labTemperature 37.1°C (98.8°F)Apical pulse 102/minRespiratory rate: 26/minBlood pressure 104/70 mm HgPulse oximetry 96% on oxygen 2 L per nasal cannula Diagnostic Results CBC:RBC 4.0 million/mm3 (4.7 to 6.1 million/mm3)WBC 4.0 x 106/mm3 (4.7 to 6.1 x 106/mm3)Hemoglobin 10,500 mm3 (5,000 to 10,000 mm3)Hematocrit 10 g/dL (14 to 18 g/dL)Platelets 60,000/mm3 (150,000 to 40,000/mm3) Basic Metabolic Profile:BUN 18 mg/dL (10 to 20 mg/dL)Creatinine 1.0 mg/dL (0.6 to 1.3 mg/dL)Total calcium 9.5 mg/dL (9.0 to 10.5 mg/dL)Carbon dioxide 27 mEq/L (23 to 30 mEq/L)Chloride 101 mEq/L (98 to 106 mEq/L)Glucose 80 mg/dL (74 to 106 mEq/L)Potassium 4.2 mEq/L (3.5 to 5 mEq/L)Sodium 104 mEq/L (136 to 145 mEq/L) conditions: DIC hyperkalemia hyponatremia pneumonia acute nephritic syndrome findings: potassium level sodium level platelet count oxygen saturation level BUN and creatine

The client is at risk for developing DIC due to their platelet count When analyzing cues, the nurse should identify that the client findings of fatigue, headache, bruising, and decreased platelet count are related to thrombocytopenia. Clients who have this condition are at risk for disseminated intravascular coagulation which is condition that causes spontaneous excessive bleeding due to decreased clotting ability of the blood. The client's current platelet count is below the expected reference range, therefore, the client is as risk for bleeding and the nurse should monitor the client for the development of DIC.

A nurse in a clinic is caring for a client who has a new diagnosis of systemic lupus erythematosus (SLE). Client's first visit: Client presents with rash and cutaneous plaques on scalp, face, and neck, areas of hyperpigmentation also noted. Butterfly rash present on cheeks. Client reports progressive fatigue, despite increasing rest periods. Client reports joint tenderness. +1 edema noted. Client denies chest pain or discomfort. S1 S2 noted on auscultation. No pericardial friction rub or murmurs noted. Chest clear on auscultation. Client denies shortness of breath, cough, or congestion. Bowel sounds present in all four quadrants. Denies any gastrointestinal issues. Reports urinating with no difficulty. Laboratory tests: Urinalysis to check for proteinuria Echocardiogram to assess heart function Screening for osteoporosis with baseline bone mineral density test Begin medication therapy with low dose prednisone and NSAID. Will add immunosuppressive therapy following verification of diagnostics. Client's second visit: Client returns to clinic for medication check and to discuss diagnostic findings. Drag words from the choices below to fill in each blank in the following sentence. The client is at risk for developing photosensitivity chronic fatigue weight loss hypoglycemia

The client is at risk for developing photosensitivity and chronic fatigue. Chronic fatigue and photosensitivity are correct. The client's ANA and ESR tests confirm the diagnosis of SLE. Clients who have SLE are at risk of developing chronic fatigue as the disease progresses. The CBC report indicates the client is experiencing anemia, which will also contribute to fatigue. Clients who have SLE are also at risk of developing photosensitivity as the disease progresses. The assessment of the client reveals rash development, which can result from sensitivity to sunlight.

A nurse is caring for a client who has HIV. Vital Signs 1000: Temperature 38.1° C (100.6° F) Heart rate 122/min Respiratory rate 26/min BP 136/85 mm Hg Oxygen saturation 93% on room air Physical Examination 1000: Reports flu-like symptoms of headache, body aches, sore throat, low-grade fever, shortness of breath, productive cough. Swollen lymph nodes. Dry skin with rash. Weight loss of 15 lb over last 3 months with report of diarrhea and anorexia, difficulty eating due to oral ulcers. Diagnostic Results 1200: Chest x-ray: Areas of increased density and white infiltrates to lower right lobe indicative of pneumonia. 1600: Hemoglobin 11 g/dL (12 g/dL to 16 g/dL) Hematocrit 36% (37% to 47%) Platelet count 155,000/mm3 (150,000 to 400,000/mm3) WBC count 4,500/mm3 (5,000 to 10,000/mm3) CD4-T-Cell count 400 cells/mm3 (600 to 1,500 cells/mm3) Sputum culture Pneumocystis jirovecii The client is at risk for developing ____________ and ___________ word choices: sepsis tuberculosis hemorrhage malnutrition

The client is at risk for developing sepsis and malnutrition. Sepsis and malnutrition are correct. The client's sputum culture along with chest x-ray both indicate the client is experiencing pneumonia and the CD4-T-cell count is low, all of which places the client at risk for developing sepsis. The client is experiencing malnutrition as evidenced by weight loss, diarrhea, and oral ulcers, which places the client at risk for developing sepsis.

A client with major depression is scheduled to receive electroconvulsive therapy. The nurse understands that this treatment is typically used in which situation? The client is experiencing catatonia. The client is tolerating the initial drug therapy. Suicidality is of little concern. The level of depression is mild to moderate.

The client is experiencing catatonia. Explanation: Electroconvulsive therapy is an effective treatment for clients with severe depression. It is generally reserved for those whose disorder is refractory or intolerant to initial drug treatments and who are so severely ill that rapid treatment is required (e.g., clients with malnutrition, catatonia, or suicidality).

The nurse is seeing a client who is suspected of having the personality trait called alexithymia. Which characteristic about the client is consistent with this psychological explanation for somatization? The client experiences angry outbursts The client talks in detail about childhood experiences The client talks a great deal about external events The client is often tearful during clinic visits

The client talks a great deal about external events Explanation: A psychological theory that attempts to explain somatization refers to the personality trait alexithymia, which is associated with somatic symptom disorder. Individuals with alexithymia have difficulty identifying and expressing their emotions. They have a preoccupation with external events and are described as concrete externally oriented thinkers. Discussing in detail personal childhood experiences would be internally oriented, a characteristic unlikely in people with alexithymia. A show of emotion such as anger or sadness would be unlikely in a person with alexithymia as there is a disconnection from emotion, and as a result the person will have difficulty discharging and neutralizing the emotion.

After teaching a group of nursing students about somatic symptom disorder, the instructor determines that additional education is needed when the students identify which as true? The client believes he/she has a serious illness. The client usually thinks anxiety is behind the symptoms. The client believes that his/her condition is catastrophic and disabling. The client embraces the "sick role."

The client usually thinks anxiety is behind the symptoms. Explanation: Individuals with somatic symptom disorder perceive themselves as being "sicker than the sick" and report all aspects of their health as poor. Many eventually become disabled and cannot work. They typically visit health care providers multiple times per month and quickly become frustrated because their primary health care providers do not appreciate their level of suffering and are unable to validate that a particular problem accounts for their extreme discomfort. Clients do not have insight to identify anxiety as a problem.

The nurse is caring for a 35-year-old man whose severe workplace injuries necessitate bilateral below-the-knee amputations. How should the nurse anticipate that the client will respond to this news? The client will go through the stages of grief over the next week to 10 days. The client will progress sequentially through five stages of the grief process. The client will experience grief in an individualized manner. The client will require psychotherapy to process his grief.

The client will experience grief in an individualized manner. Explanation: Loss of limb is a profoundly emotional experience, which the client will experience in a subjective manner, and largely unpredictable, manner. Psychotherapy may or may not be necessary. It is not possible to accurately predict the sequence or timing of the client's grief. The client may or may not benefit from psychotherapy.

The client has severe headaches that are debilitating. The nurse has encouraged the client with a somatic symptom illness to keep a journal. Which treatment outcomes might be met by journaling? The client will identify the occurrence of physical symptoms when stressed. The client will express emotions privately. The nurse will control external stressors that trigger the client's physical symptoms. The nurse will assess the onset of physical symptoms.

The client will identify the occurrence of physical symptoms when stressed. Explanation: Teaching about the relationship between stress and physical symptoms is a useful way to help clients begin to see the mind-body relationship. This involves the expression of emotions, but the primary purposes is identifying a link between the two phenomena. Journaling is not suggested as a means for the nurse to control the client's stressors or to assess symptoms; the client is the focus of the exercise.

The nurse is creating a plan of care for a client with major depressive disorder. Which outcome will the nurse assign as the highest priority? The client will not experience agitation. The client will avoid causing harm to others. The client will independently carry out activities of daily living. The client will be free from stress.

The client will independently carry out activities of daily living. Explanation: The most realistic and measurable outcome is for the client to be able to attend to self-care needs independently such as hygiene, nutrition, etc. This will indicate that medications and therapy have been effective. Avoiding agitation and stress is not a realistic goal since these are not possible to avoid. The client should learn techniques for management of stress and agitation. It is less likely that a client that has major depressive disorder will be harmful to others.

A nurse is caring for a client with multiple sclerosis. Client education about the disease process includes which explanation about the cause of the disorder? The immune system recognizes one's own tissues as "self." The immune system recognizes one's own tissues as "foreign." Excess cytokines cause tissue damage. Regulatory mechanisms fail to halt the immune response.

The immune system recognizes one's own tissues as "foreign." Explanation: The immune system's recognition of one's own tissues as "foreign" rather than self is the basis of many autoimmune disorders, including multiple sclerosis. When regulatory mechanisms fail to halt the immune response or excess cytokines are produced, pathology occurs (e.g., allergies, hypersensitivity).

The nurse is caring for a client with hypoxia. What does the nurse understand is true regarding the client's oxygen level and the production of red blood cells? The brain senses low oxygen levels in the blood and stimulates hemoglobin, which binds to more red blood cells. The bone marrow is stimulated by low oxygen levels in the blood and stimulates erythropoietin, maturing the red blood cells. The kidneys sense low oxygen levels in the blood and stimulate erythropoietin, stimulating the marrow to produce more red blood cells. The kidneys sense low oxygen levels in the blood and stimulate hemoglobin, stimulating the marrow to produce more red blood cells.

The kidneys sense low oxygen levels in the blood and stimulate erythropoietin, stimulating the marrow to produce more red blood cells. Explanation: If the kidney detects low levels of oxygen, as occurs when fewer red cells are available to bind oxygen (as with anemia), erythropoietin levels increase, stimulating the marrow to produce more erythrocytes (red blood cells).

A 64-year-old male client, who leads a sedentary life-style, and a 31-year-old female client, who has a very stressful and active life-style, require a vaccine against a particular viral disorder. As the nurse, you would know that in one of these clients the vaccine will be less effective. In which client is the vaccine more likely to be less effective and why? The male client because of his age The female client because of her age The male client because of his life-style The female client because of her life-style

The male client because of his age Explanation: Vaccines are less effective in an older adult than in a younger adult because the activity of the immune system declines with the aging process. The lifestyle or gender of the client does not have great implications on the effectiveness of a vaccine.

The nurse is teaching a client who has been diagnosed with Hashimoto's thyroiditis. Which statement correctly describes the process of autoimmunity? The normal protective immune response attacks the body, damaging tissues. The body overproduces immunoglobulins. The body produces inappropriate or exaggerated responses to specific antigens. A deficiency results from improper development of immune cells or tissues.

The normal protective immune response attacks the body, damaging tissues. Explanation: Autoimmunity happens when the normal protective immune response paradoxically turns against or attacks the body, leading to tissue damage. It is not an immune deficiency. An exaggerated immune response describes a hypersensitivity. An overproduction of immunoglobulins is the definition of gammopathies.

A client is treated in the clinic for a sexually transmitted infection, and the nurse suspects that the client is at risk for HIV. The physician determines that the client should be tested for the virus. What responsibility does the nurse have? The nurse will call the client with the results of the test. The nurse ensures a written consent is obtained prior to testing. The nurse will inform the client that the results will have to be reported to the Centers for Disease Control and Prevention (CDC). The nurse should send the client to have the blood drawn without informing him about the specific screening test.

The nurse ensures a written consent is obtained prior to testing. Explanation: The nurse ensures that a written consent is obtained before testing for human immunodeficiency virus (HIV) and keeps the results of HIV testing confidential. The client should never be tested without his knowledge. The physician will review the results when the client comes in for a follow-up visit. It is not necessary for the nurse to report results to the CDC.

A nurse is caring for a 24-year-old client who reports a recent fall, hitting their head and right shoulder. Medical History Client reports falling and hitting their head and right shoulder after slipping on a wet floor yesterday. Denies loss of consciousness. Complains of pain in right shoulder. Has taken both acetaminophen and ibuprofen for pain with minimal relief obtained. Stayed up entire night playing video games yesterday to distract self from pain. Reports intermittent nausea and vomiting. Nurses' Notes 0900: Reports pain in right shoulder. Limited range of motion noted. Rates pain as 7 on a scale of 0 to 10. Denies numbness and tingling in arm. No swelling or bruising over the shoulder. Fingers warm with capillary refill time less than 3 seconds, sensation intact. Drowsy. Oriented to person, place, and time. Irritable and restless at times. PERRLA. Glascow score of 15. No hematomas noted on head. No nausea or vomiting at this time. 1000: Continues to report pain in right shoulder. Pain increased from 7 to 8 on a scale of 0 to 10. Increased drowsiness noted. Glascow unchanged. Vital Signs Oral temperature 37.5º C (99.6º F) Heart rate 76/min and regular Respiratory rate 20/min even and unlabored Blood pressure 112/70 mm Hg in left arm Pulse oximetry 98% on room air Complete the following sentence by using the list of options. The nurse should first address the client's ______ followed by the client's _____.

The nurse should first address the client's drowsiness followed by the client's right shoulder pain. Drowsiness is correct. Loss of consciousness is the most important variable to assess with head injuries. A decrease or a change in loss of consciousness is often the first sign of deterioration and can indicate increased intracranial pressure. This should be reported to the provider immediately. Right shoulder pain is correct. The nurse should address the client's right shoulder pain after addressing the client's drowsiness. A client's recovery can be affected by pain by inhibiting their ability to become active and involved in self-care. The goal is to provide pain relief so that the client is able to participate in their recovery and to improve the client's functional status. Assessment of pain should include intensity, quality, duration, and location.

Which statement accurately reflects current stem cell research? Clinical trials are underway only in clients with acquired immune deficiencies. Stem cell transplantation has been performed in the laboratory only. Stem cell transplantation cannot restore immune system functioning. The stem cell is known as a precursor cell that continually replenishes the body's entire supply of both red and white cells.

The stem cell is known as a precursor cell that continually replenishes the body's entire supply of both red and white cells. Explanation: The stem cell is known as a precursor cell that continually replenishes the body's entire supply of both red and white cells. Stem cells comprise only a small portion of all types of bone marrow cells. Research conducted with mouse models has demonstrated that once the immune system has been destroyed experimentally, it can be completely restored with the implantation of just a few purified stem cells. Stem cell transplantation has been carried out in human subjects with certain types of immune dysfunction, such as severe combined immunodeficiency. Clinical trails are underway in clients with a variety of disorders with an autoimmune component, including systemic lupus erythematosus, rheumatoid arthritis, scleroderma, and multiple sclerosis.

A client for whom the nurse is caring has positron emission tomography (PET) scheduled. In preparation, what should the nurse explain to the client? A.The test may result in dizziness or lightheadedness. B.An allergy to iodine precludes getting the radio-opaque dye. C.The client will need to endure loud noises during the test. D.The test will temporarily limit blood flow through the brain.

The test may result in dizziness or lightheadedness.

What can happen if you confront a family member with a Fictitious disorder (if they are the one perpetrating the disorder on the patient)?

They can take the child and leave AMA, which is dangerous for the child.

A psychiatric-mental health nurse is teaching a class about schizophrenia. When describing delusions, which information would the nurse most likely include? They are implausible within the person's ethnic background. They are easily changed with conflicting evidence. They are variable in nature. They may include elements of a situation that could occur in real life.

They may include elements of a situation that could occur in real life. Explanation: Delusions are fixed, false beliefs that cannot be changed by conflicting evidence. They can be situations that could occur in real life and are plausible in the context of the person's ethnic and cultural background, or they may be clearly fantastical. They usually involve a misinterpretation of the client's experience.

A client arrives at the clinic and reports a very sore throat as well as a fever. A rapid strep test returns a positive result and the client is given a prescription for an antibiotic. How did the streptococcal organism gain access to the client to cause this infection? From being outside in the cold weather and decreasing resistance Breathing in airborne dust Through the skin Through the mucous membranes of the throat

Through the mucous membranes of the throat Explanation: In a streptococcal throat infection, the streptococcal organism gains access to the mucous membranes of the throat.

When completing a baseline assessment of a client with depression, which diagnostic tests would the nurse anticipate? Thyroid function tests Abdominal ultrasound Renal function tests Coagulation profile

Thyroid function tests Explanation: A physical examination is recommended with baseline vital signs and baseline laboratory tests, including a comprehensive blood chemistry panel, complete blood counts, liver function tests, thyroid function tests, urinalysis, and electrocardiograms. These physical examinations can help to rule out any underlying medical conditions that may be causing or exacerbating an existing depression. The other diagnostic tests indicated in the options are not related to identifying underlying medical conditions that are commonly found comorbid to depression.

A nurse is caring for a client who has delayed hypersensitivity reaction. The nurse should expect which of the following manifestations? bronchospasm serum sickness tissue damage at the site excessive mucus secretion

Tissue damage at the site The nurse should expect the manifestations of edema, induration, ischemia, and tissue damage at the site occurring hours to days after exposure. A positive purified protein derivative test for tuberculosis is an example of a type IV hypersensitivity reaction.

A client with recurrent headaches has been told by the physician that the cause is likely psychosomatic. The client reports this conversation to the nurse and says, "That just can't be true! My head hurts so bad sometimes that it makes me sick to my stomach." Which is the nurse's best response? To say "the pain in your head is very real." To say "well, that's not what your doctor thinks." To give the client some privacy and time to calm down To say nothing and sit quietly with the client

To say "the pain in your head is very real." Explanation: When the nurse says, "The pain in your head is very real," the nurse is validating the client's pain as real. The client is asking for some type of validation. In the situation presented, the client's headaches are very real to him or her. The client needs to talk out the feelings regarding what the physician has told him or her. It would be inappropriate for the nurse to say nothing. Leaving to give the client privacy does not engage the client or validate his or her circumstances. Saying "Well, that's not what your doctor thinks," would put the client on the defensive.

A nurse is caring for a client who was admitted with acute psychosis and is being treated with haloperidol. The nurse should suspect that the client may be experiencing tardive dyskinesia when the client exhibits which of the following? (sata) urinary retention and constipation tongue thrusting and lip smacking fine hand tremors and pill rolling facial grimacing and eye blinking involuntary pelvic rocking and hip thrusting movements

Tongue thrusting and lip smacking Facial grimacing and eye blinking Involuntary pelvic rocking and hip thrusting movements

The brain stem holds the medulla oblongata. What is the function of the medulla oblongata? Transmits sensory impulses from the brain to the spinal cord Controls parasympathetic nerve impulses in the PNS Transmits motor impulses from the brain to the spinal cord Controls striated muscle activity in blood vessel walls

Transmits motor impulses from the brain to the spinal cord Explanation: The medulla oblongata lies below the pons and transmits motor impulses from the brain to the spinal cord and sensory impulses from peripheral sensory neurons to the brain. The medulla contains vital centers concerned with respiration, heartbeat, and vasomotor activity (the control of smooth muscle activity in blood vessel walls).

ocular myasthenia gravis treatment

Treatment: 1) medications exist that improve the communication between nerves/muscles 2) stay away from meds that cause muscle weakness 3) patching/prisms may be used to treat individuals w/ double vision

Is the following statement true or false? Polyarticular refers to a rheumatic disease affecting more than one joint.

True Rationale: Monoarticular means affecting a single joint; polyarticular means affecting multiple joints.

The spinal cord is composed of 31 pairs of spinal nerves. How many pairs of thoracic nerves are contained within the spinal column? Twelve Eight One Five

Twelve Explanation: There are twelve pairs of thoracic nerves, five lumbar and sacral nerves, eight cervical, and one coccygeal.

The client presented to the emergency department with a report of chest pain. The nurse performs a thorough physical examination for this client, who has a history of a somatic symptom illness. Which is the best rationale for the physical exam? Underlying pathology should be ruled out. Physical exams are reimbursed by third-party payers. Physical disorders underlie somatic disorders. Ease the client's mind that the nurse is looking for physical illness.

Underlying pathology should be ruled out. Explanation: The nurse must investigate physical health status thoroughly to ensure that there is no underlying organic pathology requiring treatment. When a client has been diagnosed with a somatic symptom illness, it is important not to dismiss all future complaints because at any time the client could develop a physical condition that would require medical attention. Financial factors do not drive the nurse's plan for assessments and it would be inappropriate to perform assessment for the sole purpose of easing the client's mind. Psychological, not physical, problems underlie somatic disorders.

While caring for a client, the nurse notes petechiae on the client's trunk and lower extremities. What precaution will the nurse take when caring for this client? Apply supplemental oxygen to maintain the client's oxygenation. Elevate the client's head of the bed. Use an electric razor when assisting client with shaving. Where a mask when entering the client's room.

Use an electric razor when assisting client with shaving. Explanation: Petechiae are associated with severe thrombocytopenia, placing the client at risk for bleeding. The nurse should use an electric razor when assisting the client with shaving. Elevating the head of the bed and applying supplemental oxygen would be appropriate for a client with decreased oxygenation. Wearing a mask when entering the client's room would be appropriate for a client with neutropenia, not thrombocytopenia.

A 38-year-old client has begun to suffer from rheumatoid arthritis and is being assessed for disorders of the immune system. The client works as an aide at a facility that cares for children infected with AIDS. Which is the most important factor related to the client's assessment? Home environment Diet Use of other drugs Age

Use of other drugs Explanation: The nurse needs to review the client's drug history. These data will help her to assess the client's susceptibility to illness because certain past illnesses and drug use, such as corticosteroids, suppress the inflammatory and immune responses. The client's age, home environment, and diet do not have any major implications during assessment because they do not indicate the client's susceptibility to illness.

The client is to receive a unit of packed red blood cells. What is the nurse's first action? Check the label on the unit of blood with another registered nurse. Verify that the client has signed a written consent form. Observe for gas bubbles in the unit of packed red blood cells. Ensure that the intravenous site has a 20-gauge or larger needle.

Verify that the client has signed a written consent form. Explanation: All the options are interventions the nurse will do to ensure the blood transfusion is safe. The question asks about the first action of the nurse. The first action would be verifying that the client has signed a written consent form. Then, the nurse would ensure the intravenous site has a 20-gauge or larger needle. The nurse would proceed to obtain the unit of blood, check the blood with another registered nurse, and observe for gas bubbles in the unit of blood.

a nurse is reviewing the medical record of a client who has a new prescription for clozapine for the treatment of schizophrenia. which of the following findings indicates a contraindication to clozapine? asthma fasting blood glucose 120mg/dL WBC count 3,3000/mm3 hypertension

WBC count 3,300/mm3 Clozapine is contraindicated in patients that have a WBC count of LESS THAN 3,500 before therapy

A client is admitted to the psychiatric hospital with a diagnosis of schizophrenia. During the physical examination, the client's arm remains outstretched after the nurse obtains the pulse and blood pressure, and the nurse must reposition the arm. The nurse interprets this as what? Waxy flexibility Echopraxia Retardation Hypervigilance

Waxy flexibility Explanation: Waxy flexibility, the ability to assume and maintain awkward or uncomfortable positions for long periods, is characteristic of catatonic schizophrenia. Clients commonly remain in these awkward positions until someone repositions them. Exchopraxia refers to the involuntary imitation of another person's movements and gestures. Hypervigilance refers to the sustained attention to external stimuli, as if expecting something important or frightening to occur. Retardation refers to slowed movements.

A client had a lumbar puncture performed at the outpatient clinic and the nurse has phoned the client and family that evening. What does this phone call enable the nurse to determine? A.Whether the client understood accurately why the test was done B.Whether the client has had any complications of the test C.Whether the client's family had any questions about why the test was necessary D.What are the client's and family's expectations of the test

Whether the client has had any complications of the test

A client is preparing for discharge from the emergency department after sustaining an ankle sprain. The client is instructed to avoid weight bearing on the affected leg and is given crutches. After instruction, the client demonstrates proper crutch use in the hallway. What additional information is most important to know before discharging the client? Whether the client needs to navigate stairs routinely at home Whether the client drives a car with a stick shift Whether the client parks his car on the street Whether pets are present in the home

Whether the client needs to navigate stairs routinely at home Explanation: Knowing whether the client must routinely navigate steps at home is most important. If the client must navigate steps, special crutch-walking techniques must be taught to safely navigate the stairs. Although pets, parking on the street, and driving a car with a stick shift can pose problems for the client, these factors aren't important to know before discharging the client with crutches.

A patient has been diagnosed with meningococcal meningitis at a community living home. When should prophylactic therapy begin for those who have had close contact with the patient? Therapy is not necessary prophylactically and should only be used if the person develops symptoms. Within 48 hours after exposure Within 24 hours after exposure Within 72 hours after exposure

Within 24 hours after exposure Explanation: People in close contact with patients with meningococcal meningitis should be treated with antimicrobial chemoprophylaxis using rifampin (Rifadin), ciprofloxacin hydrochloride (Cipro), or ceftriaxone sodium (Rocephin). Therapy should be started within 24 hours after exposure because a delay in the initiation of therapy limits the effectiveness of the prophylaxis.

a nurse is caring for a client who has rheumatoid arthritis and tells the nurse that she wears a copper bracelet to help her feel better. which of the following responses should the nurse make? yes i understand that you feel better wearing your bracelet why do you think the copper helps with your arthritis believing objects have powers to make you feel better has no scientific basis i think you should rely more on your medication therapy than on your bracelet.

Yes, I understand that you feel better wearing your bracelet. The nurse illustrates the therapeutic communication technique of accepting. The nurse demonstrates the knowledge that the bracelet is harmless for the client and shows respect for the client's beliefs.

a nurse is caring for several clients who have mental health disorders at an assisted-living facility. which of the following clients should the nurse determine needs to be seen by a provider immediately? a client who is taking olanzapine and experiences dizziness when first standing up a client who is taking chlorpromazine and reports vomiting twice a client who is taking thioridazine and has daytime drowsiness a client who is taking clozapine, and has flu-like manifestations

a client who is taking clozapine, and has flu-like manifestations

Huntington's Disease (HD)

a fatal genetic neurological disorder whose onset is in middle age Results in progressive involuntary choreiform movement and dementia Genetic disorder each child of a parent with Huntington has a 50% chance of getting it 10 to 20 years after diagnosis develop heart failure, pneumonia, or infection or fall or choke

Which client does the nurse identity as having the greatest risk for developing somatic symptom disorder (SSD)? a white female with a college degree who works in advertising a female of African descent who works two minimum wage jobs a male of African descent with an associate degree in nursing a white male who works two minimum wage jobs

a female of African descent who works two minimum wage jobs Explanation: Epidemiologic studies have reported that SSD occurs primarily in nonwhite, less educated women, particularly those with a lower socioeconomic status and high emotional distress. Men are less likely to be diagnosed with SSD, partly because of stereotypic male traits, such as a disinclination to admit discomfort or seek help for their symptoms. Based on this information, the client who is at greatest risk for SSD is the female of African descent who works two minimum wage jobs because there are three risk factors (ethnicity, gender, and low socioeconomic status). The white male, the white female, and the male client of African descent each have one risk factor for SSD.

Subdural Hematoma (SDH)

a mass of blood between the dura mater and the arachnoid mater of the brain; the most common type of traumatic brain injury (TBI) can be caused by coagulopathies (inability to clot), rupture of aneurysm, or trauma

manifestations of myasthenia gravis

a motor disorder initially symptoms involve ocular muscles; diplopia and ptosis weakness of facial muscles, swallowing and voice impairment, generalized weakness

A client with a tentative diagnosis of myasthenia gravis is admitted for a diagnostic workup. Myasthenia gravis is confirmed by: Brudzinski's sign. a positive sweat chloride test. Kernig's sign. a positive edrophonium (Tensilon) test.

a positive edrophonium (Tensilon) test. Explanation: A positive edrophonium test confirms the diagnosis of myasthenia gravis. After edrophonium administration, most clients with myasthenia gravis show markedly improved muscle tone. Kernig's sign and Brudzinski's sign indicate meningitis. The sweat chloride test is used to confirm cystic fibrosis.

Multiple Sclerosis (MS)

a progressive immune-related demyelination disease 4 types: 1. relapsing-remitting 2. primary progessive 3. secondary progessive 4. progressive-relapsing classic age for MS diagnosis is 20s progressive weakening of muscles

A nurse is reading a journal article about children and somatic symptom disorder. Which of the following would the nurse expect to find as the most common symptoms reported by children with this disorder? Select all that apply. Abdominal pain Fatigue Nausea Tingling in the hands Headache Diarrhea

abdominal pain fatigue nausea headache In children, the most common symptoms are frequent abdominal pain, headache, fatigue, and nausea.

A nurse is caring for a client on a medical-surgical unit. Nurse's Notes Client admitted to the unit for a lower GI bleed. Continues to have frequent bloody stools and is scheduled for a lower endoscopy in 4 hr. The client is receiving their fourth unit of packed red blood cells (packed RBCs). Unit of fourth packed RBCs started at a rate of 250 cc/hr. Thirty minutes after the transfusion started, client started reporting dyspnea and restlessness. Crackles auscultated in bilateral lower lobes, oxygen saturation 92% on 2L nasal cannula, and jugular vein distension noted. Vital Signs Temperature 37.3° (99.1° F) Blood pressure 189/90 mm Hg Pulse rate 100/min Respiratory rate 28/min specify what condition the client is most likely experiencing, two actions the nurse should take to address that condition, and two parameters the nurse should monitor to assess the client's progress. actions to take: administer furosemide administer epinephrine decrease the infusion rate administer diphenhydramine administer an antibiotic potential condition: acute intravascular hemolytic reaction bacterial reaction anaphylactic reaction transfusion associated circulatory overload parameters to monitor: hives weights chills I&Os lower back pain

actions to take: administer furosemide decrease the infusion rate potential condition: transfusion associated circulatory overload parameters to monitor: I&Os weight The nurse should decrease the rate of the transfusion and administer a furosemide or other diuretic as prescribed because the client is most likely experiencing transfusion-associated circulatory overload (TACO). The nurse should monitor the client's intake and output as well as the client's weight to assess fluid volume status, especially after the diuretic is administered.

a nurse is caring for a client who has schizophrenia who consistently does the opposite of what the nurse asks of him. the nurse recognizes this as which of the following alterations in behavior? automatic obedience waxy flexibility active negativism impaired impulse control

active negativism

subdural hematoma timelines

acute: symptoms 24-48 hours major trauma with contusion subacute: 48 hrs to 2 weeks less severe trauma chronic: 3 weeks to months from minor injuries / seen in elderly

medical management of encephalitis

acyclovir for HSV infection amphotericin and or other antifungal agents for fungal infection

A mental health nurse is referring a client who has an alcohol addiction to a 12-step alcoholics anonymous program. the nurse should inform the client that which of the following is the basic concept of a 12-step program? admit life is unmanageable. detoxifying from the addictive substance identifying stimuli that promote drinking including family in counseling sessions

admit life is unmanageable

manifestations of CJD

affective, sensory, motor, and cognitive impairments

Which of the following signs, symptoms, or diseases is most commonly seen when evaluating a patient for the "5 A's" of Alzheimer's disease? Agnosia Agnathia Alopecia Acrodermatitis Aneurysm Achondroplasia

agnosia Agnosia is the inability to recognize familiar objects, tastes, sounds, and other sensations.

Nontremor-dominant PD

akinetic-rigid syndrome (bradykinesia which are small slow movements) (rigidity) postural instability gait disorder (stooped posture, decreased arm swing, and shuffling gait and feet)

IgE antibodies are involved in

allergic disorders

a nurse is caring for a client who has developed gout. which of the following medications should the nurse prepare to administer? zolpidem alprazolam sprionolactone allopurinol

allopurinol MY ANSWER Allopurinol is a xanthene oxidase inhibitor that reduces uric acid synthesis. The medication is prescribed to treat gout.

causes of ICP changes

alterations in volume in skull (blood flow, tissue edema) hemorhaggic stroke brain tumors hydrocephalus

Which of the following signs, symptoms, or diseases is most commonly seen when evaluating a patient for the "5 A's" of Alzheimer's disease? Achromatopsia Agyria Amnesia Alopecia Abasia Acanthosis

amnesia Amnesia is defined as a loss of memory

A nurse is caring for a client who has sickle cell disease. Nurses' Notes 0800: Client reports fatigue, muscle weakness, joint pain, and dyspnea. Sclerae is jaundiced. 2.5 cm (1 in) by 2.5 cm (1 in) open ulcer noted on inner left ankle. Vital Signs 0800: Temperature 37.5° C (99.5° F) Blood pressure 122/68 mm Hg Heart rate 95/min Respiratory rate 28/min Oxygen saturation 95% on room air 1000: Temperature 37.5° C (99.5° F) Blood pressure 88/56 mm Hg Heart rate 112/min Respiratory rate 26/min, labored Oxygen saturation 90% on room air Diagnostic Results 1000: Hct 26% (37% to 47%) Hgb 8 g/dL (12 to 16 g/dL) For each client finding, click to specify if the finding is consistent with sickle cell disease, iron deficiency anemia, or leukemia. Each finding may support more than 1 disease process. jaundice joint pain respiratory status HR 100 ankle ulcer

ankle ulcer: sickle cell disease respiratory status: sickle cell disease, iron deficiency anemia, leukemia joint pain: sickle cell disease, leukemia heart rate 100: sickle cell disease, iron deficiency anemia, leukemia jaundice: sickle cell disease

which of the following drug classes is most likely used to treat schizophrenia?

antipsychotics Patients who are diagnosed with schizophrenia should be treated with antipsychotics (neuroleptics) to help improve their quality of life. These medications; however, have several side effects, so you should always be aware of extrapyramidal symptoms and neuroleptic malignant syndrome in these patients

rheumatic diseases most commonly manifest the clinical features of ____________________ and pain, with periods of remission and exacerbation.

arthritis (joint inflammation)

a nurse is caring for a client who has a mental health disorder. the client asks about his medications and their effects. the nurse asks the client why he needs to know this. which of the following nontherapeutic communication techniques is the nurse using? changing the subject asking for an explanation behaving defensively arguing

asking for an explanation

The genetic theory, when applied to the occurrence of depression, supports that the psychiatric nurse should ... encourage the client to seek genetic counseling before considering a pregnancy. assess for depression in the client's family history. educate the client regarding the symptoms of related physical disorders. prepare the client for diagnostic genetic testing to confirm the diagnosis.

assess for depression in the client's family history. Explanation: The genetic theory, when applied to the occurrence of depression, supports that the psychiatric nurse should assess for depression in the client's family history.

Which of the following symptoms is best considered "positive" and can display as changing topics of conversation through nonsensical associations?

associative looseness Associative looseness is a type of thinking displayed by the patient's speech pattern. They display speech in which ideas shift from one unrelated subject to another, and the patient is typically unaware that the topics are unconnected.

atopic allergic reaction

asthma, allergic rhinitis, atopic dermatitis familial

two types of allergic reactions

atopic and nonatopic

T lymphocytes

attack invaders directly, secrete cytokines, and stimulate immune system responses

a nurse in a mental health facility is caring for a client in the busy facility dining room during lunchtime when suddenly the client becomes angry and throws a chair. which of the following interventions should the nurse perform first? restrain the client to prevent injury to himself or others place the client in a monitored seclusion room until he is calm administer a PRN antianxiety medication attempt to talk the client down

attempt to talk the client down

Monro-Kellie hypothesis

because of limited space in the skull, an increase in any one of components of the skull (brain tissue, blood, CSF) will cause a change in the volume of the others

hemorrhagic stroke

bleeding in the brain caused by a broken or leaking blood vessel caused by: - hypertension - aneurysm - AV malformations - increased ICP - compression / secondary ischemia

Type 2 cytotoxic reaction examples

blood transfusion reactions autoimmune hemolytic anemia erythroblastosis fetalis myasthenia gravis Goodpasture syndrome pernicious anemia thrombocytopenia

humoral or antibody response

bone marrow response b cells > plasma cells > antibodies

complications of ICP changes

brainstem herniation ( a lot of our goal is to prevent herniation) diabetes insipidus SIADH infection

A nurse is caring for a client who has a history of a cerebral aneurysm. Which diagnostic test does the nurse anticipate to monitor the status of the aneurysm? electroencephalogram echoencephalography cerebral angiography milligram

cerebral angiography Explanation: The nurse would anticipate a cerebral angiography, which detects distortion of the cerebral arteries and veins . A milligram detects abnormalities of the spinal canal. An electroencephalogram records electrical impulses of the brain. An echoencephalography is an ultrasound of the structures of the brain.

aura phase of migraine

characterized by focal neurologic symptoms like: - visual disturbances - numbness - tingling - mild confusion - drowsiness - dizziness

pharmacologic therapy of myasthenia gravis

cholinesterase inhibitor: pyrostigmine bromide (mestinon) immunomodulating therapy -plasmapheresis (plasma cleansing like dialysis) -thymectomy (removal of thymus)

Meningitis symptoms

classic triad of headache, neck stiffness and fever that remains high -photophobia, vomiting, altered mental status, rash older adults may have mental status and neurological changes

A psychiatric-mental health nurse is conducting an in-service education program for a group of staff nurses about somatic symptom disorder (SSD). The nurse determines that the teaching was successful when the group identifies which characteristic as being associated with SSD? Select all that apply. Clients often report that their overall health is otherwise good. Medical tests demonstrate an underlying physical disorder. Client issues are often diffuse and complex. The client's symptoms remain relatively constant. Clients typically move from one provider to another.

clients typically move from one provider to another. client issues are often diffuse and complex. Explanation: Somatic symptom disorder (SSD) is one of the most difficult disorders to manage because its symptoms tend to change, are diffuse and complex, and vary and move from one body system to another. Individuals with SSD perceive themselves as being "sicker than the sick" and report all aspects of their health as poor. They typically visit health care providers many times each month and quickly become frustrated because their primary health care providers do not appreciate their level of suffering and are unable to validate that a particular problem accounts for their extreme discomfort. Consequently, individuals with SSD tend to "provider shop," moving from one to another until they find one who will give them new medication, hospitalize them, or perform surgery. Because the source of worrisome physical symptoms cannot be determined through medical or laboratory tests, medical or psychiatric interviews, or medical imaging, they repeatedly seek a medical reason for their discomfort and ask for relief from suffering.

what med is used to treat schizophrenia when no other 2nd generation antipsychotic is effective?

clozapine

Medical management of arthropod-borne virus (arboviral) encephalitis is aimed at preventing renal insufficiency. preventing muscular atrophy. maintaining hemodynamic stability and adequate cardiac output. controlling seizures and increased intracranial pressure.

controlling seizures and increased intracranial pressure. Explanation: There is no specific medication for arbovirus encephalitis; therefore symptom management is key. Medical management is aimed at controlling seizures and increased intracranial pressure.

hemorhagic stroke prevention

controlling that hypertension girl!!!!!

Hypoproliferative anemia

defect in production of RBCs Caused by iron, vitamin B12, or folate deficiency, decreased erythropoietin production, cancer

Thallasemia

defective gene, red blood cells short lived hypochromia (decreased Hgb of RBC), microcytosis (small RBC) - defective creation of Hgb - characterized by severe anemia, marked hemolysis, and ineffective erythropoiesis - excessive iron can cause organ dysfunction - inherited

Chronic illnesses may contribute to immune system impairment in various ways. Renal failure is associated with increased incidence of infection. deficiency in circulating lymphocytes. altered production of white blood cells. decreased bone marrow function.

deficiency in circulating lymphocytes. Explanation: Renal failure is associated with a deficiency in circulating lymphocytes. Diabetes mellitus is associated with increased incidence of infection. Chemotherapy causes decreased bone marrow function. Leukemia is associated with altered production of white blood cells.

Illness Anxiety Disorder

does not experience symptoms previously referred to as Hypochondriasis - they frequently check for signs and symptoms - goes to extraordinary lengths to prevent illness - preoccupied with having / getting an illness

which of the following symptoms is best considered an affective symptom in patients with schizophrenia?

dysphoria Due to their psychological state, delusions and symptoms, patients may become dysphoric. This means that they are in a state of unease or dissatisfaction with their condition. This dysphoria is usually accompanied with depression, anxiety and agitation, and increases suicide risk in patients.

a nurse is caring for a client who has severe manifestations of schizophrenia and is medicated PRN for agitation with haloperidol. the nurse should assess the client for which of the following adverse effects? dysrhythmias cataracts pancreatitis bleeding

dysrhythmias haloperidol can cause QT prolongation so monitor ECG

a nurse is planning to administer haloperidol to a client who has acute psychosis. the nurse should monitor the client for which of the following findings as an adverse effect of the medication? excess salivation increased agitation diarrhea dystonia

dystonia

manifestations of huntington disease

early: subtle changes in personality, cognition, and movement -chorea (rapid jerky purposeless movements), cognitive impairments, tics, slurred speech, difficulty chewing and swallowing, irritability, anger, depression abnormal involuntary movements (as it progresses may involve twisting of entire body), intellectual decline, and emotional disturbances - trouble swallowing, facial tics, anger, agitation

type 4: delayed or cellular reaction symptoms

erythema itching

a nurse is assessing a client who has disseminated intravascular coagulation (DIC). which of the following findings should the nurse expect? excessive thrombosis and bleeding progressive increase in platelet production immediate sodium and fluid retention increased clotting factors

excessive thrombosis and bleeding MY ANSWER The nurse should expect excessive thrombosis and bleeding of mucous membranes because both DIC impairs both coagulation and anticoagulation pathways.

Which of the following is most likely a component of the Glasgow Coma Scale?

eye opening Eye opening is rated 1 to 4 points. Examples of how to rate eye opening: If your patient is spontaneously moving their eyes (4). You have to say something to your patient for them to open their eyes (3). You have to painfully stimulate them to open (2) and they do not open their eyes no matter what you do (1). 4 = spontaneous response, 3 = to voice, 2 = to pain, 1= none.

types of seizures

focal: begin in one part of brain simple focal: consciousness remains intact (staring off into distance) complex focal: impairment of consciousness generalized seizures: involve the entire brain, usually a loss of consciousness involved.

functional neurologic disorders

formerly known as Converwsion Disorder - neurologic / sensory problems with no obvious cause (aphonia, anosmia, akinesia) - IT IS REAL TO THE CLIENT - can be caused from neuro changes in brain or an ADVERSE CHILD EVENT

spenic sequestration

functioning spleen has outflow sickling causing a sickling crisis here, and the spleen collects all the blood during this crisis. this causes the blood circulatory volume to drop because the spleen is holding onto the blood. sx: abdominal pain, tachycardic, tachypneic, hypotension this patient presents similar to a patient in shock

A psychiatric-mental health nurse is reviewing a journal article about schizophrenia and disorders related to it. The nurse demonstrates understanding of the article by identifying which information as a major factor associated with increasing a person's risk for schizophrenia? the individual's birth order in the family genetics underactivity of the dopamine pathways poor family dynamics

genetics Explanation: With schizophrenia, genetic associations have been identified in a number of regions of the brain, and some have already been tied to altered dopamine transmission. However, these associations are more likely related to the susceptibility of developing the schizophrenia disorder rather than the cause of it. Positive symptoms of schizophrenia, specifically, hallucinations and delusions, are thought to be related to dopamine hyperactivity. The likelihood of first-degree relatives (including siblings and children), not birth order, developing schizophrenia has long been recognized as 10 times more likely than in individuals in the general population. Poor family dynamics has not be associated with the development of schizophrenia.

in vivo transfusion

give small amount of known incompatible blood to patient, watch for reaction, then give more blood

a nurse is caring for a client who has schizophrenia. which of the following statements by the client indicates concrete thinking? i am aware that each problem has only one solution i am a prophet of the most high king the voices tell me that i must avoid large crowds i know that you and the other nurses are trying to poison me

i am aware that each problem only has one solution

a nurse is conducting a group therapy session for several clients. the group is laughing at a joke one of the clients told, when a client who is schizophrenic jumps up and runs out of the room yelling, "you are all making fun of me!" The nurse should identify this behavior as which of the following characteristics of schizophrenia? magical thinking delusions of grandeur ideas of reference looseness of association

ideas of reference When ideas of reference are present, the client believes all events, situations, or interactions are directly related to him.

A client who has been having difficulty functioning in daily life comes to the nurse and states, "I'm really afraid. I've had these funny feelings in my stomach. I'm scared that I might have cancer." The client has been seen by numerous health care professionals and no evidence of cancer has been demonstrated. The nurse suspects what? Functional neurologic symptom disorder Factitious disorder Conversion disorder Illness anxiety disorderi

illness anxiety disorder Explanation: When individuals are fearful of developing a serious illness based on their misinterpretation of body sensations, the classification of illness anxiety disorder can be used to describe this preoccupation. The fear of having an illness continues despite medical reassurance, and this interferes with psychosocial functioning. The individual spends time and money on repeated examinations looking for feared illnesses. With factitious disorder, the illness or injury is intentionally caused to gain attention of health care workers. Functional neurologic symptom disorder or conversion disorder is a psychiatric condition in which severe emotional distress or unconscious conflict is expressed through physical symptoms.

Which of the following is best considered a "cognitive" symptom in patients with schizophrenia?

illogical thinking illogical thinking is a thought process that contains clear internal contradictions or in which conclusions are reached that are clearly erroneous, given the initial premises. This includes the thinking that "This log will help me unlock my handcuffs!"

postop intracranial surgery nursing interventions

improve cerebral perfusion - control of cerebral edema - avoid extreme head rotation as it raises ICP - turn the body as a unit to prevent strain on incision and tearing sutures - HOB elevated slowly - prevent disruption of ICP monitoring when turning pt and use a turning sheet or lift sling under pts head to midthigh to move pt regulate temp - treat fever vigorously - remove blankets - place ice packs - antipyretics improve gas exchange - reposition q2h - yawning, sighing, deep breathing, incentive spirometry, coughing (unless contraindicated) - oral suctioning (but cautiously bc it can increase ICP) - increase humidity in oxygen coping with sensory deprivation - elevating HOB to reduce periorbital edema - apply cold compresses over eyes - announce presence when entering room enhancing self-image - encourage to verbalize feelings and frustrations about change in appearance - attention to grooming - attention to use of personal clothes - covering head with a turban or wig - social interaction w friends and family monitoring and managing potential complications - increased BP and decrease in pulse can indicate increased ICP - accumulation of blood under the bone flap may pose a threat to life - a clot must be suspected in any patient who does not awaken as expected or whose condition deteriorates

intracerebral hemorrhage

into the substance of the brain causes: HTN, aneurysm, abnormalities, tumors may be due to trauma or a nontraumatic cause

a nurse is assessing a client who has a diagnosis of conversion disorder. which of the following is an expected finding? frequent manic episodes refusal of medication due to paranoia preoccupation with manifestations of various illnesses involuntary loss of sensory function

involuntary loss of sensory function

erythrocytes need ____________ for development. without this, the cells develop in a size that is too small or shallow, causing low Hgb.

iron

contact dermatitis

irritated or allergic response of the skin that can lead to an acute or chronic inflammation

autonomic reflexia

life threatening due to hypertension (its like hypertensive crisis) can be caused by distended bladder (most common), distention or contraction of visceral organs (constipation), or stimulation of skin and may occur years after the injury. symptoms: severe pounding headache, sudden increase in BP, profuse sweating, nausea, nasal congestion, and bradycardia can cause stroke or aneursym

The nurse provides care to a client who is diagnosed with major depressive disorder (MDD). The current treatment modalities of medication and psychotherapy have been ineffective. Which traditional Western medicine option should the nurse explore for inclusion in the plan of care with the client's practitioner? Select all that apply. light therapy electroconvulsive therapy (ECT) acupuncture repetitive transcranial magnetic stimulation massage therapy

light therapy electroconvulsive therapy repetitive transcranial magnetic stimulation For clients who do not respond as expected to medication and psychotherapy, other medical options are available, including ECT, light therapy, and repetitive transcranial magnetic stimulation. If the depressive episodes are cyclic and seasonal, light therapy can be very effective. Although acupuncture and massage therapy may by appropriate therapies for MDD, these are classified as alternative or complementary therapies for the treatment of depression.

a nurse is developing a plan of care for a newly admitted client who has schizophrenia and experiences frequent hallucinations and paranoid delusions. which of the following actions should the nurse plan to take? place the client in seclusion if visual hallucinations are present limit the number of questions asked during assessments use frequent touch to provide client support directly tell the client that delusions are not real

limit the number of questions asked during assessments

SSD is more common in what socioeconomic status

low socioeconomic status

Antibodies (IgE, IgD, IgG, IgM, and IgA) are formed by

lymphocytes and plasma cells

The nurse is caring for a client who was in a motorcycle accident 2 months ago. The client says he still has terrible neck pain, but he will be better once he gets "a big insurance settlement." The nurse suspects that the client is A hypochondriac Malingering Exhibiting somatization disorder Having a conversion reaction

malingering Explanation:Malingering refers to the situation in which an individual intentionally produces illness symptoms because the motivation is another specific self-serving goal, such as being classified as disabled or avoiding work. Clients with factitious disorder injure themselves covertly. The illnesses are produced in such a manner that the health care provider is tricked into believing that a true physical or psychiatric disorder is present. Factitious disorder imposed on another involves a person who inflicts injury on another person. It is commonly a mother, who inflicts injuries on her child to gain the attention of the health care provider through her child's injuries. Functional neurologic symptom disorder is a psychiatric condition in which severe emotional distress or unconscious conflict is expressed through physical symptoms.

A nurse is reviewing a client's morning laboratory results and notes a left shift in the band cells. Based on this result, the nurse can interpret that the client may be developing an infection. has leukopenia. has thrombocytopenia. may be developing anemia.

may be developing an infection. Explanation: Less mature granulocytes have a single-lobed, elongated nucleus and are called band cells. Ordinarily, band cells account for only a small percentage of circulating granulocytes, although their percentage can increase greatly under conditions in which neutrophil production increases, such as infection. An increased number of band cells is sometimes called a left shift or shift to the left. Anemia refers to decreased red cell mass. Leukopenia refers to a less-than-normal amount of white blood cells in circulation. Thrombocytopenia refers to a lower-than-normal platelet count.

what best describes a consideration when managing a schizophrenic patient with pharmacotherapy?

monitor for Neuroleptic Malignant Syndrome Patients taking antipsychotic medications can develop neuroleptic malignant syndrome, which is a severe side effect. This syndrome is composed of rigidity of muscle ("lead-pipe"), fever, encephalopathy, and unstable vital signs. Treat patients with cooling blankets and pharmacologic intervention. Mild cases are treated with bromocriptine (Parlodel), while severe cases are treated with dantrolene (Dantrium).

closed brain injury treatment

multiple types treatment based on type / severity

anemia of chronic disease

must have chronic disease of inflammation, infection, and malignancy cause this type of anemia.

treatment of huntington's disease

no cure, but medication will manage the involuntary movements. slowly progresses. succumb to illness around 10-20 years due to HF, pneumonia, infections, or fall / choking injury

natural immunity

nonspecific response to any foreign invader; Initial response (regardless of previous exposure)

allergic rhinitis: patient teaching

oInstruction to minimize allergens oUse of medications oImportance of keeping appointments oDesensitization procedures

allergic reaction

oManifestation of tissue injury resulting from interaction between an antigen and an antibody oBody encounters allergens that are types of antigens oBody's defenses recognize antigens as foreign oSeries of events occurs in an attempt to render the invaders harmless, destroy them, and remove them from the body

problem with BMA or BMB

patients are often times awake with conscious sedation and topical anesthesia patients feel pressure, burning, or stinging when they pull the plunger back to get the bone marrow

SSD in teens

pelvic / abdominal pain is normally the most common complaints

Where is blood cell formation limited to?

pelvis ribs vertebrae sternum

most common antibiotic that causes anaphylaxis

penicillin

gynecologic manifestations of HIV/AIDS

persistent vaginal candidiasis in women

secondary polycythemia vera treatment

phlebotomy

which of the following is considered a "negative" symptom in patients with schizophrenia?

poverty of thought Poverty of thought is a restriction in the amount of spontaneous speech. Patients display brief and unelaborated responses to questions, and you may have to prompt them to give you additional information.

factitious disorder imposed on self

previously known as Muchausen's Syndrome - Intentional injury/illness without obvious personal gain -Desires attention/dependent role -Poss learned behavior -Occurs covertly/hides cause of sickness

stages of HIV: stage 1

primary/acute - period from infection with HIV to the development of HIV-specific antibodies CD4 counts drop dramatically (500 or less)

Which of the following interventions is most likely to be helpful for patients with schizophrenia?

reality testing Often, it is important to follow up patients who have schizophrenia with reality testing. This helps conclude whether treatment is effective, or if delusions and hallucinations are persisting

atopy

refers to IgE-mediated diseases, such as allergic rhinitis, that have a genetic component

The nurse has completed evaluating the client's cranial nerves. The nurse documents impairment of the right cervical nerves (CN IX and CN X). Based on these findings, the nurse should instruct the client to have their spouse bring in the client's glasses. refrain from eating or drinking for now. use the walker when walking. wear any hearing aids while in the hospital.

refrain from eating or drinking for now. Explanation: Significant findings of CN IX (glossopharyngeal) include difficulty swallowing (dysphagia) and impaired taste, and significant findings of CN X (vagus) include weak or absent gag reflex, difficulty swallowing, aspiration, hoarseness, and slurred speech (dysarthria). Based on these findings, the nurse should instruct the client to refrain from eating and drinking and should contact the health care provider. The other instructions are associated with abnormalities of CN II (optic) and CN VIII (acoustic).

A nurse is caring for a client who has anemia. Diagnostic Results 0900: Hct 28% (37% to 47%)Hgb 9 g/dL (12 g/dL to 16 g/dL)WBC 6,000/mm3 (5000 to 10,000/mm3)Potassium 3.8 mEq/L (3.5 mEq/L to 5 mEq/L) Nurses Notes: 1000: One unit of packed RBCs started through a #18 g IV in right antecubital. Unit of blood infusing well. IV site dry and intact, without redness or swelling. 1015: Client appears flushed and reports new onset of low back pain and chills. Client voided 300 mL of brown-colored urine. Bilateral breath sounds clear and present throughout. Vital Signs 1000:Temperature 37.2º C (99º F)Blood pressure 104/56 mm HgHeart rate 106/minRespirations 22/minOxygen saturation 96% on room air 1015:Temperature 39.6° C (103.3° F)Blood pressure 78/50 mm HgHeart rate 122/minRespirations 28/minOxygen saturation 95% on room air select the 3 findings that require immediate follow up Oxygen saturation Report of low back pain Potassium level Urine color Temperature at 1015 Breath sounds Condition of IV site

report of lower back pain urine color temperature at 1015 Temperature at 1015 is correct. The client's temperature at 1015 is above the expected reference range. Fever is a manifestation of a transfusion reaction and requires interventions to reduce the risk for further injury. Therefore, the nurse should stop the transfusion and notify the provider. Report of low back pain is correct. Low back or flank pain is a manifestation of a transfusion reaction that requires intervention to reduce the risk for further injury. Therefore, the nurse should stop the transfusion and notify the provider. Urine color is correct. Reddish or dark brown urine are manifestations of a transfusion reaction that require intervention to reduce the risk for further injury. Therefore, the nurse should stop the transfusion, notify the provider, and send a urine sample to the laboratory.

clinical manifestations of HIV: respiratory

respiratory manifestations: - SOB, dyspnea, cough, chest pain - Pneumocystis pneumonia, Mycobacterium avium complex, Tuberculosis

GBS treatment

respiratory support, plasmapheresis, IVIG most patients recover completely

passive acquired immunity

results when a person receives protective substances produced by another human or animal (breast fed or IVIG)

long term complications of spinal cord injury

risk for autonomic reflexia (above T6 lesion)

A client reports severe pain during intercourse since being sexually assaulted three years ago. Which is the first step in confirming the diagnosis of a pain disorder? ruling out a physical cause of pain evaluating the client's understanding of a pain disorder assessing the client for posttraumatic stress disorder asking the client to keep a journal of her feelings regarding the assault

ruling out a physical cause of pain (While psychological factors have an important role in the onset, severity, exacerbation, or maintenance of the pain, initially the presence of a physical cause of the pain must be ruled out. The assessment of the patients understanding of the disorder or recording of feelings regarding the trauma are not priorities until a diagnosis of pain disorder is made. Posttraumatic stress disorder is not generally characterized with reports of sustained pain.)

acute spinal cord injury complications

spinal/neurogenic shock neuro shock thromboembolism

vector-borne encephalitis medical management

supportive care frequent assessment

The nurse is instructing a community class when a student asks, "How does someone get super strength in an emergency?" The nurse should respond by describing the action of the: sympathetic nervous system. parasympathetic nervous system. endocrine system. musculoskeletal system.

sympathetic nervous system. Explanation: The division of the autonomic nervous system called the sympathetic nervous system regulates the expenditure of energy. The neurotransmitters of the sympathetic nervous system are called catecholamines. During an emergency situation or an intensely stressful event, the body adjusts to deliver blood flow and oxygen to the brain, muscles, and lungs that need to react in the situation. The musculoskeletal system benefits from the sympathetic nervous system as the fight-or-flight effects pump blood to the muscles. The parasympathetic nervous system works to conserve body energy not expend it during an emergency. The endocrine system regulates metabolic processes.

a nurse in an impatient mental health unit is planning care for a client who is in restraints. which of the following findings should indicate to the nurse that the client is ready to reintegrate into the unit? the client's vital signs are within the expected reference range the client requests to use the bathroom the client eats all of the food provided for each of her meals the client follows directions

the client follows directions

A nurse is caring for a client who has AIDS. Nurses' Notes Client admitted to medical-surgical floor from the emergency department (ED). Client has a history of HIV, first diagnosed 15 years ago, that has recently progressed to AIDS. Client presents with headache, diarrhea, night sweats, and weight loss for approximately 1 week. Day 1: Temperature: 37.3° C (99.1° F)Heart rate: 98/minRespiratory rate: 16/minBP: 104/74 mm HgOxygen saturation: 96% on room air Day 2:Temperature 38.4 C (101.1 F)Heart rate 100/minRespiratory rate 18/minBP 98/54 mm HgOxygen saturation 95% on room air Day 1: WBC count 3,500/mm3 (5,000 - 10,000 mm3)Hemoglobin 16 g/dL (12 to 18 g/dL)Hematocrit 48% (37% to 52%)Potassium 3.7 mEq/L (3.5 to 5 mEq/L)Sodium 141 mEq/L (136 to 145 mEq/L)CD4 t-cell count 200 mm3 (800 to 1,000 mm3) Day 2: WBC count 3,100/mm3 (5,000 to 10,000 mm3)Hemoglobin 17 g/dL (12 to 18 g/dL)Hematocrit 51% (37% to 52%)Potassium 3.9 mEq/L (3.5 to 5 mEq/L)Sodium 140 mEq/L (136 to 145 mEq/L)CD4-T-cell count 198 mm3 (800 to 1000 mm3) conditions: impaired gas exchange cardiac dysrhythmias infection pneumocystis pneumonia hypokalemia findings: oxygen saturation CD4 T-cell count respiratory findings potassium level diarrhea

the client is at highest risk for developing infection due to their CD4 T cell count. Infection is correct. When using the urgent vs non-urgent approach to client care, the nurse should determine that the client is at greatest risk for infection. This client has a CD4-T-cell count of 198 - 200 mm3, indicating that the client has decreased immune function. CD4-T-cell count is correct. This client has a CD4-T-cell count of 198 - 200 mm3, indicating that the client's has decreased immune function. This decrease in the function of the client's immune system places the client at greatest risk for opportunistic infections.

coma

unarousable, unresponsive. can be this way for 2-4 weeks. movement; opens eyes.

The nurse is assessing the client's pupils following a sports injury. Which of the following assessment findings indicates a neurologic concern? Select all that apply. Pupil reaction quick Pupil reacts to light Absence of pupillary response Unequal pupils Pinpoint pupils

unequal pupils pinpoint pupils absence of pupillary response Normal assessment findings includes that the pupils are equal and reactive to light. Pupils that are unequal, pinpoint in nature, or fail to respond indicate a neurologic impairment.

Bell's Palsy Manifestations

unilateral facial muscle weakness or paralysis with facial distortion, increased lacrimation, and painful sensations in the face; may have difficulty with speech and eating

amyotrophic lateral sclerosis (ALS)

unknown loss of motor neurons - cells controlling muscles in the anterior horns of the spinal cord and lower brainstem area affects the Upper and lower motor neurons. Upper neurons motor cerebral cortex and motor info to spinal cord. Lower neurons carry from spinal cord to skeletal muscle for movement. most common between ages 40-60

akinetic mutism

unresponsiveness; this is AFTER they have been in coma for 2-4 weeks. this is NOT voluntary.

a nurse in a mental health facility is interacting with a client who is angry and becoming increasingly aggressive. which of the following actions should the nurse take? move the client to a private area so the conversation will not be disturbed use clarification to determine what the client is feeling speak to the client using an authoritative voice maintain constant eye contact with the client

use clarification to determine what the client is feeling

SSD in children

usually they only have 1 prominent complaint they are not normally worried about the severity

Which of the following is a most likely a component of the Glasgow Coma Scale (GCS)?

verbal response Verbal response is rated 1 to 5 points. Examples of how to rate verbal response: They participate in conversation and are oriented to time, place, and person (5). They are speaking understandably but some words are slurred or sluggish and they may seem disoriented (4). They think they are speaking but it isn't understandable (3). They only speak in sounds or grunts (2). They are completely mute, even with a painful stimulus (1). 5 = normal conversation, 4 = disoriented conversation, 3 = words, but not coherent, 2 = no words, only sounds, 1 = none.

causes of encephalitis

viral infections fungal infections

complications of meningitis

visual impairment deafness seizures paralysis hydrocephalus septic shock

Generalized Myasthenia Gravis

weakness of face, limbs, trunk, including muscles of breathing - dysphonia - dysphagia

Gullain-Barre Syndrome (GBS) manifestations

weakness, paralysis, paresthesia, pain, diminished or absent reflexes, starting with lower extremities and progressing upwards bulbar (respiratory) weakness, cranial nerve symptoms, tachycardia, bradycardia, hypertension, hypotension

HIV stages: stage 2

when CD4 counts are between 200 and 499

a nurse is providing teaching to a client who has schizophrenia and is to begin taking haloperidol. which of the following information should the nurse include in the teaching? you may experience dizziness upon standing while taking this medication this medication will decrease your symptoms of OCD this medication may cause excessive salivation you can stop taking the medication if the adverse effects are bothersome

you may experience dizziness upon standing while taking this medication

a nurse is caring for an adolescent client who has a new diagnosis of schizophrenia. the client's parents are tearful and express feelings of guilt. which of the following statements should the nurse make? you said that you feel guilty about your daughter's diagnosis. lets talk about what is causing you to feel this way. you should not feel guilty about your daughter's diagnosis. schizophrenia is unpreventable. im sure your daughter's diagnosis very difficult to deal with, but everything will be all right once she receives the proper treatment. your provider has explained the causes of schizophrenia. why do you feel guilty about your daughter's diagnosis?

you said that you feel guilty about you daughter's diagnosis. lets talk about what is causing you to feel this way.

a nurse is caring for a client who has schizophrenia and tells the nurse, "they lie about me all the time and they are trying to poison my food." which of the following statements should the nurse make? you are mistaken. nobody is lying about you or trying to poison you. you seem to be having very frightening thoughts. why do you think you are being lied about and poisoned? who is lying about you and trying to poison you?

you seem to be having very frightening thoughts.

management of head trauma

•Assume cervical spine injury until this is ruled out •Therapy to preserve brain homeostasis and prevent secondary damage •Treat cerebral edema •Maintain cerebral perfusion; treat hypotension, hypovolemia and bleeding, monitor and manage ICP •Maintain oxygenation; cardiovascular and respiratory function •NG to prevent aspiration; nutrition

Neutropenia (<2000)

•Decreased production or increased destruction of leukocytes •Increased risk for infection •Absolute neutrophil count (ANC): if less than 2000, you are neutropenic, normally cancer patients are less than 500.

neurogenic shock

•Due to the loss of function of the autonomic nervous system •Blood pressure, heart rate, and cardiac output decrease •Venous pooling occurs due to peripheral vasodilation •Paralyzed portions of the body do not perspire

thrombocytopenia

•Low platelet production or increased destruction of platelets •Can have HIT (Heparin Induced Thrombocytopenia) day 5 of Heparin administration treat underlying disease

Primary Polycythemia Vera

•Myeloproliferative neoplasm •Proliferative disorder: Disorder of myeloid stem cells •Ruddy complexion, splenomegaly. Increased volume., increased RBC/HCT

spinal cord injury treatment

•Proper handling •Steroids •immobilization

Disseminated Intravascular Coagulation (DIC)

•Seen in Sepsis and Trauma •Increased PT and PTT, with low fibrinogen and platelet counts •Abnormal clotting cascade •Treat cause

DIC triggers

•Sepsis •Trauma •shock cancer •abrupto placenta •Toxins •allergic reactions.

Diffuse Axonal Injury (DAI)

•Wide spread shearing/rotational forces •Prolonged traumatic coma •No lucid moment, immediate coma, posturing, and cerebral edema - outcomes vary

hemophilia

•commonly bleeding into joints. risk for compartment syndrome. Give them Factor, cold to bleeding sites •Safety issues •COLD compresses for joints there are 2 kinds and they are determined by lab

Primary Polycythemia Vera risks

•thrombosis complications (CVA, MI) •bleeding from dysfunctional platelets.

Nursing Care: Bell's Palsy

◦Provide and reinforce information and reassurance that stroke has not occurred. ◦Protection of the eye from injury; cover eye with shield at night, instruct patient to close eyelid, use of eye ointment, sunglasses. Facial exercises and massage to maintain muscle tone


Related study sets

Microeconomic Principles (Mankiw 4,5).....Final !!!!!!

View Set

2019 All Sections 1 - 15 Multiple Choice

View Set

vocabulary workshop level h unit 1-9

View Set

Series 7 Chapter 15: Ethics, Recommendations, and Taxation

View Set